Download as pdf or txt
Download as pdf or txt
You are on page 1of 214

ACE EDUCATION CHEMISTRY

O’LEVEL

Grade 10 − 12
GCSE, GCE

NSWANA CHING’AMBU

The Authorship and Career Network


authorshipandcareernetwork@gmail.com
+260976008283, +260972719373

i
The Authorship and Career Network
‘Impossibility Our Possibility’

The Authorship and Career Network is an organisation with a sole


objective of promoting excellence in research, literacy, scholarship, education
and skill development by supporting authors in Africa and beyond
to publish their works.

Published for Africa by


The Authorship and Career Network
Indeco House, Cairo Road, Lusaka,
Zambia.

Copyright © 2021 by Nswana Ching’ambu

All rights reserved. No part of this publication may be reproduced,


stored or transmitted in any form or by any means, electronic, mechanical,
photocopying, recording, scanning, or otherwise without written
permission from the publisher. It is illegal to copy this book,
post it to a website, or distribute it by any other means
without permission.

Nswana Ching’ambu
asserts the moral right as an author

First edition

ISBN: 978-9982-913-16-4

ii
To the students
Ace Education Book Series aims at giving detailed material in the simplest way to help students
understand and recall information easily. The books also highlight the importance and application of each
topic in real life so that students can understand why they are learning the material, how the material relates to
or can be used in real life.

Ace Education Chemistry O’level consists of 19 units. Each unit begins with the introduction and
overview of the unit, and ends with the review questions and solutions. To fully benefit, students are
advised to cover everything in each unit considered. Students can also learn more from our social media
platforms. On these platforms, students can find additional information, ask questions or participate in
helping other students.

With full confidence, this book will help a number of students, not just by boosting their scores, but also to
understand Chemistry Ordinary Level.

Ace Education

About the author


Nswana Ching’ambu has a bachelor’s degree in Human Biology from the University of Zambia and Teaching
Methodology from Gideon Robert University. He has been lecturing Clinical Medicine courses at Gideon
Robert University for many years. He lectures Anatomy and Physiology, Medical Biochemistry and Nutrition,
and previously used to lecture Cellular Pathology and Basic Microbiology. He is the founder of Ace
Education and the author of Ace Education Book Series for O’Levels: Biology, Chemistry, Physics and
Mathematics.
+260967744388
nswanac@gmail.com
cnswana@hotmail.com
aceeducationbooks@gmail.com

iii
TABLE OF CONTENTS

UNIT 1: PARTICULATE NATURE OF MATTER………..…………………………………………...…..1

UNIT 2: EXPERIMENTAL TECHNIQUES…………………….……………………………………..…...11

UNIT 3: METHODS OF PURIFICATION………………………………………………………….……...14

UNIT 4: ELEMENTS, COMPOUNDS AND MIXTURES………………………………………….…….22

UNIT 5: ATOMIC STRUCTURE………..…………………………………………………………....…….26

UNIT 6: CHEMICAL BONDING……………..………………………………………………….....………32

UNIT 7: CHEMICAL FORMULA AND EQUATIONS………….……………………………………….42

UNIT 8: STOICHIOMETRY AND MOLE CONCEPT……..………………………………….…………50

UNIT 9: PERIODIC TABLE……………………………………………………………………….………..76

UNIT 10: ACIDS, BASES AND ALKALIS……………………………………………………….………86

UNIT 11: SALTS………………………………………………………………………………….………….98

UNIT 12: CHEMICAL EQUATIONS………….……………………………………………….…………113

UNIT 13: ENERGY CHANGES….…………………………………………………………..……………124

UNIT 14: REDOX REACTIONS…………………...………………………………………………..…….135

UNIT 15: ELECTROLYSIS……………..…………………………………………………………….……145

UNIT 16: METALS……………………………..…………………………………………………..………166

UNIT 17: ATMOSPHERE AND ENVIRONMENT……..……………………………………………….177

UNIT 18: ORGANIC CHEMISTRY…..…………………………………………………………..………185

UNIT 19: MACROMOLECULES…………………………………………………………...………..……204

iv
1.0 PARTICULATE NATURE OF
MATTER

Introduction
All the things in the world are made up of matter. This book, the pen in your hands, your cellphone, computer,
the air we breathe, the food we eat, etc. are all made up of matter. Even our bodies, bodies of animals, plants
and stones, all consist of matter.

How do we know the properties and how matter interact with each other? It is by studying chemistry!
Chemistry is the study of matter, its composition, properties and how matters interact with each other.

So why study chemistry? As previously stated, all objects around us are made up of matter. By studying
chemistry, we come to know the composition and properties of substances and objects around us. For
example, we understand why sugar dissolves in water, but cooking oil does not, why petrol is used to power
cars, how plastics are made, etc. Chemistry is also applied in other fields such as biology, physics, medicine
and engineering.

This book introduces you to introductory chemistry. The first unit covers the states of matter. Matter is
anything that has mass and occupies space. There are states of matter, namely, solid, liquid and gas. What
happens when matter gains or loses heat? Why do we smell food from afar? This unit covers these and many
more questions.

Specific outcomes
By the end of this unit, you will be able to:
❖ State the kinetic theory of matter
❖ State the physical properties and arrangement of particles in:
• Solid
• Liquid
• Has
❖ Describe the following change of states of matter:
• Melting
• Boiling
• Condensation
• Freezing
• Sublimation
• Deposition
❖ Diffusion:
• Define diffusion
• Describe the diffusion in gas
• Describe the diffusion in liquid
• Describe the factors that affect the rate of diffusion

Particulate Nature of Matter 1


PARTICULATE NATURE OF MATTER
1.1 MATTER
• Matter is anything that has mass and occupies space.
• There are three states of matter; solid, liquid and gas.
• Kinetic theory of matter states that:
1. Matter is made up of tiny particles. These particles can be atoms, molecules or ions.
2. Particles are in constant motion.
3. The degree of movement of particles depends on their temperature.

• The state of matter of a substance depends on the arrangement and movement of particles of that
substance.
1. SOLID
❖ Physical properties of solid:
• Solids have a fixed shape and volume.
• Solids are incompressible.
• Solids do not flow.
• Solids have high density.

❖ Arrangement and movement of particles:


• Particles are tightly packed.
• Particles are arranged in a regular pattern.
• Particles vibrate at a fixed position.

2. LIQUID
❖ Physical properties of liquid:
• Liquids have a fixed volume.
• Liquids have no fixed shape.
• Liquids are incompressible.
• Liquids have high density.

❖ Arrangement and movement of particles:


• Particles are loosely packed.
• Particles have some freedom to move about.
• Particles can move from one point to another.

3. GAS
❖ Physical properties of gas:
• Gases have no fixed volume or shape.
• Gases are compressible.
• Gases have low density.

❖ Arrangement and movement of particles:


• Particles are very far apart.
• Particles freely move randomly at high speed.

2 Particulate Nature of Matter


1.2 CHANGE OF STATE OF MATTER
• A substance can change its state based on the amount of heat it gains or loses.
• During the change of state temperature of a substance remain unchanged until it changes the state.
• This is because all the heat supplied to the substance is used to break the force of attraction between
particles of the substance.
1. MELTING
• Melting is the change of state from solid to liquid.
• Particles in solid gain heat energy and increase vibrating. This results in particles being
loosely packed, changing the state of a substance from solid to liquid.
• The temperature required to change a solid substance to a liquid is called the melting point.
• Substances have different melting points. Water melts at 0oC.

2. BOILING
• Boiling is the change of state from liquid to gas.
• Particles in a liquid state gain heat energy and move randomly at high speed. Therefore,
this results in particles being further apart, changing the state of a substance to a gas.
• Particles with high kinetic energy escape from the liquid as gas.
• The temperature required to change a liquid substance to gas is called boiling point.
• Substances have different boiling points. Water boils at 100oC.
• The change of state from liquid to gas below the substance boiling point is called
evaporation.
• Evaporation occurs at the surface of a liquid.

3. CONDENSATION
• Condensation is the change of state from gas to liquid.
• Gas particles lose heat energy. As a result, they reduce their vibration and move closer to
each other, changing the substance's state from gas to liquid.
• The temperature required to change gas to liquid is called condensation point.
• Substances condense at different temperatures.
• Water condenses at 100oC, same as its boiling point.

4. FREEZING
• Freezing is the change of state from liquid to solid.
• Particles in liquid substances lose heat energy. As a result, they reduce their vibration and
become tightly packed forming a solid state.
• The temperature required to change liquid to gas is called freezing point.
• Substances freeze at different temperatures.
• Water freeze at 0oC; same as its melting point.

Particulate Nature of Matter 3


5. SUBLIMATION AND DEPOSITION
• Sublimation is the change of state directly from solid to gas.
• In sublimation, heat energy is absorbed by the substance changing the arrangement of particles of
the substance from being tightly packed to further apart.
• Example of sublimation; dry ice changes directly from solid to gas when exposed to air.
• Deposition is the change of state from gas to solid.
• In deposition, heat energy is lost from the substance changing the arrangement of particles of the
substance from being far apart to being tightly.
• Example of deposition; water vapour solidifies directly into ice during cold weather.

1.3 DIFFUSION
• Diffusion is the movement of a substance from a region of higher concentration to a region of lower
concentration.
• During diffusion, particles spread evenly to fill the environment.
• Because of diffusion, we can smell substances from afar. When we smell food, for example, the
scent of the food spread from the pan and are picked up by our nose.

❖ GAS DIFFUSION
• EXPERIMENT: DIFFUSION OF BROMINE GAS
➢ SET UP
i. Place bromine gas into a gas jar and
cover it immediately.
ii. Place another gas jar on top of a jar
with bromine.
ii. Remove the cover.

➢ RESULTS
• Reddish-brown bromine vapour spread
throughout both jar after some time.

➢ EXPLANATION
• Bromine moves from the lower gas jar, a place of higher bromine concentration, to the
upper gas jar, a place of lower bromine concentration.
• Air particles move from the upper gas jar, a place of higher air concentration, to the
lower gas jar, a place of lower air concentration.

4 Particulate Nature of Matter


❖ LIQUID DIFFUSION
• EXPERIMENT: DIFFUSION OF POTASSIUM PERMANGANATE
➢ SET UP
i. Fill water in a beaker.
ii. Add crystals of potassium permanganate.
iii. Observe what happens to the crystals and
colour change of water.

➢ RESULTS
• Potassium permanganate crystals
dissolve.
• Pink colour spread throughout the water.

➢ EXPLANATION
• Potassium permanganate particles move from the region of higher concentration to lower
concentration.
• Colour spread throughout shows an even distribution of potassium permanganate
throughout the water.

❖ FACTORS THAT AFFECT THE RATE OF DIFFUSION


1. MASS OF PARTICLE
• The larger the mass of particles the lower the rate of diffusion.
• This is so because heavy particles diffuse slowly, light particles diffuse faster.

2. TEMPERATURE
• The higher the temperature the faster the rate of diffusion.
• Higher temperature increases the movement of particles, therefore increasing the rate of
diffusion.
• Lower temperature decreases particle movement, decreasing the rate of diffusion.

3. EXTENT OF THE CONCENTRATION GRADIENT


• When one region is very concentrated while the other region is less concentrated there is rapid
diffusion of the substance from the concentrated region to the less concentrated region.
• Therefore, the greater the difference in concentration between regions, the faster the rate of
diffusion.
• On the other hand, the closer the difference in concentration between two regions the slow the
rate of diffusion.

4. SOLVENT DENSITY
• The denser the solvent, the slower the rate of diffusion.
• This is so because diffusing particles have much difficulty passing through a denser medium
than a less dense medium.

Particulate Nature of Matter 5


REVIEW QUESTIONS
1. According to the kinetic theory of matter, which substances are made of particles.
A. Gases only
B. Liquid only
C. All matter
D. All matter except solids
2. According to the kinetic theory of matter, particles of matter.
A. Are in constant motion
B. Have different shapes
C. Have different colours
D. Are always fluid
3. A kind of matter that keeps its shape is
A. Solid
B. Liquid
C. Gas
D. Air
4. A liquid has
A. Rigid shape
B. Fixed volume
C. Both rigid shape and fixed volume
D. Neither rigid shape nor a fixed volume
5. A gas
A. Has a definite volume but no definite shape
B. Has a definite shape but no definite volume
C. Has fast moving particles
D. Has particles that are always close together
6. A change of state from solid to liquid.
A. Boiling
B. Condensation
C. Melting
D. Freezing
7. A change of state from liquid to gas.
A. Sublimation
B. Boiling
C. Deposition
D. Melting
8. A change of state from gas to liquid.
A. Freezing
B. Boiling
C. Melting
D. Condensation
9. A change of state from liquid to solid.
A. Freezing
B. Sublimation
C. Melting
D. Deposition
10. A change of state directly from solids to gas.
A. Deposition
B. Boiling
C. Sublimation
D. Freezing

6 Particulate Nature of Matter


11. A change of state directly from gas to solid.
A. Melting
B. Deposition
C. Sublimation
D. Condensation
12. What happens when a liquid becomes a gas?
A. Particles give off energy
B. Particles absorb energy
C. Particles move closer together
D. Particles slow down
13. What happens when gas becomes liquid?
A. Particles give off energy
B. Particles move closer together
C. Particles slow down
D. All the above
14. Which of the following is true about a substance’s melting point?
A. It is same as its boiling point
B. It is the same as its evaporation point
C. It varies depending on temperature
D. It is the same as its freezing point
15. Which description shows the particles of copper room temperature and pressure?
A. Stationary and close together
B. Stationary and randomly arranged
C. Vibrating and in a regular arrangement
D. Vibrating and in a random arrangement
16. An inflated balloon will shrink if placed in a refrigerator. This is because the gas particles in the balloon
move.
A. Faster and become closer together
B. Faster and become further apart
C. Slower and become close together
D. Slower and become further apart
17. A soluble crystal of potassium permanganate, purple in colour is placed at the bottom of a beaker
containing water for a long period. What will be observed?
A. Colourless layer below a purple one
B. Colourless liquid with a purple crystal at the bottom
C. Deep purple layer below a light purple layer
D. Uniform purple solution
18. Diffusion is the
A. Movement of substance from a region of higher concentration to a region of lower concentration
B. Movement of substance from a region of lower concentration to a region of higher concentration
C. Movement of substance from a region of higher concentration to a region of higher concentration
D. Movement of substance from a region of lower concentration to a region of higher concentration
19. How does the size of particles affect the rate of diffusion?
A. The larger the particle the faster the rate of diffusion
B. The larger the particle the slower the rate of diffusion
C. Larger particles stop diffusion
D. The size of the particle does not affect the rate of diffusion

Particulate Nature of Matter 7


20. How does temperature affect the rate of diffusion?
A. The higher the temperature the faster the rate of diffusion
B. The higher the temperature the slower the rate of diffusion
C. Higher temperature stop diffusion
D. Temperature does not affect the rate of diffusion
21. How does concentration gradient affect the rate of diffusion?
A. The larger the concentration gradient the faster the rate of diffusion
B. The larger the concentration gradient the slower the rate of diffusion
C. Concentration gradient stop diffusion
D. Differing concentration gradient does not affect the rate of diffusion
22. How does solvent density affect the rate of diffusion?
A. The denser the solvent the faster the rate of diffusion
B. The denser the solvent the slower the rate of diffusion
C. Dense solvent stops diffusion
D. The density of the solvent does not affect the rate of diffusion
23. Explain why
i. Solids have a fixed shape
ii. Liquids flow
iii. Gases can be compressed
24. The graph below shows a heating curve for a pure substance. The temperature rises with time as the
substance is heated.

A. What physical state is the substance at points L, N and P?


B. What physical states are present at points M and O? Why is the temperature constant?
C. Describe the movement of particles in substance at points L, N and P
D. What is the melting point and boiling point of the substance?

8 Particulate Nature of Matter


25. The diagram below illustrates the experiment demonstrating the diffusion of two gases (ammonia; NH3,
molecular weight 17 g/mol and hydrochloric acid; HCL, molecular weight 36 g/mol). The white cloud of
ammonium chloride is formed after a while.

A. Why do you think the white cloud is formed nearer to the concentrated hydrochloric acid (HCl)?
B. Name two other factors that will affect the rate of diffusion.
C. Will the location of the white cloud be affected if the experiment is repeated with double the amount
of hydrochloric acid, HCl? Explain why?

26. The table below shows different gases


Name of gas Formula Molar mass (g/mol)
Nitrogen N2 28
Methane CH4 16
Sulphur dioxide SO2 64
Oxygen O2 32
Carbon monoxide CO 28

A. Name the gas which will diffuse at the


i. fastest rate
ii. slowest rate
B. Which two gases will diffuse at the same rate?
C. Name a gas that will diffuse at a faster rate than any of the gases in the table above.

Particulate Nature of Matter 9


SOLUTIONS
1. C
2. A
3. A
4. B
5. C
6. C
7. B
8. D
9. A
10. C
11. D
12. B
13. D
14. D
15. C
16. C
17. D
18. A
19. B
20. A
21. A
22. B

23. i. Solids have a fixed shape because particles in them are in a fixed position and tightly packed.
Though they vibrate, they cannot move from one part of the solid to another.
ii. Liquids flow because particles in them are farther apart and can move about.
iii. Gases can be compressed because there are large empty spaces between their particles. Therefore
particles can be brought closer by compressing the gas.

24. L: solid, N: liquid, P: gas


B. M: solid and liquid, O: liquid and gas
C. Particles in substance L are tightly packed and vibrate at a fixed position. Particles in substance
N are more loosely packed and can move about, taking the shape of the container. Particles in
substance P are very far apart and move freely taking the shape of the container.
D. Melting point is 80oC, boiling point is 160oC.

25. A. White cloud of ammonium chloride is formed near hydrochloric acid because the molecular
weight of ammonia is lower than that of hydrochloric acid. As lighter particles diffuse faster
than heavier ones, ammonia particles diffuse faster than hydrochloric acid particles.
B. Concentration of ammonia and hydrochloric acid and temperature.
C. Yes. Doubling the amount of hydrochloric acid increases its particle diffusion, the white cloud
can be formed near ammonia.

26. A. i. Methane
ii. Sulphur dioxide
B. Nitrogen and carbon monoxide
C. Hydrogen gas

10 Particulate Nature of Matter


2.0 EXPERIMENTAL TECHNIQUES

Introduction
Chemistry is a practically oriented discipline of science. It involves several practical methods which are
conducted to accomplish a particular task. To discover a new fact or demonstrate the already existing fact,
experiments are conducted. For example, to prove that salt dissolves in water an experiment can be conducted
where salt and water are mixed. To conduct the experiment students are required to know the instruments
used.

Specific outcomes
This unit covers many instruments often used in chemistry. By the end of this unit, you will be able to know
instruments for:
❖ Measuring volume
❖ Measuring mass
❖ Measuring time
❖ Measuring temperature
❖ Instruments with other functions

Experimental Techniques 11
EXPERIMENTAL TECHNIQUES
2.1 MEASURING VOLUME
• Volume is the amount of space occupied by matter.
• SI unit: cubic metre (m3)
1 litre = 1000 cm3 = 0.001 m3

1. BEAKER
• It is used to hold, mix store liquid.
• It measures liquid volume according to the graduated mark.

2. CONICAL FLASK
• It is used for holding and mixing liquid by swirling.
• It measures according to the graduated mark.

3. MEASURING CYLINDER
• It is used to measure the volume of liquid.
• It measures volume to 1 cm3 accuracy.

4. BURETTE
• It is used to measure the volume of liquid.
• It measures volume to 0.1 cm3 accuracy.

5. PIPETTE
• It is used to transfer a measured volume of liquid.

6. VOLUMETRIC FLASK
• It is used to measure the fixed amount of liquid.

7. GAS SYRINGE
• It is used to measure the volume of gas produced from the chemical reaction.

12 Experimental Techniques
2.2 MEASURING MASS OF SUBSTANCE
• Mass is the amount of matter in a substance.
• SI unit: kilogram (kg)
1 kg = 1 000 g = 1 000 000 µ
1 tonne = 1 000 kg
• Mass is measured with:
• Triple beam balance
• Electric balance

2.3 TIME
• SI unit: second (s)
• Measured with:
• Stopwatch
• Clock

2.4 TEMPERATURE
• Temperature is the measure of coldness or hotness.
• SI unit: kelvin (K)
• Degree celcius (OC) is commonly used
• K = oC + 273
• Temperature is measured with a thermometer.

2.5 OTHER INSTRUMENTS


1. FUNNEL: Used for guiding liquid or powder into a small opening.
2. SPATULA: Used for breaking, picking up and transferring solids.
3. CLAMP STAND: Support and hold other instruments.
4. TEST TUBE: Used for holding, mixing and performing chemical reactions in them.
5. BUNSE BURNER: Used for heating, sterilization and combustion.
6. GLASS ROD: Used for mixing chemicals.
7. EVAPORATING DISH: Used for evaporating excess solvent.

Experimental Techniques 13
3.0 METHODS OF PURIFICATION

Introduction
Most substances around us are not pure. Instead, they are a mixture of different substances or contain
impurities. These impurities affect their physical and chemical properties. For example, pure water boils at
100oC. However, impure water boils at a higher boiling point. To obtain a pure substance, purification is
performed. Purification is the physical or chemical removal of contaminants from a substance.

Purification is significant in real life. Though substances around us are impure, most substances need to be
purified before use. For example, petrol is purified before sent for consumption. Pharmaceutical companies
make sure that drugs sent to the market are pure as having impurities might cause a drug to be toxic or
ineffective. Most metals are mined from the earth in their impure states. To be used, they are first purified.

Specific outcomes
This unit covers the methods of purifying substances by physical means. By the end of this unit, you will be
able to describe the following separating techniques:
❖ Separating funnel
❖ Simple distillation
❖ Fractional distillation
❖ Paper chromatography

14 Methods of Purification
METHODS OF PURIFICATION
3.1 PURIFICATION
• A pure substance is a substance that has definite composition and properties which are constant
throughout the sample.
• Pure substances exhibit very well defined physical properties. For example, pure water always
melts at 0oC and boils at 100oC.
• The combination of two or more substances with different compositions forms a mixture.
• A pure substance that has other substances affecting its physical properties become an impure
substance.
• Impurities affect the physical properties of a substance. For example, impurities decrease the
melting point and increase the boiling point of water.
• Purification is the process of separating a mixture into its pure substances by physical means.
• Separation techniques depend on the type of a mixture, its composition, properties and its state.

3.2 SEPARATING SOLID/LIQUID MIXTURES


1. FILTRATION
• Filtration is a separation technique used to separate liquid from insoluble solid.
• Liquid passes through the filter paper but solid particles are trapped.
• Residue: insoluble solids that remain on the filter paper.
• Filtrate: the liquid that passes through the filter paper.

2. EVAPORATION
• Evaporation is a separation technique used to obtain crystals from solvent by heating.
• The solution is heated to evaporate solvent leaving behind a saturated solution.
• The saturated solution can no longer hold dissolved solutes any more. Therefore, solutes form
solid crystals.

Methods of Purification 15
3. CRYSTALLISATION
• Crystallisation is s separation technique used to obtain dissolved solute from its solution as crystals.
• The solution is heated until it becomes saturated.
• The saturated solution is left to cool.
• Crystals start forming in the solution as it cools.
• This is because the solubility of the solution decreases with low temperature. Therefore, dissolved
solutes form solid crystals.

3.3 SEPARATING LIQUID/LIQUID MIXTURE


❖ IMMISCIBLE LIQUIDS
• Immiscible liquids are liquids that do not mix, for example, water and oil.
• These liquids can be separated using a separating funnel.
1. SEPARATING FUNNEL
• Close the tap of the funnel.
• Pull immiscible fluid into the funnel and close with the funnel with a stopper.
• Allow fluids to settle.
• The denser liquid will sink at the bottom and the less dense floats on top.
• To separate, remove the stopper and turn on the tap. The denser liquid will pull out
leaving the less dense liquid in the funnel.

16 Methods of Purification
❖ MISCIBLE LIQUIDS
• Miscible liquids are liquids that mix together, for example, water and alcohol.
• These liquids can be separated using distillation techniques.
2. SIMPLE DISTILLATION
• Simple distillation is used to separate liquids in a mixture with a wide difference in boiling
points.
• Liquid mixtures to be separated should have different boiling points.
• The mixture is heated. A liquid with a lower boiling point evaporates as vapour, cooled in a
condenser, and formed a liquid called distillate.
• A liquid with a high boiling point remains in the flask.
• Simple distillation is useful in alcohol production industries.
• In desert areas, this technique is used to separate pure water from saltwater.

3. FRACTIONAL DISTILLATION
• Fractional distillation is used to separate liquids in a mixture with close boiling points.
• It separates a mixture into many fractions based on their boiling points.
• A fractionating column contains beads that condense vapour with a higher boiling point
back into the distillation flask while allowing vapour with a lower boiling point to escape.
• Vapour is cooled by the condenser and changes to liquid that is collected.
• It is used to separate crude oil (petroleum) into fractions, for example, petrol, diesel,
methane.

Methods of Purification 17
3.4 PAPER CHROMATOGRAPHY
• Paper chromatography is used to separate dye mixture based on solubility and adsorption.
• Solubility is the capacity of a substance to dissolve in a solvent.
• Adsorption is the capacity of a substance to stick to surfaces.
• There are two types of paper chromatography; ascending and descending paper chromatography.

❖ HOW TO PERFORM SIMPLE ASCENDING PAPER CHROMATOGRAPHY


➢ STEPS
i. Draw a horizontal line with a pencil near one end of the paper. Use a pencil because it does
not dissolve.
ii. Drop a sample of dye to be separated on the line.
iii. Put the paper in a solvent and cover it with a container to prevent evaporation of the dye.
iv. When the dye is colourless add a locating agent. A locating agent reacts with the dye to
produce a coloured dye.

➢ OBSERVATION
• As the solvent rises through the paper, dye is carried with it and produce spots of different
colours on the paper.

➢ EXPLANATION
• The dye is separated into different components due to substances in the dye having
different solubility and adsorption.
• A more soluble substance in the dye is dissolved faster and carried higher by the solvent.
• A substance in the dye with high adsorption tends to stick to the paper, therefore, move
slower on the paper.
• On the diagram above the substance that forms a purple spot has high solubility and low
adsorption therefore it moved faster and is more elevated. Conversely, a substance with a
red spot has low solubility and high adsorption therefore moved slower and it is lower.

➢ RETENTION FACTOR
• Retention factor is the ratio of the distance moved by a spot from the start line to the
distance moved by the solvent.

distance moved by spot


Rf value =
distance moved by solvent

• Example: During a paper chromatography experiment a pigment moved 3cm while the
solvent moved 4cm. Calculate the Rf value.
3cm
Rf value =
4cm
= 0.75

18 Methods of Purification
REVIEW QUESTIONS
1. You accidentally mix cooking oil and water. Which of the following separating techniques would you use
to obtain cooking oil?
A. Filtration
B. Paper chromatography
C. Separating funnel
D. Evaporation
2. Fractional distillation can be used to separate alcohol from water. This method of separation uses the fact
that alcohol and water.
A. Contain different sized molecules
B. Boil at different temperature
C. Contain different pigments
D. Burn at different temperature
3. A mixture contains two substances. What property must the two substances have for them to be separated
by paper chromatography?
A. Both substances must be soluble in the solvent
B. Both substances must have a low boiling point
C. Both substances must be coloured
D. Both substances must be solid at room temperature
4. Which of these methods would separate sand and water?
A. Crystallisation
B. Distillation
C. Filtration
D. Chromatography
5. Which of the following physical properties is a criterion when using filtration?
A. Solid substance insoluble in the solvent
B. Liquid substance immiscible with the solvent
C. Substances with different boiling points
D. Substances with different solubility
6. Salt is separated from seawater by
A. Crystallisation
B. Evaporation
C. Filtration
D. Distillation
7. A. Explain what is meant by a pure substance?
B. i. Many people know that the boiling point of water is 100oC. However, a student of Chemistry
was surprised to find that water he heated boiled at 105oC. State two possibilities that could
have caused an increase in the boiling point of water.
ii. Would you expect the water sample which boiled at 105oC to freeze below or above 0oC at the
same pressure?
C. Why is the purity of substances important in the food industry?
D. Filtration is very vital in everyday life. Give one example of the industrial application of
filtration in everyday life.

Methods of Purification 19
8. The apparatus below was used in the extraction of ethanol from fermented maize grain.

A. Name the apparatus labelled Y.


B. What improvement can be made to the above arrangement to get pure ethanol?
C. State the purpose of cold water which passes through Y, and why it has to enter through the bottom
D. Name the separation technique being used above.
E. State the industrial application for the technique shown in the diagram above.
9. The diagram below shows a chromatogram obtained during the analysis of some substances. A, B and C
are standard substances whereas P, Q and R are unknown substances.

A. Which of substances P, Q and R


i. is a pure substance
ii. is a mixture
B. Which of the standard substances can be used to make substance Q?
C. One spot of substance R moved 20 cm from baseline and the solvent front is 30 cm. Calculate the Rf
value of that spot?
D. Explain why the level of solvent is kept below the baseline at the start of the chromatographic
process.

20 Methods of Purification
SOLUTIONS
1. C
2. B
3. A
4. B
5. A
6. B

7. A. Pure substance is a substance that has definite composition and properties which are
constant throughout the sample.
B. i. Presence of impurities in water, higher atmospheric pressure.
ii. Below 0oC (impurities lower melting point, raise boiling point)
C. To make food safe for consumption.
D. Filter tea from tea leaves, tap water is filtered at the distribution centre, air conditioners filter air.

8. A. Y is the condenser.
B. By adding a fractionating column.
C. Continuous supply of cold water provides a cooler environment in the condenser that changes
vapour from the distillation flask into liquid. Water entering from the bottom aids to fill
the entire volume of the condenser with cold water for effective cooling.
D. Simple distillation.
E. Used for obtaining alcohol from fermented grains during alcohol production, obtaining
distilled water from seawater for drinking and medical purpose.

9. A. i. P
ii. Q and R
B. A and C

distance moved by spot


C. Rf =
distance moved by solvent
20cm
=
30cm
2
=
3

D. So that the sample does not dissolve in the solvent and affect the results.

Methods of Purification 21
4.0 ELEMENTS, COMPOUNDS AND
MIXTURES

Introduction
An element is a substance that cannot be broken down into smaller substances by chemical or physical means.
There are 118 known elements. Out of these, 94 are naturally occurring. The elements are represented in the
periodic table, where each element is abbreviated by a chemical symbol. Examples of elements are hydrogen
(H), oxygen (O), carbon (C), calcium (Ca) and sodium (Na).

What if two or more elements are chemically bonded? A compound is formed! A compound is therefore a
group of two or more elements chemically bonded together. For example, when one oxygen atom chemically
bonds with two hydrogen atoms, water (H2O) a compound, is formed.

A mixture is a group of two or more substances that are physically combined. Mixtures are not chemically
bonded. The air is one of the commonest mixtures around us. It consists of a mixture of gases such as oxygen
(O2), nitrogen (N2) and carbon dioxide (CO2). Elements and compounds are pure substances while mixtures
are not pure substances.

Specific outcomes
This unit differentiates elements, compounds and mixtures. By the end of this unit, you will be able to:
❖ Elements:
• Define an element
• Give examples of elements
❖ Compounds:
• Define a compound
• Give examples of compounds
❖ Mixtures:
• Define a mixture
• Give examples of mixtures
❖ Differentiate compounds and mixtures

22 Elements, Compounds and Mixtures


ELEMENTS, COMPOUNDS AND MIXTURES
4.1 ELEMENTS
• An element is a substance that cannot be broken down into smaller substances by chemical or
physical means.
• Exist as atoms or molecules:
• Atom is the smallest unit of matter that has the properties of an element. Examples of atoms:
hydrogen (H), Neon (Ne), copper (Cu), calcium (Ca).
• A molecule is a group of two or more atoms chemically bonded together. Examples of
molecules: water (H2O), hydrogen molecule (H2).
• Elements are classified as metals or non-metals:
• Hydrogen, carbon, water are examples of non-metals.
• Copper, calcium are examples of metals.

4.2 COMPOUNDS
• A compound is a group of two or more elements chemically bonded together.
• Exist as covalent compound or ionic compound:
• A covalent, or molecular, compound is a compound that results from the chemical bond of two
or more non-metals. Water (H2O) is a covalent compound as hydrogen and oxygen are non-
metals
• An ionic compound is a compound formed from the chemical bond of a metal and a non-metal.
For example, sodium chloride (NaCl), sodium is a metal and chlorine is a non-metal.
• A compound can be separated into its constituent elements by chemical means.
• The properties of a compound are different from the properties of its constituent elements.

4.3 MIXTURES
• A mixture is a group of two or more substances that are physically combined.
• Substances can be elements or compounds.
• For example, a salt solution is a mixture of salt and water that is physically combined.
• A mixture can be separated into its pure substances by physical means such as filtration, distillation.

❖ DIFFERENCES BETWEEN COMPOUND AND MIXTURE

Compound Mixture
Elements chemically bonded Component substances physically mixed
Composition of elements always the same Composition of substances can vary
Properties of compound different from its Properties of mixture same as properties
individual elements of individual substances
Individual elements can be separated by Individual substances can be separated by
chemical means physical means

Elements, Compounds and Mixtures 23


REVIEW QUESTIONS
1. Which one is a pure substance that cannot be split into two or more simpler substances by chemical
processes?
A. Atom
B. Element
C. Compound
D. Mixture
2. What is a pure substance made of two or more elements that are chemically combined?
A. Element
B. Compound
C. Mixture
D. Solution
3. Which of the following statements describes elements?
A. All of the particles in the same element are different
B. Elements can be broken down into simpler substances
C. Elements have unique sets of properties
D. Elements cannot be joined together in chemical reactions
4. A group of two or more substances that are physically combined is
A. Mixture
B. Element
C. Compound
D. Molecule
5. The following can be separated by chemical means.
A. Element
B. Mixture
C. Compound
D. All the above
6. The following can be separated by physical means.
A. Element
B. Mixture
C. Compound
D. All the above
7. Which of the following substances can be separated into pure substances by physical means?
A. Saltwater
B. Calcium
C. Water
D. Gold
8. Which one of the following is a mixture?
A. Salt
B. Iron
C. Sugar
D. Air
9. Which of the following is not true about compounds?
A. Composition of elements is always the same
B. Can be separated by chemical means
C. Elements are chemically bonded
D. The properties of the compound is the same as the properties of constituent elements
10. Which of the following is not true about mixtures?
A. Properties of mixture same as properties of individual substances
B. Composition of substances can vary
C. Substances are chemically combined
D. Can be separated by physical means

24 Elements, Compounds and Mixtures


SOLUTIONS
1. B
2. B
3. C
4. A
5. C
6. B
7. A
8. D
9. D
10. C

Elements, Compounds and Mixtures 25


5.0 ATOMIC STRUCTURE

Introduction
Everything you are seeing; this book, pen, pencil, cellphone, computer, table, chair, a class block and so on
consists of tiny particles called atoms. These atoms are the basic unit of matter. They vary in their mass. What
is the composition of atoms? How do you calculate their mass? This unit gives an introduction to atomic
structure and covers these questions.

Specific outcomes
By the end of this unit, you will be able to:
❖ Differentiate electron, proton and neutron on charge and mass
❖ Define atomic number
❖ Define mass number
❖ Write the chemical symbol of the given element
❖ Write the electronic configuration of the given element
❖ Define an isotope, and give examples of isotopes
❖ State uses of isotopes

26 Atomic Structure
ATOMIC STRUCTURE
5.1 SUBATOMIC PARTICLES
• Atom is the smallest unit of matter with the properties of an element.
• An atom contains three particles that are responsible for the mass of the atom and charge:
i. Proton
ii. Neutron
iii. Electron
• Protons and neutrons are found in the nucleus of an atom located at the centre. Thus, the nucleus of
an atom is called nucleon.
• Electrons orbit the nucleus in electron shells.
• Protons are positively charged, electrons are negatively charged and neutrons are neutral.
Therefore, an atom has a net charge of zero. Therefore, it is said to be neutral.
• The mass of proton and electron is one atomic mass unit (amu) respectively. However, the mass of
an electron is too small therefore it does not contribute to the mass of an atom.
• The structure and characteristics of an atom are shown below.

5.2 ATOMIC MASS


• The mass of an atom is calculated from the number of protons and neutrons.
• The mass of electrons are too small, therefore neglected.
• Atomic number (also called proton number) is the number of protons in an element. It distinguishes
one element from another. It is used to put elements in the periodic table in a logical order.
• Mass number (also called nucleon number) is the total number of protons and neutrons of an
element. This is the mass of an element.

5.3 CHEMICAL SYMBOL


• Each element is represented by a unique chemical symbol.
• The atomic number is denoted by Z.
• The mass number is denoted by A.

X
Mass number A
Chemical symbol of an element
Atomic number Z

• For example, oxygen (with 8 protons and 8 neutrons) is represented by .

Atomic Structure 27
5.4 ELECTRONIC CONFIGURATION
• Electrons are placed in orbits called shells.
• The first shell can contain up to a maximum of 2 electrons. This refers to the duplet or duet rule.
• The second and third shells can contain up to a maximum of 8 electrons. This refers to the octet
rule.
• Electrons that are found in the outermost shell are called valence electrons.
• Valence electrons determine the chemical properties of an atom.
• Valence electrons are also responsible for the chemical bond formation of an atom with other
atoms.
• For example, chlorine has 17 electrons.
• Electronic configuration, of chlorine, for example, is written as 2,8,7 with each comma separating
one shell from another.

5.5 ISOTOPE
• Isotopes are atoms of the same element with different mass number.
• They have the same number of protons and electrons but a different number of neutrons.
• For example, carbon-12, carbon-13 and carbon-14 isotopes. All have 6 protons and electrons but 6,
7 and 8 neutrons respectively.
• Hydrogen-1 (protium), hydrogen-2 (deuterium) and hydrogen-3 (tritium) isotopes, all have 1 proton
and electron but 0, 1 and 2 neutrons respectively.

❖ PROPERTIES OF ISOTOPES
• Isotopes of the same element have the same chemical properties and slightly different physical
properties.
• Isotopes can be radioactive. They emit radiations in the form of alpha, beta or gamma radiation.
Gamma radiation can penetrate and destroy body cells, therefore used to treat cancer patients by
killing cancer cells.

28 Atomic Structure
REVIEW QUESTIONS
1. The three subatomic particles that make up an atom are
A. Proton, electron, nucleus
B. Proton, neutron, nucleus
C. Proton, neutron, electron
D. Neutron, electron, nucleus
2. Which subatomic particle has no charge
A. Proton
B. Electron
C. Neutron
D. Nucleus
3. Which subatomic particle has a negative charge
A. Proton
B. Electron
C. Neutron
D. Nucleus
4. Which subatomic particle has a positive charge
A. Proton
B. Electron
C. Neutron
D. Nucleus
5. Nucleon refers to
A. Protons and neutrons
B. Protons and electrons
C. Electrons and neutrons
D. Electrons only
6. Electrons present in the outermost shell are called
A. Orbit electrons
B. Octet electrons
C. Duplet electrons
D. Valence electrons
7. Which of the following statements accurately describes the masses of subatomic particles?
A. Mass of 1 proton equals the mass of 1 electron
B. Mass of 1 neutron equals the mass of 1840 proton
C. Mass of 1 neutron equals the mass of 1 proton
D. Mass of 1 electron equals the mass of 1840 neutron
8. Which of the following is used to identify an element?
A. Number of electrons
B. Number of nucleons
C. Number of nuclei
D. Number of protons
9. The atomic number is same as
A. Neutron number
B. Nuclei number
B. Proton number
C. Electron number
10. The electronic configuration of calcium is
A. 2,8,8
B. 2,8,2
C. 2,8,8,2
D. 2,8,7

Atomic Structure 29
11. The isotope deuterium of hydrogen has.
A. One proton and no neutron
B. One proton and one neutron
C. One neutron and two protons
D. One neutron and no proton
12. Draw an electronic configuration of magnesium.
13. The diagram below shows an electronic configuration of a particular element X.

A. How many valence electrons do element X has?


B. Write its electronic configuration.
C. If element X is a neutral atom and has 6 neutrons, what is its
i. atomic number.
ii. mass number.
D. What element is X?

30 Atomic Structure
SOLUTIONS
1. C
2. C
3. B
4. A
5. A
6. D
7. C
8. D
9. B
10. C
11. B

12. Electronic configuration of magnesium

13. A. 4 valence electrons


B. 2,4
C. i. The atomic number is 6. Since this element is neutral its number of electrons equals the number
of protons
ii. Mass number is 6
D. Carbon

Atomic Structure 31
6.0 CHEMICAL BONDING

Introduction
All things around us are made of tiny indivisible units called atoms. This book, your smartphone, computer,
pen, class block, trees and even us humans consists of these atoms. But how do these tiny units make a
complex object? First, they are joined together chemically! This is called chemical bonding. Then, why do
they bond? In simple terms, they do so to become stable.

There are three types of chemical bonding; namely, ionic bonding, covalent bonding and metallic bonding.
This unit covers these types of chemical bonding.

Specific outcomes
By the end of this unit, you will be able to:
❖ Ion:
• Define an ion
• Differentiate between cation and anion
❖ Ionic bonding:
• Define ionic bonding
• Represent ion bonding of atoms by dots and cross diagram
• State the properties of ionic compounds
❖ Covalent bonding:
• Define covalent bonding
• Represent covalent bonding of atoms by dots and cross diagram
• State the properties of covalent compounds
❖ Metallic bonding:
• Define metallic bonding
• State the properties of metallic compounds

32 Chemical Bonding
CHEMICAL BONDING
6.1 IONS
• Atoms with incomplete outer electron shells are unstable. Therefore, they gain or lose electrons to
attain a full outer electron shell.
• Atoms with 4 to 7 electrons in the outer shell tend to gain electrons that can add up to 8 electrons:
• For example, the oxygen atom has 6 electrons in the outermost shell. It gains 2 electrons to
attain 8 stable outer shell electrons.
• Chlorine atom has 7 electrons in the outer shell. It gains 1 electron to be stable.
• Atoms with 1 to 3 electrons in the outer shell tend to lose electrons in the outer shell to be stable:
• For example, magnesium has 2 electrons in the outermost shell. It loses 2 electrons and remains
with a full and stable lower outer shell.
• Sodium has 1 electron in the outer shell. It loses 1 electron to be stable.
• An exception is the hydrogen atom. It has 1 electron in the shell. It can gain 1 electron to have 2
stable electrons in the shell or can lose its electron and still be stable.
• An atom that has gained or lost electrons is called an ion.
• This atom becomes electrically charged.

❖ TYPES OF IONS
1. CATION
• Cations are ions that carry a positive charge.
• It indicates a loss of electrons.
• They are mainly metals.
• For example, Ca2+ means calcium atom has lost 2 electrons.
• Li+ means lithium has lost 1 electron.
• Remember; caTion has T, therefore posiTively charged ion.

2. ANION
• Anions are ions that carry a negative charge.
• It indicates a gain of electrons.
• They are mainly non-metals.
• For example, O2− means oxygen has gained 2 electrons.
• Cl- means chlorine has gained 1 electron.
• Remember; aNion has N, therefore Negatively charged ion.

6.2 IONIC BONDING


• Ionic bonding is the type of bonding that involves the transfer of electrons from one atom to
another.
• It occurs between metals and non-metals.
• Metals lose electrons and become positively charged.
• Non-metals gain electrons and become negatively charged.
• Bonding can be represented by dot and cross diagrams.

❖ FORMATION OF SODIUM CHLORIDE (NaCl)


• One electron is transferred from sodium atom to chlorine atom.
• Therefore, each sodium atom loses an electron and become positively charged while each
chlorine atom gains an electron and become negatively charged.

Chemical Bonding 33
❖ FORMATION OF MAGNESIUM CHLORIDE (MgCl2)
• Two electrons are transferred from one magnesium to two chlorine atoms.
• Each magnesium atom loses two electrons and becomes positively charged while each chlorine
atom gains one electron and become negatively charged.

❖ CRYSTAL LATTICE
• Ionic substances form a giant crystal lattice.
• Opposite charges attract each other.
• A group of positively and negatively charged ions packed closely to each other form a regular
crystal shape.
• For example, a group of Na+ and Cl- ions assemble closely together and form a NaCl crystal the
common salt.

❖ PROPERTIES OF IONIC COMPOUNDS


1. They form hard crystalline solid. Ionic compounds have cations and anions closely packed and
alternating with each other forming three-dimensional regular crystals.
2. They have a high melting point and boiling point. The Force of attraction between positively and
negatively charged ions is strong. Therefore, a large amount of heat energy is required to separate
them.
3. They conduct electricity in liquid form. This is because dissolved ionic compounds have free ions
that are free to move about carrying electricity. Solid ionic compounds have fixed ions, therefore,
cannot conduct electricity.
4. They are soluble in water. Water being a polar molecule, oxygen slightly positively charged and
hydrogen slightly negatively charged can dissolve ionic compounds into separate ions. The
negatively charged ion is attracted to oxygen and the positively charged ion is attracted to
hydrogen. However, some ionic compounds are insoluble in water.

34 Chemical Bonding
6.3 COVALENT BONDING
• Covalent bonding is a type of bonding formed by sharing of electrons between atoms or molecules.
• It occurs between non-metals.
• An atom or molecule shares its electrons with another atom for both to attain a full outer electron
shell.
• The following are examples of covalent bonds.

❖ FORMATION OF HYDROGEN MOLECULE

❖ FORMATION OF WATER MOLECULE

❖ FORMATION OF METHANE

Chemical Bonding 35
❖ FORMATION OF OXYGEN MOLECULE

❖ FORMATION OF NITROGEN MOLECULE

❖ PROPERTIES OF COVALENT BOND


1. They have low boiling and melting. Covalent bonds are very strong bonds; however, the
intermolecular bonds, bonds between separate covalent compounds, are weak bonds and a low
amount of heat energy is required to break them. The exceptional are giant covalent compounds
like diamond and graphite. These compounds have a very high boiling and melting point.
2. They do not conduct electricity. In solid or liquid state covalent compounds do not have free
electrons or ions to conduct electricity.
3. They are insoluble in water. Covalent compounds do not have charges. They are neural or nonpolar
molecules. Water molecules do not attract nonpolar molecules. Therefore, they cannot be dissolved
by water. However, some covalent compounds are soluble in water.
4. They exist as gas, liquid or soft solid at room temperature. This is due to their weak intermolecular
force of attraction.

36 Chemical Bonding
6.4 METALLIC BONDING
• Metallic bonding is the type of bonding that involves sharing of electrons between atoms of
metallic elements.
• Each metal atom loses electrons and becomes positively charged.
• This forms a structure of positively charged ions in a sea of delocalised electrons.
• Metals are held by a strong force of attraction between delocalised or free electrons and positively
charged ions.
• They form a giant lattice structure.

❖ PROPERTIES OF METALLIC COMPOUNDS


1. They have a high melting point and boiling point. The force of attraction between metal ions and
delocalised electrons is very strong. Therefore, a high amount of heat energy is required to break
the bond.
2. They are good conductors of electricity. Electricity is carried by free electrons.
3. They are good conductors of heat. Heat is carried by free electrons.
4. They can be bent into a thin wire, a property called ductility and can be stretched into and
shape, a property called malleability. The ductility and malleability of metals are due to the
nature of the force of attraction between metal ions and free electrons. Layers of metal ions can
slide over each other without breaking the force of attraction between metal ions and free
electrons.

Chemical Bonding 37
REVIEW QUESTIONS
1. An element that has gained or lost electron is called
A. Cation
B. Anion
C. Ion
D. Molecule
2. When an atom loses an electron, it becomes
A. Cation
B. Anion
C. Neutral
D. Molecule
3. When an atom gains an electron, it becomes
A. Cation
B. Anion
C. Neutral
D. Molecule
4. What is the formula of the ion when potassium achieves a full outer electron shell?
A. K2+
B. K+
C. K−
D. K2−
5. What is the name of the ion when chlorine achieves a full outer electron shell?
A. Cl2+
B. Cl+
C. Cl−
D. Cl2−
6. The bond that involves the transfer of electrons from one atom to another is
A. Metallic bond
B. Ionic bond
C. Covalent bond
D. All the above
7. The bond that involves sharing of electrons between atoms of metal is
A. Metallic bond
B. Ionic bond
C. Covalent bond
D. All the above
8. The bond that involves the share of electrons between non-metals is
A. Metallic bond
B. Ionic bond
C. Covalent bond
D. All the above
9. The bond in MgO is
A. Covalent bond
B. Ionic bond
C. Metallic bond
D. None of the above
10. All of the following are properties of the ionic compounds except
A. They have low melting and boiling point
B. They conduct electricity in molten form
C. They are soluble in water
D. They form regular solids

38 Chemical Bonding
11. Why ionic compounds conduct electricity in liquid form
A. Because positively and negatively ions are closely packed
B. Because the force of attraction between positively charged and negatively ions is strong
C. Because they have free electrons
D. Because the ions can be attracted to oxygen and hydrogen atoms of water
12. Why ionic compounds have high melting and boiling point
A. Because positively and negatively ions are closely packed
B. Because the force of attraction between positively charged and negatively ions is strong
C. Because they have free electrons
D. Because the ions can be attracted to oxygen and hydrogen atoms of water
13. The bond in the oxygen molecule (O2) is
A. Covalent bond
B. Ionic bond
C. Metallic bond
D. None of the above
14. All of the following are properties of covalent compounds except
A. They have high melting and boiling point
B. They are insoluble in water
C. They conduct electricity
D. They do not conduct electricity
15. The bond in the copper wire is
A. Covalent bond
B. Ionic bond
C. Metallic bond
D. None of the above
16. All of the following are properties of metallic compounds except
A. They have high melting and boiling point
B. They are ductile
C. They conduct electricity
D. They do not conduct heat
17. Why metallic compounds are good conductors of electricity
A. Because they have a sea of free electrons
B. Because the force of attraction between metal ions and electrons is strong
C. Because metal ions can slide over each other without breaking the metallic bond
D. All the above
18. Using dot and cross diagrams, show the structures of the following compounds (show all shells)
A. Magnesium oxide
B. Water
C. Oxygen
19. Two elements X and Y represented by the notations below reacted to form a compound.

A. What changes in electronic structures occurred when atoms X and Y reacted?


B. What type of bonding occurs between X and Y?
C. Draw the structure of the compound formed between X and Y (show outer shell only).
D. What you expect the melting point of the compound formed to be high or low. Give a reason for your
answer.
E. Using the actual chemical symbols for X and Y, write down the chemical formula of the compound
formed.
20. State four differences between ionic bonding and covalent compounds.

Chemical Bonding 39
SOLUTIONS
1. C
2. A
3. B
4. B
5. C
6. B
7. C
8. B
9. B
10. A
11. C
12. B
13. A
14. C
15. C
16. D
17. A

18. A.

B.

C.

40 Chemical Bonding
19. A. X loses one electron from its outermost shell. Y gains two electrons to its outermost shell.
B. Ionic bonding
C.

D. The compound formed has a high melting point. Because the force of attraction between ions X+ and
Y2− is strong. Therefore, a large amount of heat is required to break the bond.
E. Na2O

20. i. Ionic compounds result from the transfer of electrons while covalent compounds from sharing
of electrons.
ii. Ionic compounds have high melting and boiling points while covalent compounds have low
melting and boiling point.
iii. Ionic compounds dissolve in water while covalent compounds do not.
iv. Ionic compounds conduct electricity in liquid form while covalent compounds do not in any
form.

Chemical Bonding 41
7.0 CHEMICAL FORMULA AND
EQUATIONS

Introduction
Unit the previous unit, you learnt that elements chemically join together to form a compound. The compound
formed is represented by a chemical formula. How do you write the correct chemical compound?

Substances that are used up in a chemical reaction are called reactants. Substances that are produced at the end
of the reaction are called products. A chemical equation shows the chemical formula of reactants and products
in a chemical reaction. As no atom is lost or created in a chemical reaction, the number of the same atoms in
the reactants and products must be equal. This is referred to as a balanced chemical reaction. How do you
write a balanced chemical reaction? This unit covers these and many more questions.

Specific outcomes
By the end of this unit, you will be able to:
❖ Write chemical symbols of elements
❖ Valency:
• Define valency
• State the valency of atoms and compounds
❖ Write the chemical formula of compounds
❖ Write balanced chemical equations

42 Chemical Formula and Equations


CHEMICAL FORMULA AND EQUATIONS
7.1 CHEMICAL SYMBOL
• Each element has its unique chemical symbol contain one or two letters.
• The chemical symbol of an element is derived from the name of an element or its Greek or Latin
name.
• The table below shows the first 20 elements and their chemical symbols.
Element Chemical symbol
Hydrogen H
Helium He
Lithium Li
Beryllium Be
Boron B
Carbon C
Nitrogen N
Oxygen O
Fluorine F
Neon Ne
Sodium Na
Magnesium Mg
Aluminium Al
Silicon Si
Phosphorus P
Sulphur S
Chlorine Cl
Argon Ar
Potassium K
Calcium Ca

7.2 VALENCY
• Valency is the combining capacity of an element.
• Hydrogen and chlorine can combine with only one atom. Therefore, they have a valency of two.
• Based on the valency of hydrogen and chlorine, the valency of any element can be defined as the
number of hydrogen or chlorine atoms with which an element can combine with.
• For metals, valency is equal to the number of electrons in the outermost shell.
• For non-metals, valency is equal to 8 minus the number of electrons in the outermost shell.
• Below is the valency of elements according to the groups of the periodic table.
Group 1 2 3 4 5 6 7 8
valency 1 2 3 4 3 2 1 0

Chemical Formula and Equations 43


7.3 CHEMICAL FORMULA
• Elements combine chemically to form a compound.
• Writing a compound can be simplified into a chemical formula. For example, when oxygen and
hydrogen combine chemically water is formed. Therefore, water can be written in the chemical
formula as H2O.

❖ HOW TO WRITE A CHEMICAL FORMULA


1. Write down the name of the compound, for example, water.
2. Write down the chemical symbol of each element in the compound, for example, H O.
3. Write the valency of each element on top using the group number of the periodic table,
for example, H1 O2.
4. Cross over the valency number of elements in step three; for example, H1 O2 become H2O.
5. Reduce the number if necessary.

❖ EXAMPLES
1. Write the chemical formula of the following compounds.
A. Aluminium chloride
B. Magnesium oxide

❖ SOLUTIONS
A. Aluminium chloride
1. Aluminium chloride
2. Al Cl
3. Al3 Cl1 aluminium is in group 3 and chlorine in group 7
4. AlCl3

B. Magnesium oxide
1. Magnesium oxide
2. Mg O
3. Mg2 O2 magnesium is in group 2 and oxygen in group 6
4. Mg2O2
5. MgO reduced by cancelling 2 on both elements

❖ WRITING COMPLEX CHEMICAL EQUATIONS


• Some compounds contain unpaired electrons. These compounds are called radicals.
• These compounds have charges.
• Examples of radicals include; sulphate (SO42−), nitrate (NO3−), ammonium (NH4+).
• These compounds are very reactive. For example, sodium (Na) can react with sulphate (SO42−)
to form sodium sulphate (Na2SO4).
• Writing chemical formula for the compound formed between an element and a radical involve
the same rules for simple compounds.

❖ EXAMPLES
1. Write the chemical formula for a compound formed in the following reactions between an
element and a radical.
A. Chlorine and ammonium
B. Sodium and sulphate
C. Calcium and nitrate

44 Chemical Formula and Equations


❖ SOLUTIONS
A. Chlorine and ammonium
1. Ammonium chloride
2. NH4− Cl NH4− valency is 1; master valences of common radicals
1 1
3. NH4 Cl chlorine is in group 7
4. NH4Cl

B. Sodium and sulphate


1. Sodium sulphate
2. Na SO42−
3. Na1 SO42 sodium is in group 1, SO42− valency is 2; master valences of common radicals
4. Na2SO4 do not reduce to NaSO2 as SO4 act as a single compound

C. Calcium and nitrate


1. Calcium nitrate
2. Ca NO3−
3. Ca2 NO31 calcium is in group 2, NO3− valency is 1; master valences of common radicals
4. Ca(NO3)2

7.4 CHEMICAL EQUATIONS


• A chemical equation shows the chemical formula of reactants and products in a reaction.
• It consists of the chemical formula of reactants and products, yield arrow, state symbols and
coefficient.
• Yield arrow ( ) separate reactants and products. It shows the direction of the reaction. A
reversible reaction that goes forward and backward is shown by a double arrow (⇋).
• State symbols are written as (s), (l) and (g) for solid, liquid and gas respectively. Substances
dissolved in water, in other terms, in an aqueous solution are represented by the (aq) symbol.
• A coefficient is a number in front of the chemical formula. It shows the number of molecules of the
substance that are involved in the reaction.
• For example, hydrogen and oxygen react to form water. This reaction can be written in chemical
equations as written below.

7.5 BALANCING CHEMICAL EQUATIONS


• Atoms cannot be created or destroyed. Therefore, the number of atoms on reactants must be equal
to the number of atoms on products.
• The following are steps involved when balancing a chemical equation:
1. Write the unbalanced equation.
2. Write down the number of atoms used and produced for each element.
3. Find the coefficient to balance the equation, dealing with one atom at a time. Usually, it is best
to start with the most complex compound.
4. Reduce the coefficient if necessary.

Chemical Formula and Equations 45


❖ EXAMPLES
Balance the following reactions
A. H2 + O2 → H2O
B. C3H8 + O2 → CO2 + H2O
C. Na3PO4 + CaCl2 → Ca3(PO4)2 + NaCl

❖ SOLUTIONS
A. 1. H2 + O2 → H2O
2. O left side 2 : right side 1
H left side 2 : right side 2
3. H2 + O2 → 2H2O balance O by adding 2 on H2O
2H2 + O2 → 2H2O balance H by adding 2 on H2

B. 1. C3H8 + O2 → CO2 + H2O


2. C 3 : 1
H 8:2
O 2:3
3. C3H8 + O2 → 3CO2 + H2O balance C by adding 3 on CO2
C3H8 + O2 → 3CO2 + 4H2O balance H by adding 4 on H2O
C3H8 + 5O2 → 3CO2 + 4H2O balance 5 on O2

C. 1. Na3PO4 + CaCl2 → Ca3(PO4)2 + NaCl


2. Na 3 : 1
P 1:2
O 4:8
Ca 1 : 3
Cl 2 : 1
3. Na3PO4 + 3CaCl2 → Ca3(PO4)2 + NaCl balance Ca by adding 3 on CaCl2
2Na3PO4 + 3CaCl2 → Ca3(PO4)2 + NaCl balance P by adding 2 on Na3PO4
2Na3PO4 + 3CaCl2 → Ca3(PO4)2 + 6NaCl balance Na and Cl by adding 6 on NaCl

46 Chemical Formula and Equations


REVIEW QUESTIONS
1. Write the chemical formula of the following compounds.
A. Sodium chloride
B. Carbon dioxide
C. Lithium oxide
D. Methane (has carbon and hydrogen atoms)
E. Calcium oxide
F. Magnesium chloride
G. Zinc oxide
2. Write the chemical formula of the compound that contains a radical.
A. calcium carbonate
B. sodium bicarbonate
C. sulphuric acid (has hydrogen and sulphate)
D. sodium hydroxide
E. copper sulphate
F. ammonium sulphate
G. sodium phosphate
3. Balance the following chemical equations.
A. Zn + HCL → ZnCl2 + H2
B. Na + H2O → NaOH + H2
C. Fe2O3 + C → Fe + CO2
D. Pb(NO3)2 + NaCl → NaNO3 + PbCl2
E. NaHCO3 → NaCO3 + CO2 + H2O

Chemical Formula and Equations 47


SOLUTIONS
1. A. Na1 Cl1 Na is in group 1; Cl in group 7
NaCl

B. C4 O2 C is in group 4; O in group 2
C2O4
CO2

C. Li1 O2 Li is in group 1; O in group 6


Li2O1
Li2O

D. C4 H1 C is in group 4; H in group 1
CH4

E. Ca2 O2 Ca is in group 2; O in group 6


Ca2O2
CaO

F. Mg2 Cl1 Mg is in group 2; Cl in group 7


MgCl2

G. Zn2 O2 Zn valency is 2; O is in group 2


Zn2O2
ZnO

2. A. Ca2 CO32− Ca valency is 2; CO32− valency is 2


Ca2(CO3)2
CaCO3

B. Na1 HCO3− Na valency is 1; HCO3− valency is 1


NaHCO3

C. H1 SO42− H valency is 1; SO42− valency is 2


H2SO4

D. Na1 OH− Na valency is 1; OH− valency is 1


NaOH

E. Cu2 SO42− Cu valency is 2; SO42− valency is 2


Cu2(SO4)2
CuSO4

F. NH4+ SO42− NH4+ valency is 1; SO42− valency is 2


(NH4)2SO4

G. Na1 PO43− Na valency is 1; PO43− valency is 3


Na3PO4

48 Chemical Formula and Equations


3. A. Zn + 2HCl → ZnCl2 + H2

B. 2Na + 2H2O → 2NaOH + H2

C. Pb(NO3)2 + 2NaCl → 2NaNO3 + PbCl2

D. 2NaHCO3 → Na2CO3 + CO2 + H2O

Chemical Formula and Equations 49


8.0 STOICHIOMETRY AND MOLE
CONCEPT

Introduction
Stoichiometry is the study of the relation between reactants and products of chemical reactions. Chemical
reactions must be balanced. In other words, the number of the same atoms in the reactants and products must
be equal. Therefore, when the amount of reactants are known, using stoichiometric calculations we can find
the amount of products formed and vice versa.

We use stoichiometry in our everyday lives especially when cooking. We add correct ratios of ingredients to
make a good meal. Pharmaceutical companies use the right amount of elements to make a drug. Food
processing industries use the correct amount of raw materials to make good and healthy foods.

Specific outcomes
This unit covers most stoichiometric calculations. By the end of this unit, you will be able to:
❖ Differentiate relative atomic mass, relative molecular and relative formula mass
❖ Work out mole − number of particles calculations
❖ Find the molar mass of a compound
❖ Find the percentage composition of an element in a compound
❖ Find the molecular formula of a compound
❖ Find the empirical formula of a compound
❖ Calculate the volume of gases at room temperature and pressure
❖ Calculate the concentration of a substance
❖ Work out dilution calculations
❖ Find the limiting reagent
❖ Calculate the percentage yield
❖ Calculate the percentage purity

50 Stoichiometry and Mole Concept


STOICHIOMETRY AND MOLE CONCEPT
8.1 RELATIVE ATOMIC MASS
• Relative atomic mass (Ar) is defined as the average mass of one atom compared to 1/12 of the mass
of carbon-12 atom.
• Hydrogen is the lightest element. Using hydrogen mass, carbon has the same mass as 12 hydrogen
atoms. Therefore, carbon has a relative atomic mass of 12.
• The relative atomic mass of all elements is stated in the periodic table.
• Formula:

average mass of one atom


Ar = 1 where Ar = relative atomic mass
12
× mass of carbon-12

8.2 RELATIVE MOLECULAR MASS


• Relative molecular mass (Mr) is defined as the average mass of one molecule of a substance
compared to 1/12 of the mass of carbon-12.
• Formula:

total mass of all atoms


Mr = 1 where Mr = relative molecular mass
× mass of carbon-12
12

• It is simply the sum of all relative atomic masses (Ar) in a molecule.


• It is calculated by adding masses (Ar) of individual atoms in a molecule.

❖ EXAMPLES
1. Find the relative molecular mass of the following molecules
A. H2
B. O2
C. H2O
D. CO2
E. C2H6

❖ SOLUTIONS
1. A. H2 = 1 × 2 = 2 Ar of H = 1
B. O2 = 16 × 2 = 32 Ar of O = 16
C. H2O = (1 × 2) + 16 = 18
D. CO2 = 12 + (16 × 2) = 44 Ar of C = 12
E. C2H6 = (12 × 2) + (1 × 6) = 30

Stoichiometry and Mole Concept 51


8.3 RELATIVE FORMULA MASS
• Relative formula mass is used to calculate the mass of compounds, both molecular compounds and
ionic compounds.
• It is similar to relative molecular mass, except for relative molecular mass is used for molecular
compounds but relative formula mass for both molecular compounds and ionic compounds.
• It uses the same symbol as relative molecular mass (Mr).
• It is calculated by adding masses of individual atoms or ions in a compound.

❖ EXAMPLES
1. Calculate the relative formula mass of the following compounds
A. HCl
B. NaCl
C. MgSO4
D. Na2CO3
E. Pb(NO3)2
F. (NH4)2SO4

❖ SOLUTIONS
1. A. HCl = 1 + 35.5 = 36.5 Ar of H = 1; Ar of Cl = 35.5
B. NaCl = 23 + 35.5 = 58.5 Ar of Na = 23
C. MgSO4 = 24 + 32 + (16 × 4) = 120 Ar of Mg = 24; S = 32; O = 16
D. Na2CO3 = (23 × 2) + 12 + (16 × 3) = 106 Ar of C = 12
E. Pb(NO3)2 = 207 + (14 × 2) + (16 × 6) = 331 Ar of Pb = 207; N = 14
F. (NH4)2SO4 = (14 × 2) + (1 × 8) + 32 + (16 × 4) = 132

8.4 MOLE
• A mole is a unit of particles.
• It corresponds to 6 × 1023 particles called Avogadro’s constant.
• Unit: mol.
• Just like a dozen correspond to 12 items of anything, it can be 12 books, 12 pens, 12 bricks, 12 bags
of cement. Of course, these items have different masses but they are 12 items. This concept is the
same as a mole of a substance.
• One mole of a substance contains 6 x 1023 particles. It can be atoms, molecules or ions.

❖ MOLE – NUMBER OF PARTICLES CONVERSION


• When given the number of particles of a substance, a mole can be calculated.
• When given the mole of a substance, the number of particles can be calculated.
• The formula for calculating the number of moles of a substance is given as:

number of particles
n= where n = mole of a substance
6 × 1023

❖ EXAMPLES
1. How many molecules are in 2 moles of hydrogen gas?
2. How atoms are present in 34.7 moles of magnesium?
3. How many moles are in 2.0 × 1025 molecules of silver nitrate?
4. How many moles are present in 4.09 × 1024 atoms of manganese?

52 Stoichiometry and Mole Concept


❖ SOLUTIONS
number of particles Mole = 2
1. n =
6 × 1023 # of particles = ?
number of particles = n × 6 × 1023 Avogadro’s constant = 6 × 1023
number of particles = 2 × 6 × 1023
number of particles = 1.2 × 1024

number of particles
2. n = Mole = 34.7
6 × 1023
number of particles = n × 6 × 1023 # of particles = ?
number of particles = 34.7 × 6 × 1023 Avogadro’s constant = 6 × 1023
number of particles = 2.08 × 1025

number of particles
3. n = Mole = ?
6 × 1023
2.0 × 1025 # of particles = 2.0 × 1025
n= Avogadro’s constant = 6 × 1023
6 × 1023
n = 33.3 mol

number of particles
4. n = Mole = ?
6 × 1023 # of particles = 4.09 × 1024
4.09 × 1024
n= Avogadro’s constant = 6 × 1023
6 × 1023
n = 6.82 mol

8.5 MOLAR MASS


• Molar mass is defined as the mass of a substance per one mole of that substance.
• The symbol for molar mass is Mr
• Unit: g/mol
• Molar mass is calculated by adding the relative atomic mass of each element in a compound.

❖ EXAMPLES
1. Find the molar mass of H2O
2. Calculate the molar mass of CaCO3
3. What is the molar mass of NaCl?

❖ SOLUTIONS
1. Mr (H2O) = (1 × 2) + 16
= 18 g/mol

2. Mr (CaCO3) = (1 × 40) + 12 + (16 × 3)


= 100 g/mol

3. Mr (NaCl) = 23 + 35.5
= 58.5 g/mol

Stoichiometry and Mole Concept 53


❖ MASS – MOLE CONVERSION
• When given the mole of a substance the mass of that substance can be found.
• When given the mass of a substance the mole of that substance can be found.
• The relationship between mass and mole of a substance is expressed by the formula below.
• Formula:

m
n= where
Mr n = mole of a substance
m = mass of a substance
Mr = molar mass

❖ EXAMPLES
1. How many moles are in 5 grams of O2?
2. Find the number of moles in 60 grams of H2SO4?
3. How much does 4.2 moles of Ca(NO3)2?
4. How many grams of potassium sulphate (K2SO4) are there in 25.3 moles?

❖ SOLUTIONS
mass given
1. n = Mole = ?
molar mass Mass = 5g
5
n= Mr (O2) = 32 g/mol
32
n = 0.16 mol

mass given
2. n = Mole = ?
molar mass Mass = 60g
60
n= Mr (H2SO4) = 98 g/mol
98
n = 0.61 mol

3. mass = n × Mr Mole = 4.2 mol


mass = 4.2 × 164 Mass = ?
mass = 688.8g Mr (Ca(NO3)2) = 164 g/mol

4. mass = n × Mr
mass = 25.3 × 174 Mole = 25.3
mass = 4402.2g Mass = ?
Mr (K2SO4) = 174 g/mol

54 Stoichiometry and Mole Concept


8.6 PERCENTAGE COMPOSITION OF A COMPOUND
• Percentage composition is the ratio of the amount of each element to the total amount of individual
elements in a compound multiplied by 100%.
• For example, the percentage composition of element A in compound ABC can be expressed as:
A
% of A = × 100%
ABC
• The formula for calculating the percentage composition of an element in a compound is as given:

molar mass of element


% of element = × 100%
molar mass of compound

1. EXAMPLES
1. What is the percentage composition of hydrogen in water?
2. What is the percentage composition of oxygen in CO2?
3. Find the percentage composition of sodium in Na3PO4

2. SOLUTIONS
Mr (H)
1. % (H) = × 100% % (H) = ?
Mr (H2 O) Mr (H) = 1 × 2 = 2 g/mol
2 Mr (H2O) = 18 g/mol
% (H) = × 100%
18
% (H) = 11.1%

Mr (O)
2. % (O) = × 100% % (O) = ?
Mr (CO2 ) Mr (O) = 16 × 2 = 32 g/mol
32 Mr (CO2) = 44 g/mol
% (O) = × 100%
44
% (O) = 72.7%

Mr (Na)
3. % (Na) = × 100% % (Na) = ?
Mr (Na3 PO4 )
Mr (Na) = 23 × 3 = 69 g/mol
69
% (Na) = × 100% Mr (Na3PO4) = 164 g/mol
164
% (Na) = 42.1%

Stoichiometry and Mole Concept 55


8.7 MOLECULAR AND EMPIRICAL FORMULA
• The molecular formula is a chemical formula that gives the total number of atoms of each element
in a compound.
• The empirical formula is a chemical formula that gives the smallest whole number ratio of elements
in a compound.
• For example, the molecular and empirical formula for butane (C4H10) and butene (C4H8) are;
• Butane:
• Molecular formula: C4H10
• Empirical formula: C2H5

• Butene:
• Molecular formula: C4H8
• Empirical formula: CH4

1. HOW TO FIND EMPIRICAL FORMULA FROM MASS DATA


1. Find the number of moles of each element.
2. Divide the moles by the smallest number.

❖ EXAMPLES
1. Find the empirical formula of 2.45g sample of which 1.78g is iron and 0.68g is oxygen.
2. The unknown compound is found to contain 13.5g calcium, 10.8g oxygen and 0.68g
hydrogen. Calculate the empirical formula of this compound.

❖ SOLUTIONS
mass given Mole = ?
1. n (Fe) =
molar mass Mass = 2.45g
2.45 Mr (Fe) = 56 g/mol
n (Fe) =
56
n (Fe) = 0.04 mol

mass given
n (O) = Mole = ?
molar mass
0.68 Mass = 0.68g
n (O) = Mr (O) = 16 g/mol
16
n (O) = 0.04 mol

mol of Fe mol of O
Ratio of Fe = Ratio of O =
smallest mol smallest mol
0.04 0.04
Ratio of Fe = Ratio of O =
0.04 0.04
Ratio of Fe = 1 Ratio of O = 1

Empirical formula = Fe1O1


= FeO

56 Stoichiometry and Mole Concept


mass given Mole = ?
2. n (Ca) =
molar mass Mass = 13.5g
13.5 Mr (Ca) = 40 g/mol
n (Ca) =
40
n (Ca) = 0.34 mol

mass given Mole = ?


n (O) =
molar mass Mass = 10.8g
10.8 Mr (O) = 16 g/mol
n (O) =
16
n (O) = 0.68 mol

mass given Mole = ?


n (H) =
molar mass Mass = 0.68g
0.68 Mr (H) = 1 g/mol
n (H) =
1
n (H) = 0.68 mol

mol of Ca mol of O mol of H


Ratio of Ca = Ratio of O = Ratio of H =
smallest mol smallest mol smallest mol
0.34 0.68 0.68
Ratio of Ca = Ratio of O = Ratio of H =
0.34 0.34 0.34
Ratio of Ca = 1 Ratio of O = 2 Ratio of H = 2

Empirical formula = Ca1O2H2


= Ca(OH)2

2. HOW TO FIND EMPIRICAL FORMULA FROM PERCENTAGE COMPOSITION


1. Treat percentage as mass and calculate the mole of each element.
2. Divide the moles by the smallest mole number.

❖ EXAMPLES
1. The unknown compound contains 40% sulphur and 60% oxygen. What is the empirical formula
of the compound?
2. Find the empirical formula of the compound containing 28.83% magnesium, 14.24% carbon
and 56.93% oxygen.

Stoichiometry and Mole Concept 57


❖ SOLUTIONS
% given Mole = ?
1. n (S) =
molar mass % (S) = 40%
40 Mr (S) = 32 g/mol
n (S) =
32
n (S) = 1.25 mol

% given Mole = ?
n (O) =
molar mass % (O) = 60%
60 Mr (O) = 16 g/mol
n (O) =
16
n (O) = 3.75 mol

mol of S mol of O
Ratio of S = Ratio of O =
smallest mol smallest mol
1.25 3.75
Ratio of S = Ratio of O =
1.25 1.25
Ratio of S = 1 Ratio of O = 3

Empirical formula = S1O3


= SO3

% given Mole = ?
2. n (Mg) =
molar mass % (Mg) = 28.83%
28.83 Mr (Mg) = 24 g/mol
n (Mg) =
24
n (Mg) = 1.2 mol

% given
n (C) =
molar mass Mole = ?
14.24 % (Mg) = 14.24%
n (C) = Mr (S) = 12 g/mol
12
n (C) = 1.19 mol

% given
n (O) =
molar mass Mole = ?
56.93 % (O) = 56.93%
n (O) = Mr (S) = 16 g/mol
16
n (O) = 3.56 mol

58 Stoichiometry and Mole Concept


mol of Mg mol of C mol of O
Ratio of Mg = Ratio of C = Ratio of O =
smallest mol smallest mol smallest mol
1.2 1.19 3.56
Ratio of Mg = Ratio of C = Ratio of O =
1.19 1.19 1.19
Ratio of Mg = 1 Ratio of C = 1 Ratio of O = 2.99
=3

Empirical formula = Mg1C1O3


= MgCO3

3. HOW TO FIND THE MOLECULAR FORMULA


1. Find the empirical formula.
2. Calculate the empirical mass.
3. Divide the molar mass by the empirical mass to get n.
4. Multiply n with the empirical formula.

❖ EXAMPLES
1. A sample is analysed and found to have an empirical formula CH2. It has a molar mass of
70 g/mol. Find its molecular formula.
2. A compound of empirical formula P2O5 has a molar mass of 284 g/mol. What is its molecular
formula?

❖ SOLUTIONS
1. Empirical formula = CH2
Empirical mass = 12 + (1 × 2)
= 14
molar mass
n=
empirical mass
70
n=
14
n=5

Molecular formula = empirical formula × n


= (CH2) × 5
= C5H10

2. Empirical formula = P2O5


Empirical mass = (31 × 2) + (16 × 5)
= 142
molar mass
n=
empirical mass
284
n=
242
n=2

Molecular formula = empirical formula × n


= (P2O4) × 2
= P4O8

Stoichiometry and Mole Concept 59


8.8 MOLAR VOLUME OF GASES
• 1 mol of any gas has 24 dm3 volume of gas at room temperature and pressure. This volume is called
molar volume of a gas.
• With known moles of a gas at room temperature and pressure, its volume can be calculated.
• With a known volume of a gas at room temperature and pressure, its mole can be calculated.
• The relationship between mole and volume of a gas at room temperature and pressure is expressed
below.
• Formula:

volume of gas Remember


Mole = 1 dm3 = 1 L
molar volume
1 L = 1000 mL
1 mL = 1 cm3
volume of gas
Mole =
24 dm3 Volume of gas should be in dm3 or L

❖ EXAMPLES
1. What is the number of moles of 3.6 dm3 of oxygen gas at room temperature and pressure?
2. Calculate the moles of nitrogen gas in 700ml at room temperature and pressure.
3. Calculate the volume of 10g of fluorine gas at room temperature and pressure.

❖ SOLUTIONS
volume of gas Mole = ?
1. Mole =
24 dm3 Volume (O2) = 3.6 dm3
3.6 Molar volume = 24 dm3
Mole =
24
Mole = 0.15 mol

volume of gas Mole = ?


2. Mole =
24 dm3 Volume (O2) = 700ml ÷ 1000
0.7 = 0.7 dm3
Mole = Molar volume = 24 dm3
24
Mole = 0.03 mol

mass given Mole = ?


3. Mole =
molar mass Mass given = 10g
10 Molar mass (F2) = 19 × 2
Mole = = 38 g/mol
38
Mole = 0.26 mol

Volume of F2 = mole × 24 dm3 Mole = ?


= 0.26 × 24 Volume (O2) = 3.6 dm3
= 6.24 dm3 Molar volume = 24 dm3

60 Stoichiometry and Mole Concept


8.9 CONCENTRATION OF SOLUTION
• Concentration is the amount of solute dissolved in a solvent or solution.
• Units: mol/dm3
• Concentration indicates how much solute particles are in a given solution.
• A concentrated solution contains a high amount of solute. Therefore the solution has more solute
particles.
• A dilute solution contains a low amount of solute. Therefore the solution has few solute particles.
• Molarity is the number of moles per litre or cubic decimetre in a solution. It is also called molar
concentration.
• Formula:

𝐧 Where Note
C=
𝐕 C = molar concentration (in mol/dm3, mol/L or M) 1 dm3 = 1 L
n = mole of solute 1 dm3 = 1000 cm3
V = volume of solution (in dm3 or L)

❖ EXAMPLES
1. Calculate the concentration of a solution containing 15g of NaOH dissolved in 0.3dm3 solution.
2. Calculate the molarity of a solution prepared by dissolving 32g of KMnO4 in 500cm3 of a
solution.
3. Calculate the number of moles in 100cm3 of 1.5 mol/dm3 of H2SO4.
4. How many grams of KNO3 will be required to prepare 800cm3 of 1.4 mol/dm3 of KNO3?

❖ SOLUTIONS
mass given
1. n (NaOH) =
molar mass
15
=
40
= 0.375 mol

n
C=
V
0.375
=
0.3
= 1.25 mol/dm3

mass given
2. n (NaOH) =
molar mass
32
=
158
= 0.2 mol

n
C=
V
0.2
=
0.5
= 0.4 mol/dm3

Stoichiometry and Mole Concept 61


3. n=C×V
= 1.5 × 0.1
= 0.15 mol

4. n=C×V
= 1.4 × 0.8
= 1.12 mol

mass of KNO3 = n × Mr
= 1.12 × 101
= 113.12g

8.10 DILUTION
• Dilution is the decreasing of the concentration of a solute by the addition of a solvent.
• Dilution reduces the concentration of a solute by increasing the concentration of a solvent.
• The moles of a solute in the initial solution and diluted solution are the same.
• The following formula calculates dilution.
• Formula:

M1V1 = M2V2 where


M1 = initial molarity (in mol/dm3)
M2 = final molarity (in mol/dm3)
V1 = initial volume (in dm3)
V1 = initial volume (in dm3)

❖ EXAMPLES
1. If 0.17dm3 of 2 mol/dm3 HCl is diluted to 0.5dm3, what is the concentration of the diluted
solution?
2. What volume of 3 mol/dm3 H2SO4 is required to make 0.7dm3 of 1 mol/dm3 solution?

❖ SOLUTIONS
1. M1V1 = M2V2
M1 V1
M2 =
V2
2 × 0.17
=
0.5
= 0.68 mol/dm3

2. M1V1 = M2V2
M2 V2
V1 =
M1
1 × 0.7
=
3
= 0.23 mol/dm3

62 Stoichiometry and Mole Concept


8.10 LIMITING REAGENT
• A limiting reagent is a reactant that runs out first.
• Limiting reagents determine how much products can be formed.
• When the limiting reagent runs off the chemical reaction stops.
• The reactant that is left after the completion of a reaction is called the excess reagent.

❖ HOW TO FIND THE LIMITING REAGENT


1. Balance the chemical equation.
2. Convert the given reactants to moles.
3. Find the ratio of reactants.
4. Cross multiply the ratio of the reactants. The reactant with the smallest mole ratio is the limiting
reactant. The reactant with a larger mole ratio is the excess reagent.
5. If asked to find the mass of the product formed, use the mole ratio of the limiting reagent to find
the moles of the product. Then convert moles of the product to mass.

❖ EXAMPLES
1. If 4g of hydrogen reacts with 4g of oxygen
2H2(g) + O2(g) → 2H2O(l)
A. What is the limiting reagent?
B. Calculate the amount of water produced.

2. When 5g of sodium reacts with 5g of oxygen, sodium oxide is formed


4Na(s) + O2(g) → 2Na2O(s)
A. What is the limiting reagent?
B. How many grams of sodium oxide is formed?

3. If 4.8 mole of calcium reacts with 2 moles of nitrogen, what is the limiting reagent?
3Ca(s) + N2(g) → Ca3N2(s)

❖ SOLUTIONS
mass given mass given
1. A. n(H2) = n(O2) =
molar mass molar mass
4 4
= =
2 32
= 2 moles = 0.125 moles

the mole ratio of H2 to O2 is 2:1, so 2 moles of H2 is used, 1 mole of O2 also used


we have 0.125 moles of O2, so oxygen is the limiting reagent

B. Mole ratio of O2 to H2O is 1:2


1 2
=
0.125 x moles of H2 O
x moles of H2 O = 0.25 mol

mass of H2O produced = mol × molar mass


= 0.25 × 18
= 4.5g

Stoichiometry and Mole Concept 63


mass given mass given
2. A. n(Na) = n(O2) =
molar mass molar mass
5 5
= =
23 32
= 0.217 mol = 0.156 mol

the mole ratio of Na to O2 is 4:1. If 0.217 mol of Na is used, moles of O2 needed is


4 1
=
0.217 x moles of O2
x moles of O2 = 0.05425 mol
we have 0.156 mol of O2, therefore sodium is a limiting reagent

B. Mole ratio of Na to Na2O is 4 to 2. Moles of Na2O produced by 0.217 mol of Na is


4 2
=
0.217 x moles of Na2 O
x moles of O2 = 0.1085 mol
mass of Na2O produced = mol × molar mass
= 0.1085 × 62
= 6.7g

3. The mole ratio of Ca to N2 is 3:1. If 4.8 mol of Ca is used, the mol of N2 required is
3 1
=
4.8 x moles of N2
x moles of N2 = 1.6 mol

we have 2 moles of nitrogen, therefore calcium is a limiting agent

8.10 PERCENTAGE YIELD


• Percentage yield is defined as the ratio of the actual yield to the theoretical yield.
• Usually, 100% of the expected products are not obtained.
• This may be due to various reasons such as not all reactants used up, reactants containing impurities
or some products lost in the experiment.
• Percentage yield shows the amount of products formed to the expected amount of products.
• The formula for calculating the percentage yield of a substance is as given:

𝐚𝐜𝐭𝐮𝐚𝐥 𝐲𝐢𝐞𝐥𝐝
Percentage yield = × 100%
𝐭𝐡𝐞𝐨𝐫𝐞𝐭𝐢𝐜𝐚𝐥 𝐲𝐢𝐞𝐥𝐝

❖ EXAMPLES
1. What is the percentage yield if 14g of CaO produced 24g of CaCO3 when heated?
CaCO3(s) → CaO(s) + CO2(g)
2. When 2.8g of nitrogen was reacted with 1g of hydrogen 5.1g of ammonia was produced
A. What is the limiting reagent?
B. What is the percentage yield?

64 Stoichiometry and Mole Concept


❖ SOLUTIONS
24
1. n(CaCO3) =
100
= 0.24 mol

mole ratio of CaCO3 to CaO is 1:1, 0.24 mol of CaCO3 produces 0.24 mol of CaO
mass of CaO produced = mol × molar mass
= 0.24 × 72
= 17.28g

actual yield
Percentage yield = × 100%
theoretical yield
14
= × 100%
17.28
= 81.0185
= 81%

mass given mass given


2. A. n(N2) = n(H2) =
molar mass molar mass
2.8 1
= =
14 2
= 0.2 mol = 0.5 mol

the mole ratio of N2 to H2 is 1:3


1 3
=
0.2 x moles of O2
x moles of H2 = 0.6 mol
we have 0.5 mol H2, so hydrogen is the limiting reagent

B. mole ratio of H2 to NH3 is 3:2, then the of NH3 produced is


3 2
=
0.5 x moles of NH3
x moles of NH3 = 0.333 mol

mass of NH3 produced = mol × molar mass


= 0.333 × 17
= 5.661
= 5.7g

Stoichiometry and Mole Concept 65


8.11 PERCENTAGE PURITY
• Percentage purity is defined as the mass of pure substance in an impure sample.
• Impurities affect the physical and chemical properties of a substance.
• With drugs, impurities may cause side effects.
• Percentage purity tells us how pure a substance is.
• The formula for calculating the percentage purity of a substance is as given:

𝐦𝐚𝐬𝐬 𝐨𝐟 𝐚𝐜𝐭𝐮𝐚𝐥 𝐬𝐮𝐛𝐬𝐭𝐚𝐧𝐜𝐞


Percentage purity = × 100%
𝐦𝐚𝐬𝐬 𝐨𝐟 𝐢𝐦𝐩𝐮𝐫𝐞 𝐬𝐮𝐛𝐬𝐭𝐚𝐧𝐜𝐞

❖ EXAMPLES
1. A 12g sample of copper ore contains 10.2g of pure copper. What is the percentage purity?
2. You have 13.9g sample of impure iron pyrite. The sample is heated in the air to produce iron
(III) oxide and sulphur dioxide.
4FeS2(s) + 11O2(g) → 2Fe2O3(s) + 8SO2(g)
If you obtain 8.02g of iron (III) oxide, what is the percentage purity of iron pyrite in the
sample?

❖ SOLUTIONS
mass of actual substance
1. Percentage purity = × 100%
mass of inpure substance
10.2
= × 100%
12
= 85%

2. Find the mass iron pyrite that reacted with oxygen to produce 8.02g of iron (III) oxide
8.02
n(Fe2O3) =
160
= 0.050125
= 0.05 mol

the mole ratio of Fe2O3 to FeS2 is 2:4, so to produce 0.05 mol of Fe2O3 moles of FeS2 that
reacted is
2 4
=
0.05 x moles of Fe2 O3
x moles of Fe2 O3 = 0.1 mol

mass of FeS2 that reacted with oxygen


mass = mol × molar mass
= 0.1 × 120
= 12g

mass of actual substance


Percentage purity = × 100%
mass of inpure substance
10.2
= × 100%
12
= 86.3%

66 Stoichiometry and Mole Concept


REVIEW QUESTIONS
1. Find the relative formula mass of calcium phosphate, Ca3(PO4)2
A. 135
B. 175
C. 230
D. 310
2. Which of the following is true about 1 mole of water and 1 mole of carbon dioxide at r.t.p? They both
A. contain the same number of molecules
B. contain the same number of oxygen atoms
C. have the same density
D. have the same mass
3. In athletics, banned drugs such as nandrolone have been taken illegally to improve performance.
Nandrolone has the molecular formula C18H26O2. What is the relative molecular mass of nandrolone?
A. 306
B. 274
C. 150
D. 46
4. The relative atomic mass of chlorine is 35.5. What is the mass of 2 moles of chlorine atoms?
A. 17.8g
B. 35.5g
C. 71.0g
D. 142g
5. 18g of water contain the same number of molecules as
A. 2g of hydrogen gas
B. 14g of nitrogen gas
C. 16g of oxygen gas
D. 18g of ammonia
6. Deduce the empirical formula of a compound formed by reacting 1.15g of sodium with 0.8g of sulphur
A. Na4S2
B. NaS
C. NaS2
D. Na2S
7. A hydrocarbon consists of 80% by mass of carbon atoms. What is the empirical formula?
A. CH2
B. CH3
C. C2H3
D. C2H5
8. If 32g of an element Y combine with 48g of oxygen, what will be the empirical formula of the compound
formed? (Relative atomic mass of Y = 32)
A. Y3O
B. YO3
C. Y2O3
D. Y3O3
9. A compound has the empirical formula of CH2O and a relative molecular mass of 60. What is the
molecular formula of this compound?
A. C2H4O
B. C3H4O2
C. C3H4O3
D. C2H4O2

Stoichiometry and Mole Concept 67


10. An oxide of hydrogen has a percentage composition by mass H = 5.9% and O = 94.1%. Its relative
molecular mass is 34. Calculate the molecular formula of the oxide
A. HO
B. H2O
C. H2O2
D. H2O3
11. One mole of oxygen (O2) and one mole of nitrogen (N2) at r.t.p ….
A. weigh the same
B. occupy 24dm3 volume
C. contain the same elements
D. have the same mass
12. What mass of dinitrogen oxide, N2O occupies a volume of 18dm3 measured at r.t.p?
A. 44g
B. 33g
C. 24g
D. 22g
13. What is the percentage yield when 6.4g of copper are heated in air and 7.6g of copper (II) oxide is
obtained?
A. 20%
B. 80%
C. 95%
D. 98%
14. What mass of methane (CH4) occupies the same volume, measured at r.t.p as 11g of carbon dioxide?
A. 4g
B. 16g
C. 176g
D. 264g
15. What is the concentration of iodine molecules (I2) in a solution containing 2.4g of iodine in 250cm3 of the
solution?
A. 0.01 mol/dm3
B. 0.02 mol/dm3
C. 0.04 mol/dm3
D. 0.08 mol/dm3
16. What volume of 0.5 mol/dm3 sodium carbonate (Na2CO3) solution contains 2.0g of the solute?
A. 37.7 cm3
B. 39.7 cm3
C. 337.7 cm3
D. 377.7 cm3
17. 25.0cm3 of 1.00 mol/dm3 potassium hydroxide just neutralises 20.0cm3 of solution of hydrochloric acid.
What is the concentration of the acid?
A. 1.25 mol/dm3
B. 1.00 mol/dm3
C. 0.80 mol/dm3
D. 0.022 mol/dm3
18. What is the measured amount of a product obtained from a chemical reaction?
A. Theoretical yield
B. Actual yield
C. Percentage yield
D. Percentage purity

68 Stoichiometry and Mole Concept


19. If the mole ratio of reactants and products in a chemical reaction are known
A. The speed of the chemical reaction can be calculated
B. The energy released from the reaction can be calculated
C. The mass of products formed from the chemical reaction can be calculated
D. All the above
20. In the reaction CH4(g) + 2O2(g) → CO2(g) + 2H2O(l), what is the mole ratio of methane to water?
A. 1:1
B. 1:2
C. 2:1
D. 2:2
21. 16g of methane (CH4) is burnt completely in oxygen to form carbon dioxide (CO2) and water (H2O). What
mass of water would be produced?
A. 18g
B. 36g
C. 54g
D. 72g
22. Which reactant controls the amount of products formed in a chemical reaction?
A. Limiting reagent
B. Excess reagent
C. Catalyst
D. All the above
23. In a chemical reaction, the limiting reagent
A. Must be in solution
B. Speeds up the reaction
C. Remains after completion of the reaction
D. Is used up first
24. An organic compound contains 40.00% C, 6.99% H and the rest is oxygen by mass. The mass of 0.02mol
of the compound is 3.60g. Calculate
A. Empirical formula of the compound.
B. Molecular formula of the compound.
25. An iron salt was analysed and found to have 20.2% Fe, 11.5% S, 23% O and 45.3 H2O of crystallisation.
The relative formula mass of the compound is 278
A. Work out the formula of the salt.
B. 6.95g of the hydrated salt were dissolved in distilled water to make 250cm3 of solution. Calculate the
concentration of the solution in mol/dm3.
26. 5.0g of pure calcium carbonate were completely decomposed by heat. 900cm3 of carbon dioxide were
evolved at room temperature and pressure
CaCO3(s) → CaO(s) + CO2(g)
A. Calculate the volume of carbon dioxide which was expected to be produced.
B. Calculate the percentage (%) yield of carbon dioxide gas.
27. In an experiment, 2.60g of zinc was heated with 1.71g of sulphur according to the reaction below
8Zn + S8 → 8ZnS
A. Which chemical is the limiting reagent?
B. Calculate the mass of zinc sulphide produced.

Stoichiometry and Mole Concept 69


28. Methanol reacts with carbon dioxide in the presence of a catalyst to produce ethanoic acid
CH3OH(l) + CO(g) → CH3COOH(l)
In the above experiment, 8.0g of methanol reacted completely with carbon monoxide in a reaction vessel
as shown below.

A. What is the theoretical yield of ethanoic acid?


B. If the actual yield is 12.0g. What is the percentage yield?

70 Stoichiometry and Mole Concept


SOLUTIONS
1. D
2. B
3. B
4. C
5. A
6. D
7. B
8. B
9. D
10. C
11. B
12. B
13. C
14. A
15. C
16. A
17. A
18. A
19. C
20. B
21. B
22. A
23. D

% given Mole = ?
24. n (C) =
molar mass % (C) = 40%
40 Mr (C) = 12 g/mol
n (Mg) =
12
n (Mg) = 3.33 mol

% given
n (C) = Mole = ?
molar mass % (H) = 6.99%
6.99 Mr (H) = 1 g/mol
n (C) =
1
n (C) = 1.19 mol

% given Mole = ?
n (O) =
molar mass % (O) = 53.01%
53.01 Mr (O) = 16 g/mol
n (O) =
16
n (O) = 3.31 mol

Stoichiometry and Mole Concept 71


mol of Mg mol of C mol of O
Ratio of C = Ratio of H = Ratio of O =
smallest mol smallest mol smallest mol
3.33 6.99 3.31
Ratio of C = Ratio of H = Ratio of O =
3.31 3.41 3.31
Ratio of C = 1 Ratio of H = 2 Ratio of O = 1

Empirical formula = C1H2O1


= CH2O

mass given
B. Molar mass =
mole
3.60
=
0.02
= 180g/mol

molar mass
n=
empirical mass
180
n=
30
n=6

Molecular formula = empirical formula × n


= (CH2O) × 6
= C6H12O6

% given Mole = ?
24. A. n (Fe) =
molar mass % (C) = 40%
20.2 Mr (C) = 12 g/mol
n (Fe) =
56
n (Fe) = 0.36 mol

% given
n (S) = Mole = ?
molar mass % (H) = 6.99%
11 Mr (H) = 1 g/mol
n (S) =
32
n (S) = 0.36 mol

% given Mole = ?
n (O) =
molar mass % (O) = 53.01%
23 Mr (O) = 16 g/mol
n (O) =
16
n (O) = 1.44 mol

72 Stoichiometry and Mole Concept


% given
n (H2O) = Mole = ?
molar mass % (H2O) = 53.01%
45.3 Mr (H2O) = 16 g/mol
n (H2O) =
18
n (H2O) = 2.52 mol

mol of Mg mol of C mol of O


Ratio of Fe = Ratio of S = Ratio of O =
smallest mol smallest mol smallest mol
0.36 0.36 1.44
Ratio of Fe = Ratio of S = Ratio of O =
0.36 0.36 0.36
Ratio of Fe = 1 Ratio of S = 1 Ratio of O = 4

mol of Mg
Ratio of H2O =
smallest mol
2.52
Ratio of H2O =
0.36
Ratio of H2O = 7

Empirical formula = Fe1S1O1⋅7 H2O


= FeSO⋅7 H2O

mass given
B. n (FeSO⋅7 H2O) = Mole = ?
molar mass Mass of FeSO⋅7 H2O = 6.95
6.95
n (FeSO⋅7 H2O) = Mr of FeSO⋅7 H2O = 278 g/mol
278
n (FeSO⋅7 H2O) = 0.025 mol

moles
concentration = concentration = ?
volume mole = 0.025 mol
0.025
= volume = 250 cm3 ÷ 1000
0.25 = 0.25 dm3
= 0.1 mol/dm3

Stoichiometry and Mole Concept 73


mass given
26. A. n (CaCO3) = mole = ?
molar mass mass of CaCO3 = 53.01%
5.0 Mr of CaCO3 = 100 g/mol
n (CaCO3) =
100
n (CaCO3) = 0.05 mol

Volume produced = mol × 24dm3


= 0.05 × 24
= 1.2dm3

actual yield
B. Percentage yield = × 100%
theoretical yield
0.9
= × 100%
1.2
= 75%

mass given Mole = ?


27. A. n (Zn) =
molar mass % (H) = 6.99%
2.60 Mr (H) = 1 g/mol
=
65
= 0.04 mol

mass given Mole = ?


n (S) =
molar mass % (O) = 53.01%
1.71 Mr (O) = 16 g/mol
=
32
= 0.05 mol

the mole ratio of Zn to S is 8:1. A mole of S needed to react with 0.04 mol of Zn is
8 1
=
0.04 x moles of S
8x = 0.04
x moles of S = 0.005 mol
we have 0.05 mol S, so sulphur is excess and zinc is the limiting reagent

B. ratio of Zn to ZnS is 8:8, therefore 0.04 mol of Zn produces 0.04 mol of ZnS
mass of ZnS produced = mol × molar mass
= 0.04 × 97
= 3.88g

74 Stoichiometry and Mole Concept


mass given
28. A. n (CH3OH) =
molar mass
8
=
32
= 0.25 mol

ratio of CH3OH to CH3COOH is 1:1, therefore 0.25 mol of CH3OH produces 0.25 mol of CH3COOH
mass of CH3COOH produced = mol × molar mass
= 0.25 × 60
= 15g

actual yield
B. Percentage yield = × 100%
theoretical yield
12
= × 100%
15
= 80%

Stoichiometry and Mole Concept 75


9.0 PERIODIC TABLE

Introduction
The periodic table is a tabular arrangement of chemical elements. The first periodic table was made by Dmitri
Mendeleev in 1869. Mendeleev’s periodic table contained 63 elements known at that time. He arranged these
elements in the order of atomic mass.

The modern periodic table contains 118 currently known elements. Of these, 94 elements occur naturally on
earth. Unlike Mendeleev’s periodic table, the modern periodic table has elements arranged in the order of the
atomic number.

Elements with similar chemical properties are placed in the same vertical columns called groups. The
horizontal rows of the periodic table are called periods. This unit describes the groups and periods of the
periodic table.

Specific outcomes
By the end of this unit, you will be able to:
❖ Differentiate Mendeleev’s periodic table and the modern periodic table
❖ Define group and period of the periodic table
❖ State the properties and uses of the following groups:
• Alkali metals
• Alkaline earth metals
• Halogens
• Noble gases
• Transition metals

76 Periodic Table
PERIODIC TABLE
9.1 HISTORY OF PERIODIC TABLE
❖ MENDELEEV’S PERIODIC TABLE
• The first periodic table was made by Dmitri Mendeleev in 1869.
• He arranged the periodic table in the order of atomic mass.
• He grouped elements with similar properties in vertical columns.
• From left to right, elements were placed in order of increase in atomic mass.
• The table contained 63 elements known at that time.

❖ MODERN TIME TABLE


• Like Mendeleev’s periodic table, the modern periodic table has elements with similar properties
arranged in vertical columns called groups.
• Like Mendeleev’s periodic table, the modern periodic table has elements placed in the order of
increase in atomic size in horizontal rows called periods.
• Unlike Mendeleev’s periodic table which arranged elements in order of atomic mass, the
modern periodic table arranges elements in order of atomic number.

9.2 FEATURES OF THE PERIODIC TABLE

❖ GROUPS
• Groups are vertical columns of the periodic table.
• The periodic table has eight main groups; I, II, III, IV, V, VI, VII, O.
• Elements in the same group have the same number of electrons in the outer shell.
• Elements in the same group have similar chemical properties.
• Group I elements are called alkali metals.
• Group II elements are called alkaline earth metals.
• Elements between Group II and Group III are called transition metals.
• Group VII elements are called halogens.
• Group O, also known as Group VIII, elements are called noble gases.
• From top to bottom the number of electron shells and radius of elements increase.

Periodic Table 77
❖ PERIODS
• Periods are horizontal rows of the periodic table.
• The period number corresponds to the number of shells an element has. For example, hydrogen and
helium are in period I therefore they have one shell. Carbon and oxygen are in period II therefore
they have two shells. Sodium and chlorine are in period III therefore they have three shells etc.
• Elements are arranged in order of increasing atomic number from left to right.
• Metals are found on the left side while non-metals are on the right side.
• Metallic characteristics of elements decrease from left to right.
• General characteristics of metals: They are shiny solids except for mercury which is liquid. They
are ductile (able to be drawn into a wire). They are malleable (able to be shaped without breaking).
They are good conductors of electricity.
• General characteristics of non-metals are; they are gases except for bromine which is liquid, are not
ductile, are not malleable and are bad conductors of electricity.
• Elements in the middle are called metalloids. They have properties of metals and non-metals.

9.3 GROUP PROPERTIES


1. GROUP I: ALKALI METALS
• In order of increasing atomic number alkali metals are; lithium (Li), sodium (Na), potassium
(K), rubidium (Rb), caesium (Cs) and francium (Fr).
• They have one electron in the outer shell.
• They are the most reactive metals on the periodic table. Reactivity increases down the group
with francium (Fr) being the most reactive metal.
• They readily react with non-metals to form ionic compounds.
• They readily react with water.
• They readily with air. Therefore, they are kept in oil.
• They are soft and shiny solids. They can be cut with a plastic knife.
• They have a low melting point, boiling point and density.
• Melting and boiling point decreases down the group.
• Density increases down the group.

❖ USES OF ALKALI METALS


• Lithium: used in lithium-ion batteries.
• Sodium: sodium chloride (table salt) is used in food flavouring and food preservation.
Sodium is used in sodium vapour lamps for streetlight bulbs; it produces efficient light.
Sodium salts of fatty acids are used as soap.
• Potassium: used in fertiliser.
• Rubidium: used in fireworks.
• Caesium: used in photoelectric cells such as solar cells to convert sunlight into electricity.
• Francium: has no commercial use but for research.

2. GROUP II: ALKALINE EARTH METALS


• In order of increasing atomic number alkaline earth metals are; beryllium (Be), magnesium
(Mg), calcium (Ca), strontium (Sr), barium (Ba) and radium (Ra).
• They have 2 electrons in the outer shell.
• They are very reactive but less reactive than alkali metals. Reactivity increases down the group.
• They are reactive at standard temperature and pressure.
• They are cold with water except for beryllium, which is reactive with steam.
• They have a low melting point, boiling point and density.
• They have a low melting point, boiling point and density.
• Melting and boiling point decreases down the group.
• They are shiny, silvery white soft metals.

78 Periodic Table
❖ USES OF ALKALINE EARTH METALS
• Beryllium: it is alloyed with other metals such as copper and used to make springs.
• Magnesium: can be alloyed with other metals. Magnesium aluminium alloy is light in mass and is
used in aircrafts construction. In plants, it is an important component of chlorophyll.
• Calcium: used in the extraction of other metals from their metal oxides. In animals, it is an essential
component of bones.
• Barium: used in X-ray imaging, fireworks.
• Radium is used in the treatment of cancer.

3. GROUP VII: HALOGENS


• In order of increasing atomic number halogens are; fluorine (F), chlorine (Cl), bromine (Br), iodine
(I) and astatine (At).
• They have 7 electrons in the outer shell.
• They are the most reactive non-metals on the periodic table. Reactivity decreases down the group
with fluorine (F) being the most reactive non-metal.
• They exist as diatomic molecules (X2); Cl2, F2, Br2, I2.
• They are coloured non-metals; fluorine is yellow gas, chlorine is pale green gas, bromine is brown
liquid, iodine is purple black solid.
• They have low melting, boiling point and density.
• Melting, boiling point and density increases down the group.
• They are poisonous.
• They form ionic compounds with metals such as sodium chloride (NaCl) and covalent compounds
with non-metals such as methane (CH4).
• When a halogen reacts with metal and form an ionic compound, the halogen name changes from -
ine to -ide while the metal’s name does not change. For example, when sodium and chlorine react
the compound formed is called sodium chloride. When magnesium and bromine react, the
compound formed is called magnesium bromide.
• A more reactive halogen displaces a less reactive halogen. For example, fluorine is more reactive
than chlorine. Therefore when sodium chloride is reacted with fluorine, sodium fluoride is formed.
But when sodium fluoride is reacted with chlorine the reaction does not occur.
2NaCl(s) + F2(g) → 2NaF(s) + Cl2(g)
NaF(s) + Cl2(s) → no reaction

❖ USES OF HALOGENS
• Sodium chloride (table salt): used for food flavouring and food preservation.
• Chlorine: used in disinfectants to kill bacteria, pesticide, weed killer
• Bromine: used in insecticide, photographic film
• Fluorine: used in toothpaste to prevent tooth decay
• Iodine: used in disinfectants to kill bacteria, in photographic films, in humans it is used in the
formation of thyroid hormones

❖ HARMFUL EFFECTS OF HALOGENS


• Inhaled halogens irritate the respiratory tract and can cause lung damage.
• Chlorine can cause blindness, lung damage and even death. It was used in WWI.
• Liquid bromine is very corrosive.
• Chlorine in the atmosphere can destroy the ozone layer.

Periodic Table 79
4. GROUP O: NOBLE GASES
• In order of increasing atomic number, noble gases are; helium (He), neon (Ne), argon (Ar), krypton
(Kr), xenon (Xe) and radon (Rn).
• They have full electrons in the outer shell. Therefore, they are very stable elements.
• They are non-reactive.
• They exist as individual atoms.
• They are colourless gases.
• They have low density. Density increases down the group.

❖ USES OF NOBLE GASES


• Argon is used in electric light bulbs.
• Helium is used in balloons and airships.
• Neon is used in advertising lights.

5. TRANSITION METALS
• Transition metals are elements between Group II and Group III.
• They have variable valences, such as iron (Fe) can be Fe2+ or Fe3+.
• They form a coloured compound, such as copper sulphate (CuSO4) is blue. Therefore, they are used
in painting substances.
• Some elements conduct electricity.
• They have high melting and boiling point.
• They have high density.

❖ USES OF TRANSITION METALS


• Iron is used for building materials.
• Copper is used to make electric wires and water pipes.
• Titanium is used in paints.
• Silver is used in photographic films and jewellery.
• Platinum is used as a catalyst.

80 Periodic Table
REVIEW QUESTIONS
1. In Mendeleev’s periodic table elements were arranged in the order of
A. Reactivity
B. Atomic number
C. Density
D. Atomic mass
2. Unlike Mendeleev’s periodic table, the modern periodic table has elements arranged in the order of
A. Reactivity
B. Atonic number
C. Density
D. Atomic mass
3. Which of the following is not true about groups of the periodic table?
A. Elements of the same group have the same valence electrons
B. Elements of the same group have the same density
C. Elements of the same group have similar chemical properties
D. Elements of the same group show a trend in properties
4. Group I elements are called
A. Halogens
B. Transition metals
C. Alkali metals
D. Alkaline earth metals
5. Group II elements are called
A. Halogens
B. Noble gases
C. Alkali metals
D. Alkaline earth metals
6. Group VII elements are called
A. Halogens
B. Noble gases
C. Alkali metals
D. Alkaline earth metals
7. Group O elements are called
A. Halogens
B. Noble gases
C. Alkali metals
D. Transition metals
8. Transition metals are between
A. Group I and II
B. Group II and III
C. Group III and IV
D. Group VI and VII
9. Which of the following is not true about periods of the periodic table?
A. From left to right the metallic characteristics of elements decreases
B. The period number of an element is equal to the number of shells of that element
C. Periods are vertical columns of the periodic table
D. Periods are horizontal rows of the periodic table
10. Which of the following is a property of metals?
A. Ductile
B. Shiny
C. Good conductor
D. All the above

Periodic Table 81
11. The most reactive metal on the periodic table is
A. Sodium
B. Copper
C. Francium
D. Calcium
12. Which of the following is not a property of alkali metals?
A. They have high melting and boiling point
B. They vigorously react with air
C. They vigorously react with water
D. They are soft shiny solids
13. Which of the following elements is an alkali metal?
A. Calcium
B. Sodium
C. Copper
D. Aluminium
14. Which of the following elements is an alkaline earth metal?
A. Calcium
B. Sodium
C. Copper
D. Aluminium
15. Which of the following is not a property of alkaline earth metals?
A. They are shiny, silvery white metals
B. They react with water
C. Melting and boiling point increases down the group
D. Melting and boiling point decreases down the group
16. Which of the following elements is a halogen?
A. Helium
B. Oxygen
C. Chlorine
D. Carbon
17. Which of the following is not a property of halogen?
A. Melting and boiling point decreases down the group
B. They exist as diatomic molecules
C. They are coloured elements
D. They have 7 electrons in the outer shell
18. What is the most reactive non-metal element on the periodic table?
A. Oxygen
B. Fluorine
C. Nitrogen
D. Neon
19. Which of the following elements is a noble gas?
A. Oxygen
B. Fluorine
C. Nitrogen
D. Neon
20. Which of the following property is not a property of noble gases?
A. They are very reactive
B. They are colourless gases
C. They exist as individual atoms
D. They have full electrons in the outer shell

82 Periodic Table
21. Which of the following is not true about transition metals?
A. They have variable valences
B. They have high density
C. They have high melting and boiling point
D. They are poor conductors of electricity
22. What metal is liquid at room temperature
A. Magnesium
B. Mercury
C. Bromine
D. Calcium
23. What non-metal is liquid at room temperature
A. Chlorine
B. Mercury
C. Bromine
D. Oxygen
24. The diagram below shows a representation of a periodic table. Note that letters are not the actual symbol
of the element

A. To which group and period does the following belong?


i. Element A
ii. Element H
B. Describe the trend in reactivity in the group in which
i. A and E are placed
ii. C and G are placed
C. State a general name given to a group of elements to which the following belong
i. A, E and I
ii. B, F and J
iii. C and G
iv. D and H
D. State the letter representing an element that has the same electronic configuration as the stable ion of
i. C
ii. J

Periodic Table 83
25. The diagram below shows a representation of a periodic table. Note that letters are not the actual symbol
of the element

Using letters in the periodic table above, give a letter representing the element which
A. has the lowest density
B. has the largest relative atomic mass
C. liquid at room temperature and pressure
D. is used to make electricity cables
E. forms a divalent cation

84 Periodic Table
SOLUTIONS
1. D
2. B
3. B
4. C
5. D
6. A
7. B
8. B
9. C
10. D
11. C
12. A
13. B
14. A
15. C
16. C
17. A
18. B
19. D
20. A
21. D
22. B
23. C

24. A. i. Element A: Group 1, Period 2


ii. Element H: Group O, Period 3
B. i. In this group reactivity of elements increases from top to bottom.
ii. In this group reactivity of elements decreases from top to bottom.
C. i. Alkali metals
ii. Alkaline earth metals
iii. Halogens
iv. Noble gases
D. i. D element C has 7 electrons in the outer shell. When C gains one electron to have 8
Therefore, the ion has the electronic configuration of element D.

ii. H element H has 2 electrons in the outer shell. Ion H2+ has lost 2 electrons and therefore
has the electronic configuration of element H.

25. A. J
B. S
C. R mercury and bromine are the only liquid elements at room temperature and pressure.
Element R is bromine
D. Q
E. S divalent cation is a cation with the valence of 2+. Therefore, the element that forms a
divalent cation has to be in group 2 of the periodic table

Periodic Table 85
10 ACIDS, BASES AND ALKALIS

Introduction
We encounter several acids and bases in our everyday lives. Think of the vinegar (acetic acid) we use to
flavour and preserve our food, lemon juice (citric acid) and carbonated drinks (carbonic acid). Ammonia, one
of the commonest bases, is used to make fertiliser.

What are acids? What are bases? What are alkalis? How do acids and bases react with each other and other
substances? How do you test for the presence of acids or bases? This unit covers these and many more
questions.

Specific outcomes
By the end of this unit, you will be able to:
❖ Acids:
• Define an acid
• State the physical properties of acids
• State the chemical properties of acids
• State the common examples of acids and their uses
• Describe what is meant by acid strength and acid concentration
❖ Bases:
• Define a base
• State the physical properties of bases
• State the chemical properties of bases
• State the common examples of bases and their uses
• Describe what is meant by base strength and base concentration
❖ Describe the pH scale
❖ Describe pH indicators
❖ Differentiate acid oxides, basic oxides, amphoteric oxides and neutral oxides

86 Acids, Bases and Alkalis


ACIDS, BASES AND ALKALIS
10.1 ACIDS
• When a substance dissolves in water it forms an aqueous solution.
• Some substances after dissolving in water release hydrogen ions (H+).
• These substances that release hydrogen ions in water are called acids.
• Hydrogen ion (H+) is also called proton because it only contains a proton
• Acids are proton donors. They donator protons to the solution and increase hydrogen ion
concentration in the solution.
• For example, when hydrochloric acid (HCl) dissolves in water it dissociates into hydrogen ions (H+)
and chloride ions (Cl−).
HCl(aq) ⇋ H+(aq) + Cl−(aq)
• Hydrogen ions are responsible for acid properties.

❖ PHYSICAL PROPERTIES OF ACIDS


1. They have a sour taste (for example, lemon juice, vinegar).
2. They turn blue litmus paper red.
3. Strong acids are corrosive.
4. They are soluble in water.
5. Acid solutions have a pH value below 7.
6. They conduct electricity.

❖ CHEMICAL PROPERTIES OF ACIDS


1. Acids react with metals to form salt and hydrogen gas. For example, sodium reacts with dilute
hydrochloric acid to produce sodium chloride and hydrogen gas.
2Na(s) + 2HCl(aq) → 2NaCl(aq) + H2(g)
2. Acids react with carbonates to form salt, water and carbon dioxide. For example, calcium
carbonate reacts with sulphuric acid to produce calcium sulphate (a salt), water and carbon
dioxide.
CaCO3(s) + H2SO4(aq) → CaSO4(aq) + H2O(l) + CO2(g)
3. Acids react with bases to form salt and water. The reaction between an acid and a base is known
as neutralisation reaction. For example, dilute hydrochloric acid reacts with sodium hydroxide
(base) to produce sodium chloride (a salt) and water. Likewise, dilute hydrochloric acid reacts
with magnesium oxide (a base) to produce magnesium chloride (a salt) and water.
HCl(aq) + NaOH(aq) → NaCl(aq) + H2O(l)
2HCl(aq) + MgO(s) → MgCl2(aq) + H2O(l)

❖ COMMONLY USED ACIDS


1. Hydrochloric acid (HCl)
2. Sulphuric acid (H2SO4)
3. Nitric acid (HNO3)
4. Ethanoic acid (also called acetic acid, CH3COOH)

Acids, Bases and Alkalis 87


❖ ACID STRENGTH AND CONCENTRATION
❖ ACID STRENGTH
• Acid strength is the measure of the degree of ionisation of acids in a solution.
• Ionisation is the process by which neutral molecules split into ions in a solution. Then, the
ionisation of acid is the process by which acids split into ions in water.
• Therefore, acid strength corresponds to the extent to which an acid ionises and release H+ to the
solution.
• Strong acids are acids that completely ionise in water. Examples of strong acids include
hydrochloric acid (HCl), sulphuric acid (H2SO4) and nitric acid (HNO2). These acids
completely ionise in water.
HCl(aq) → H+(aq) + Cl−(aq)
H2SO4(aq) → 2H+(aq) + SO42−(aq)
HNO2(aq) → H+(aq) + NO2−(aq)
• Weak acids are acids that partially ionise in water. Examples of weak acids include ethanoic
acid (CH3COOH), phosphoric acid (H3PO4) and carbonic acid (H2CO3). These acids partially
ionise in water.
CH3COOH(aq) ⇋ CH3COO−(aq) + H+(aq)
H3PO4(aq) ⇋ H2PO4−(aq) + H+(aq)
H2CO3(aq) ⇋ HCO3−(aq) + H+(aq)

❖ ACID CONCENTRATION
• Acid concentration is associated with the amount of H+ dissolved in a solvent.
• A concentrated acid solution is a solution with a large amount of H+.
• A dilute acid solution is a solution with a small amount of H+.

❖ USES OF ACIDS
1. Hydrochloric acid is used to remove rust from iron sheets, clean swimming pools and the human
body uses this acid in digestion.
2. Sulphuric acid is used in car batteries. It is also used to make fertiliser, detergents, paints and
plastics.
3. Nitric acid is used to make fertiliser.
4. Ethanoic acid is used in the preservation of food.

10.2 BASES AND ALKALIS


• A base is a substance that accepts hydrogen ions from acid in an aqueous solution.
• In short, a base is a proton acceptor.
• Most bases are metal oxides such as copper (II) oxide (CuO), metal hydroxides such sodium
hydroxide (NaOH) or metal carbonates such as calcium carbonate (CaCO3).
• Bases can be soluble or insoluble in water. For example, sodium hydroxide is soluble in water while
copper (II) oxide is insoluble in water.
• An alkali is a base that is soluble in water. For examples of alkalis include sodium hydroxide
(NaOH), potassium hydroxide (KOH), calcium hydroxide (Ca(OH)2) and ammonia (NH3).
• When alkalis ionise in water they produce hydroxide ions (OH−).
NaOH(aq) → Na+(aq) + OH−(aq)
KOH(aq) → K+(aq) + OH−(aq)
• Hydroxide ions are responsible for the base properties.

88 Acids, Bases and Alkalis


❖ PHYSICAL PROPERTIES OF BASES
1. They have a bitter taste.
2. They have a slippery touch.
3. They turn red litmus paper blue.
4. They turn colourless phenolphthalein to pink.
5. Strong bases are corrosive.
6. They have a pH value above 7.
7. They conduct electricity.

❖ CHEMICAL PROPERTIES OF BASES


1. Bases react with acids to form salt and water. The reaction between a base and an acid is called
neutralisation. The base can be an alkali or a metal oxide. For example, sodium hydroxide (an
alkali) reacts with dilute hydrochloric acid to produce sodium chloride (a salt) and water. Likewise,
magnesium oxide (a base) reacts with dilute hydrochloric acid to produce magnesium chloride (a
salt) and water.
NaOH(aq) + HCl(aq) → NaCl(aq) + H2O(l)
MgO(s) + 2HCl(aq) → MgCl2(aq) + H2O(l)
2. Bases react with metals to form salt and hydrogen gas. Most metals do not react with bases.
However, some metals such as zinc, aluminium, and lead react with strong alkalis to produce salt
and hydrogen gas. For example, zinc reacts with sodium hydroxide and forms sodium zincate and
hydrogen gas. Aluminium metal reacts with sodium hydroxide and forms sodium aluminate and
hydrogen gas.
2NaOH(aq) + Zn(s) → Na2ZnO2(aq) + H2(g)
2NaOH(aq) + 2Al(s) + H2O(l) → 2NaAlO2(aq) + 2H2(g)
3. Bases react with non-metal oxides to form salt and water. For example, a reaction between calcium
carbonate and carbon dioxide (a non-metal oxide) produces calcium carbonate (a salt) and water. A
react between calcium carbonate and sulphur dioxide (a non-metal oxide) produces calcium
sulphate (a salt) and water.
Ca(OH)2(aq) + CO2(g) → CaCO3(aq) + H2O(l)
Ca(OH)2(aq) + SO2(g) → CaSO3(aq) + H2O(l)

❖ COMMONLY USED BASES


1. Sodium hydroxide (NaOH)
2. Potassium hydroxide (KOH)
3. Magnesium hydroxide (Mg(OH)2)
4. Calcium hydroxide (Ca(OH)2)
5. Ammonium hydroxide (NH4OH)

❖ BASE STRENGTH AND CONCENTRATION


❖ BASE STRENGTH
• Like acid strength, base strength corresponds to the extent to which a base ionises in a solvent.
• Strong bases are bases that completely ionise in water. Examples of strong bases include
sodium hydroxide (NaOH), potassium hydroxide (KOH) and calcium hydroxide (Ca(OH)2).
These bases completely ionise in water.
NaOH(aq) → Na+(aq) + OH−(aq)
KOH(aq) → K+(aq) + OH−(aq)+
Ca(OH)2(aq) → Ca2+(aq) + 2OH−(aq)
• Weak bases are bases that partially ionise in water. The commonest weak base is ammonia
(NH3).
NH3(g) + H2O(l) ⇋ NH4+(aq) + OH−(aq)

Acids, Bases and Alkalis 89


❖ BASE CONCENTRATION
• Base concentration is associated with the amount of OH− dissolved in a solvent.
• A concentrated base solution is a solution with a large amount of OH−.
• A dilute base solution is a solution with a small amount of OH−.

❖ USES OF BASES
1. Sodium hydroxide is used to make soap, papers and synthetic fibres.
2. Calcium hydroxide is used to neutralise the acidity of the soil. It is also used to make cement and
bleaching powders.
3. Ammonium hydroxide is used to make fertiliser and plastics.
4. Magnesium hydroxide is used in antacid treatment such as heartburn.

10.3 THE pH SCALE

• The pH scale is the measure of the acidity or basicity of a solution.


• The scale ranges from 0 to 14.
• It is based on the relative concentration of H+ and OH− in solution.
• A solution with a high H+ concentration is considered an acidic solution.
• A solution with a high OH− concentration is considered a basic solution (or alkaline solution).
• Acidic solutions have pH values below 7. Therefore, acid with a higher H+ concentration has a
lower pH value than acid with a lower H+ concentration.
• Basic solutions have pH values above 7. Therefore, a base with a higher H+ concentration has a
higher pH value than a base with a lower H+ concentration.
• pH 7 is neutral. Pure water has a pH value of 7.

❖ pH INDICATORS
• A pH indicator is a substance that changes colour over a range of pH values when added to the
solution.
• The commonly used pH indicators are litmus and universal indicators.

❖ LITMUS
• Litmus is used to find out whether the solution is acidic or basic.
• Litmus paper comes in two colours, namely blue litmus paper and red litmus paper.
• A blue litmus paper turns red in an acidic solution. However, in a basic and neutral solution, it
stays blue.
• A red litmus paper turns blue in basic solution. However, in an acidic and neutral solution, it
stays red.

90 Acids, Bases and Alkalis


❖ UNIVERSAL INDICATOR
• The universal indicator is a mixture of pH indicators that show different colours at different pH
values.
• Unlike the litmus that shows a solution as either acidic or basic, the universal indicator also shows
the strength of the acidic or basic solution.
• The table below gives the range of colours and the pH values at which they are observed.
pH 0 1 2 3 4 5 6 7 8 9 10 11 12 13 14
Colour Red Orange Yellow Green Blue Violet
Strength Strong acid Strong base

Neutral

❖ OTHER pH INDICATORS
1. Phenolphthalein: The indicator is colourless. In an acidic solution, it stays colourless while in a
basic solution it turns pink.
2. Methyl orange: The indicator is orange in colour. In an acidic solution, it turns red. In the basic
solution, it turns yellow.

10.4 TYPES OF OXIDES


• Oxides are compounds that contain at least one oxygen atom and another element.
• Oxides are classified as acidic oxides, basic oxides, amphoteric and neutral oxides.

1. ACIDIC OXIDES
• Acidic oxides are oxides of non-metals.
• Examples of acidic oxides include; carbon dioxide (CO2), sulphur dioxide (SO2) and nitrogen
dioxide (NO2).
• Acidic oxides react with water to form an acidic solution. For example, carbon dioxide reacts
with water to form carbonic acid.
CO2(g) + H2O(l) → H2CO3(aq)
• Acidic oxides react with bases to form salt and water. For example, carbon dioxide with react
calcium hydroxide and forms calcium carbonate and water.
CO2(g) + Ca(OH)2(aq) → CaCO3(s) + H2O(l)

2. BASIC OXIDES
• Basic oxides are oxides of metals.
• Examples of basic oxides include; magnesium oxide (MgO), calcium oxide (CaO), sodium
oxide (Na2O) and copper (II) oxide (CuO).
• Soluble basic oxides dissolve in water and form an alkaline solution.
Na2O(s) + H2O(l) → 2NaOH(aq)
CaO(s) + H2O(l) → Ca(OH)2(aq)
MgO(s) + H2O(l) → Mg(OH)2(aq)
• Basic oxides react with acid to form salt and water. For example, magnesium oxide reacts with
hydrochloric acid and form magnesium chloride and water.
MgO(s) + 2HCl(s) → MgCl2(aq) + H2O(l)

Acids, Bases and Alkalis 91


3. AMPHOTERIC OXIDES
• Amphoteric oxides are metal oxides that react with both acids and bases
• Examples of amphoteric oxides include; aluminium oxide (Al2O3), zinc oxide (ZnO) and lead (II)
oxide (PbO).
• The reaction between amphoteric oxides and acids or bases produce salt and water.
• For example, aluminium oxide reacts with hydrochloric acid and form aluminium chloride and
water. Likewise, aluminium oxide reacts with concentrated sodium hydroxide and forms sodium
aluminate and water.
Al2O3(s) + 6HCl(aq) → 2AlCl3(aq) + 3H2O(l)
Al2O3(s) + 2NaOH(aq) → 2NaAlO2(aq) + H2O(l)

4. NEUTRAL OXIDES
• Neutral oxides are oxides that neither show acidic or basic properties.
• They do not react with either acids or bases.
• Examples of neutral oxides include; water (H2O), carbon monoxide (CO) and nitric oxide (NO).

92 Acids, Bases and Alkalis


REVIEW QUESTIONS
1. A substance that donates protons to the solution is called?
A. Salt
B. Base
C. Alkalis
D. Acid
2. The following are responsible for acid properties
A. Oxide ions
B. Hydroxide ions
C. Hydrogen ions
D. Metal ions
3. Which of the following is a physical property of acids?
A. Aqueous solutions of acids conduct an electric current
B. They have a pH value above 7
C. They have a bitter taste
D. They turn red litmus paper blue
4. Which of the following is not a chemical property of acids?
A. Acids react with bases to form salt and water
B. Acids react with water to form hydrogen gas and salt
C. Acids react with carbonates to form salt, water and carbon dioxide
D. Acids react with metals to form salt and hydrogen gas
5. Which of the following is a strong acid?
A. H3PO4
B. H2CO3
C. H2SO4
D. CH3COOH
6. Which statement is true concerning acid concentration?
A. Concentrated acid solution has a low amount of H+ ion
B. Concentrated acid solution has a low amount of OH− ion
C. Diluted acid solution has a low amount of H+ ion
D. Diluted acid solution has a low amount of OH+ ion
7. A substance that accepts protons from the solution is known as
A. Acid
B. Salt
C. Ion
D. Base
8. A soluble base is called
A. Salt
B. Alkali
C. Cation
D. Anion
9. Which of the following is not a base?
A. HBr
B. CaCO3
C. CuO
D. KOH
10. Which of the following is not a physical property of bases?
A. They turn red litmus paper blue
B. They conduct electric current
C. They are corrosive
D. They have a pH value below 7

Acids, Bases and Alkalis 93


11. Which of the following is not a chemical property of bases?
A. Bases react with non-metal oxides to form salt and water
B. Bases react with carbonates to form salt, water and carbon dioxide
C. Bases react with acids to form salt and water
D. Bases react with metals to form salt and hydrogen gas
12. The following are responsible for base properties
A. Oxide ions
B. Hydroxide ions
C. Hydrogen ions
D. Metal ions
13. Which of the following is a weak acid?
A. KOH
B. NaOH
C. Ca(OH)2
D. NH3
14. Which statement is true concerning base concentration?
A. Concentrated base solution has a high amount of H+ ion
B. Concentrated base solution has a high amount of OH− ion
C. Diluted base solution has a low amount of H+ ion
D. Diluted base solution has a low amount of OH+ ion
15. Concerning the pH value
A. Neutral solutions have a pH value of 7
B. Basic solutions have a pH value above 7
C. Acidic solutions have a pH value below 7
D. All the above
16. Basic oxides are
A. Metal oxides
B. Non-metal oxides
C. React with both acids and bases
D. Does not react with acids and bases
17. Acids oxides are
A. Metal oxides
B. Non-metal oxides
C. React with both acids and bases
D. Does not react with acids and bases
18. Neutral oxides are
A. Metal oxides
B. Non-metal oxides
C. React with both acids and bases
D. Does not react with acids and bases
19. Amphoteric oxides are
A. Metal oxides
B. Non-metal oxides
C. React with both acids and bases
D. Does not react with acids and bases

94 Acids, Bases and Alkalis


20. Which of the following compounds can react with both aqueous solutions of sodium hydroxide and
hydrochloric acid?
A. CaO
B. FeO
C. MgO
D. ZnO
21. The pH of an aqueous solution of ethanoic acid is found be to be 4. The pH of this solution can be
increased by adding
A. Aqueous sodium hydroxide
B. Copper turnings
C. Copper (II) chloride crystals
D. Sodium chloride crystals
22. An acid can be described as a substance which
A. Liberate both OH− and H+ ions when dissolved in water
B. Does not liberate OH− and H+ ions when dissolved in water
C. Dissolves in water to liberate OH− ions as the negatively charged ions
D. Dissolves in water to liberate H+ ions as the only positively charged ions
23. An aqueous solution of D has a pH value of less than 7. Which statement of D is correct? It
A. Turns red litmus paper blue
B. Reacts with iron (II) chloride to form a green precipitate
C. Dissolves in water to form hydroxide ions as the only negative ions
D. Neutralises an aqueous solution of sodium hydroxide
24. A man suffering from an excess of acid in the stomach has no indigestion tablets. Which substance could
he take to lower his acidity?
A. Aspirin (pH 6)
B. Bicarbonate of soda (pH 8)
C. Lemon juice (pH 5)
D. Salt water (pH 7)
25. What term is used to describe a reaction between an acid and an alkali? Write an ionic equation for this
reaction.
26. Acids can be classified as ‘weak’ or ‘strong’. Explain what is meant by the term ‘week’ and ‘strong’ when
referred to acids. Give one example of each type of acid.
27. A. A solid substance S was suspected to be acidic. Describe what test you would carry out in the
laboratory to determine whether S was acidic or not.
B. When an acid reacts with an alkali, a neutralisation reaction occurs and heat is released to the
surrounding
i. Write a balanced chemical equation for the reaction of an acid and an alkali of your choice to
form salt and water
ii. Write an ion equation, with state symbols, for the reaction in question B.i.
28. A student adds ammonia solution, little by little, to dilute sulphuric acid in a beaker. The student measures
the pH value of the mixture during the solution of ammonia solution.
A. Describe how the pH value changes as the ammonia solution is added to the sulphuric acid solution.
B. Write the ionic equation to represent the reaction between ammonia solution and dilute sulphuric acid.
C. Ammonia solution is a weak alkali
i. What is meant by the term weak alkali?
ii. Give an example of a strong alkali.
D. Dilute sulphuric acid reacts with calcium carbonate to form carbon dioxide gas
i. State what you would see during the reaction.
ii. Write the chemical equation for the reaction of dilute sulphuric acid and calcium carbonate.

Acids, Bases and Alkalis 95


29. Oxides are classified as acidic, basic, amphoteric or neutral
A. What is an amphoteric oxide?
B. What products are formed when a basic oxide reacts with an acid.
C. Which of the oxides below is likely to be an acidic oxide?
MgO, CuO, SiO2, Al2O3, Na2O

96 Acids, Bases and Alkalis


SOLUTIONS
1. D
2. C
3. A
4. B
5. C
6. C
7. D
8. B
9. A
10. D
11. B
12. B
13. D
14. B
15. D
16. A
17. B
18. D
19. C
20. D
21. A
22. D
23. D
24. B

25. Neutralisation
Ionic equation; OH−(aq) + H+(aq) → H2O(l)

26. A weak acid is an acid that partially dissociates in water. Example of weak acid; phosphoric acid (H3PO4).
A strong acid is an acid that completely ionises in water. Example of strong acid; hydrochloric acid (HCl).

27. A. Dissolve substance S in water to form an aqueous solution. Insert a blue litmus paper strip into
the solution. If the blue litmus paper turns red, then substance S is acidic.
B. i. HCl(aq) + NaOH(aq) → NaCl(aq) + H2O(l)
ii. H+(aq) + OH−(aq) → H2O(l)

28. A. Before adding ammonia, the solution in a beaker containing sulphuric acid has a pH value below
7. As ammonia solution is added the pH value gradually increases from below 7 to above 7.
B. Ionic equation; NH3+(aq) + H+(aq) → NH4+(aq)
C. i. Weak alkali is an alkali that partially ionises in water.
ii. Sodium hydroxide (NaOH)
D. i. Bubbles come out and the solution turns from colourless to milky white.
ii. CaCO3(aq) + CaCO3(s) → CaSO4(aq) + CO2(g) + H2O(l)

29. A. Amphoteric oxides are metal oxides that react with both acids and bases.
B. Salt and water
C. SiO2

Acids, Bases and Alkalis 97


11 SALTS

Introduction
Table salts (sodium chloride) is found in all houses. We use it daily to flavour our foods. However, tablesalt is
not the only salt use in our everyday lives. There are dozens of other salts such as sodium bicarbonate in
baking powder and ammonium nitrate in fertiliser. All salts have similar physical properties. They are
produced by the process called neutralisation, where acid is reacted with a base.

How do salts differ from each other? How are salts prepared? This unit covers these and many more
questions.

Specific outcomes
By the end of this unit, you will be able to:
❖ Define a salt
❖ State physical properties of salts
❖ Differentiate normal salts, acidic salts and basic salts
❖ State the solubility of each salt
❖ Describe how to prepare salts by:
• Filtration method
• Titration method
• Precipitation method
❖ State how to test for the presence of:
• Cations
• Anions
• Gases

98 Salts
SALTS
11.1 SALTS
• Salt is an ionic compound that contains positively charged ions and negatively charged ions.
• Positively charged ions are called cations while negatively charged ions are called anions.
• The commonest salt is sodium chloride (commonly known as table salt), a salt used in everyday life
for food flavouring and preservation.
• Salts are produced by the process called neutralisation. In this process an acid reacts with a base and
forms salt and water.
• Salts usually have a pH of 7.

❖ PROPERTIES OF SALTS
1. They are ionic compounds. They consist of cations and anions.
2. They are hard, crystalline solids.
3. Most salts are colourless to white unless they contain a transition metal.
4. Most salts have a salty taste and are odourless.
5. They conduct electricity in an aqueous and molten solution.
6. Some salts are soluble in water while others are insoluble in water.

11.2 TYPES OF SALTS


1. NORMAL SALTS
• Normal salts are formed when all the hydrogen ions (H+) from the acid have been replaced by
metal ions or ammonium ions (NH4+).
• Normal salts have neither hydrogen (H) nor hydroxyl (OH) in their chemical formula.
• They are electrically neutral.
• Examples of normal salts include sodium chloride (NaCl), potassium nitrate (KNO3), calcium
carbonate (CaCO3) and copper (II) sulphate (CuSO4).

2. ACIDIC SALTS
• Polybasic acids are acids that can donate more than one H+ per molecule.
• Examples of polybasic acids include; sulphuric acid (H2SO4, can donate two H+) and
phosphoric acid (H3PO4, can donate three H+).
• When a polybasic acid is partially neutralised by a base the salt formed is known as acidic salt.
• Acidic salts are formed due to an insufficient supply of base for the complete neutralisation of
the polybasic acid.
• As the polybasic acid could not donate all of its H+ the salt compound formed has hydrogen (H)
atom their chemical formula.
• Examples of acidic salts include sodium bicarbonate (NaHCO3), sodium bisulphate (NaHSO4)
and monopotassium phosphate (KH2PO4).

3. BASIC SALTS
• Polyacid bases are bases that can release more than one OH− to the solution.
• Examples of polyacid bases include; calcium hydroxide (Ca(OH)2), aluminium oxide
(Al(OH)3).
• When a polyacid base is partially neutralised by an acid the salt formed is called basic salt.
• Basic salts are formed due to an insufficient supply of acid for the complete neutralisation of
the polyacid base.
• As the polyacid base could not release all its OH− the chemical formula of the salt formed has a
hydroxyl (OH) in its chemical formula.
• Examples of basic salts include zinc hydroxide chloride (ZnOHCl) and barium hydroxide
chloride (BaOHCl).

Salts 99
11.3 SOLUBILITY OF SALTS
• Some salts are soluble in water while others are insoluble.
• The table shows the solubility of common salts at room temperature.
Salts Soluble Insoluble
Sodium salts All are soluble None
Potassium salts
Ammonium salts
Nitrate salts
Chloride salts Most are soluble Silver halides
Bromide salts Silver chloride (AgCl)
Iodide salts Silver bromide (AgBr)
Silver iodide (AgI)
Lead halides
Lead (II) chloride (PbCl2)
Lead (II) bromide (PbBr2)
Lead (II) iodide (PbI2)
Sulphate salts Most are soluble Barium sulphate (BaSO4)
Lead (II) sulphate (PbSO4)
Carbonate salts Sodium carbonate (Na2CO3) Most are insoluble
Potassium carbonate (K2CO3)
Ammonium carbonate ((NH4)2CO3)

11.4 PREPARATION OF SALTS


• There are three main methods of preparing salts, namely filtration, titration and precipitation.
• The method to use depend on two factors:
1. The solubility of the salt to prepare. Is the salt soluble or insoluble?
2. The solubility of the base to use. Is the base soluble or insoluble?
• When the salt to prepare is soluble using an insoluble base or metal ion filtration method is used.
• When the salt to prepare is soluble using a soluble base titration method is used.
• When the salt to prepare is insoluble using an insoluble base precipitation method is used. The
precipitation method can also be used to prepare an insoluble salt from a soluble base and acid.

1. FILTRATION METHOD
• This method is used to prepare a soluble salt by reacting an insoluble base (can be an oxide
such as CuO, hydroxide such as Mg(OH)2 or carbonate such as ZnCO3) or metal ion with acid.
• The excess insoluble base is reacted with acid to form a soluble salt and water in a beaker.
• At the end of the reaction, the beaker has the soluble salt and water (products) and insoluble
base (excess reactant).
• Then the soluble salt can be separated from the insoluble base by filtration.
• The filtrate is finally evaporated and crystallised to obtain the desired salt.
• This method can be used to prepare salts such as copper (II) sulphate (CuSO4) and magnesium
sulphate (MgSO4), zinc nitrate (Zn(NO3)2) and copper (II) chloride (CuCl2).
CuO(s) + H2SO4(aq) → CuSO4(aq) + H2O(l)
Mg(OH)2(s) + H2SO4(aq) → CuSO4(aq) + H2O(l)
ZnCO3(s) + 2HNO3(aq) → Zn(NO3)2(aq) + H2O(l) + CO2(g)
CuCO3(s) + 2HCl(aq) → CuCl2(aq) + H2O(l) + CO2(g)
Zn(s) + H2SO4(aq) → ZnSO4(aq) + H2(q)

100 Salts
❖ EXPERIMENT: TO PREPARE COPPER (II) SULPHATE
❖ SET UP
1. Add 25cm3 of dilute sulphuric acid in 100cm3 beaker and warm it gently.
2. Add a small portion of copper (II) oxide to the beaker using a spatula and stir with a glass rod
until it dissolves.
3. Add a further small quantity of copper (II) oxide with stirring until it dissolves no more.
4. Allow the solution to cool down.
5. Once the solution is cool, filter it allowing the filtrate to pass into the evaporating dish.
6. Gently heat the solution until a small volume of liquid is left.
7. Leave the remaining solution to cool and crystallise.

❖ ADDITIONAL NOTES
• The reaction between black copper (II) oxide and sulphuric acid produces a blue solution of
copper (II) sulphate and water. This process is called neutralisation.
Word equation
Copper (II) sulphate + sulphuric acid → copper (II) sulphate + water
Chemical equation
CuO(s) + H2SO4(aq) → CuSO4(aq) + H2O(l)

• Warming sulphuric acid increases the rate of reaction. Do not boil the acid.
• When filtered, copper (II) oxide residues remain on the filter paper and copper (II) sulphate is
obtained as filtrates.
• To obtain a larger crystal do not evaporate all the liquid in the evaporating dish. Allow the
solution to cool down slowly.

2. TITRATION METHOD
• Titration method is used to prepare soluble salts by reacting a soluble base with an acid.
• A known volume of the base is reacted with acid using an indicator.
• The indicator shows the correct volume of acid needed to complete the reaction by changing colour.
• Then, the process is repeated with the acquired volume of acid and base and without an indicator.
• This method can be used to make salts such as sodium sulphate (Na2SO4), sodium chloride (NaCl),
potassium sulphate (K2SO4), sodium nitrate (NaNO3) and ammonium nitrate (NH4NO3).
2NaOH(aq) + H2SO4(aq) → Na2SO4(aq) + 2H2O(l)
NaOH(aq) + HCl(aq) → NaCl(aq) + H2O(l)
2KOH(aq) + H2SO4(aq) → K2SO4(aq) + 2H2O(l)
NaOH(aq) + HNO3(aq) → NaNO3(aq) + 2H2O(l)
NH3(aq) + HNO3(aq) → NH4NO3(aq)

Salts 101
❖ EXPERIMENT: PREPARATION OF SODIUM SULPHATE
➢ SET UP
1. Add 25cm3 of sodium hydroxide into a conical flask.
2. Add a few drops of indicator. Note the name of the indicator and the initial colour of the
solution.
3. Fill up the burette with dilute sulphuric acid to the zero mark.
4. Add sulphuric acid from the burette into the conical flask slowly. Swirl the flask carefully to
help sulphuric acid and sodium hydroxide mix until the indicator changes colour.
5. Read the volume of the acid used to neutralise 25cm3 of sodium hydroxide solution.
6. Repeat the procedure now without the indicator.
7. Transfer the solution to the evaporating dish and heat gently to evaporate some liquid.
8. Leave the remaining solution to crystallise.

➢ ADDITIONAL NOTES
• The reaction between sodium hydroxide and sulphuric acid produces sodium sulphate and
water. This process is called neutralisation.
Word equation
Sodium hydroxide + sulphuric acid → sodium sulphate + water

Chemical equation
2NaOH(aq) + H2SO4(aq) → Na2SO4(aq) + 2H2O(l)

• The indicator is used to obtain the volume of sulphuric acid needed to complete the reaction
with the known volume of sodium hydroxide.
• As sulphuric acid is added the pH of the solution lowers until all the base has reacted. Then,
sulphuric acid becomes in excess and causes the indicator to change colour.
• The table below shows the commonest indicators used in titration and their colour change.
Indicator Colour in base Colour in acid
Phenolphthalein Pink Colourless
Methyl orange Yellow Red

102 Salts
3. PRECIPITATION METHOD
• This method is used to prepare insoluble salts by reacting an insoluble base with an acid.
• Precipitation is the process by which two soluble substances in an aqueous solution combine and
form an insoluble salt.
• An insoluble salt formed during precipitation is known as a precipitate.
• The method is a two-stage process. Initially, an insoluble base is reacted with acid to form a soluble
salt. Then the soluble salt formed is reacted with another soluble salt to form a precipitate (a desired
insoluble salt).
• This method is used to prepare insoluble salts such as lead (II) iodide (PbI2), silver chloride (AgCl)
and barium sulphate (BaSO4).

❖ EXPERIMENT: PREPARATION OF LEAD (II) IODIDE


➢ SET UP
1. Add 50cm3 of dilute nitric acid in a beaker and warm it gently.
2. Add a small portion of lead (II) oxide to the beaker using a spatula with stirring until it
dissolves no more.
3. Allow the solution to cool down.
4. Once the solution is cool, filter it allowing the filtrate to pass into the conical flask.
5. Take 25cm3 of the filtrate and put it into a beaker.
6. Add 2cm3 of potassium iodide.
7. Filter the precipitate produced, wash it with water and leave it to dry.

Salts

Salts 103
➢ ADDITIONAL NOTES
• The precipitation method is a two stage process.
• The first stage involves the formation of a soluble salt by reacting with an insoluble and acid. For
the above experiment, lead (II) nitrate is formed by reacting lead (II) oxide (insoluble base) and
nitric acid.
PbO(s) + 2HNO3(aq) → Pb(NO)2(aq) + H2O(l)
• The second stage is a precipitation process which involves the formation of an insoluble salt by
reacting two soluble salts. For the above experiment, lead (II) nitrate (a soluble salt produced in the
first stage) is reacted with potassium iodide (another soluble salt) and produced lead (II) iodide (an
insoluble salt).
Pb(NO)2(aq) + 2KI(aq) → PbI2(aq) + 2KNO3(aq)

11.5 CHEMICAL TESTS


• Chemical tests are used to identify chemical substances or ions in a solution.
• Chemical tests are based on the fact that chemical substances have different properties such as
colour, solubility, and how they react with each other.

❖ TEST FOR CATIONS


Cations React with NaOH
Aluminium ion (Al3+) A white precipitate forms. The precipitate dissolves in
excess NaOH and gives a colourless solution
Calcium ion (Ca2+) A white precipitate forms
Copper ion (Cu2+) A light blue precipitate form
Iron (II) ion (Fe2+) A dark green precipitate forms
Iron (III) ion (Fe3+) A red-brown precipitate forms
Lead (II) ion (Pb2+) A white precipitate forms
Zinc ion (Zn2+) A white precipitate forms. The precipitate dissolves in
excess NaOH
Ammonium ion (NH4), a No precipitate forms
non-metal cation Ammonia gas is produced (it has a pungent smell)

❖ TEST FOR ANIONS


Anions Test Observation
Sulphate ion Add dilute HCl, then barium nitrate A white precipitate of
(SO42−) solution barium sulphate is produced
Nitrate ion Add aqueous NaOH and a small Ammonia gas is produced
(NO3−) piece of aluminium. Warm the The gas turns damp red
mixture litmus paper blue
Carbonate ion Add HCl. Pass the gas produced in Carbon dioxide is produced
(CO32−) limewater It forms a precipitate with
limewater
Chloride ion Add dilute HNO3, then silver nitrate A white precipitate of silver
(Cl−) nitrate is formed
Iodide ion (I−) Add dilute HNO3, then lead (II) A yellow precipitate of lead
nitrate (II) nitrate is formed

104 Salts
❖ TEST FOR GASES
Gas Test Observation
Ammonia (NH3) Place a damp red litmus paper in gas The red litmus paper turns blue
Carbon dioxide Bubble the gas through limewater A white precipitate of calcium
(CO2) carbonate is produced
Chlorine (Cl2) Place a damp blue litmus paper in gas The blue litmus paper turns red
Hydrogen (H2) Place a lighted splint in gas The lighted splint burns with a pop
sound
Oxygen (O2) Place a lighted splint in gas The growing splint relights
Sulphur dioxide Place a paper soaked with potassium The paper turns from orange to
(SO2) dichromate in gas green

Salts 105
REVIEW QUESTIONS
1. The process in which acids and bases reacts to form salts and water is called
A. Hydrogenation
B. Condensation
C. Neutralisation
D. Halogenation
2. The following is not a physical property of salts
A. Salts conduct electricity in an aqueous solution
B. Salts are covalent compounds
C. Salts are hard, crystalline solids
D. Some salts are insoluble in water
3. A salt formed by the complete neutralisation of H+ and OH− in the solution is known as
A. Normal salts
B. Basic salts
C. Acidic salts
D. None of the above
4. A salt formed by partial neutralisation of an acid by a base is known as
A. Normal salt
B. Basic salt
C. Acidic salt
D. None of the above
5. A salt formed by partial neutralisation of a base by acid is known as
A. Normal salt
B. Basic salt
C. Acidic salt
D. None of the above
6. Which of the following is an example of a normal salt?
A. NaHCO3
B. ZnOHCl
C. KH2PO4
D. NaCl
7. Which of the following is an example of an acidic salt?
A. Na2CO3
B. NaHSO4
C. ZnOHCl
D. Pb(NO)2
8. Which of the following is an example of a basic salt?
A. NaHSO4
B. CaCO3
C. CuSO4
D. BaOHCl
9. Among the halides, which of the following salt is soluble?
A. AgI
B. PbBr2
C. PbCl2
D. MgCl2
10. Among the sulphate salts, which of the following salt is insoluble?
A. PbSO4
B. Na2SO4
C. CuSO4
D. MgSO4

106 Salts
11. Among the carbonate salts, which of the following salt is insoluble?
A. K2CO3
B. (NH4)2CO3
C. Na2CO3
D. CaCO3
12. Which is the best pair of substances that can be used to prepare copper (II) sulphate?
A. Cu and H2SO4
B. CuO and H2SO3
C. Cu(OH)2 and H2SO3
D. CuCO3 and H2SO4
13. Which of the salts below can be prepared by an acid-alkali titration method?
A. CuSO4
B. MgCl2
C. NaNO3
D. Zn(NO3)2
14. Which of the following salts cannot be crystallised from an aqueous solution?
A. Barium chloride
B. Magnesium chloride
C. Silver chloride
D. Sodium ethanoate
15. Which of the following salts can be crystallised from its solution?
A. Barium sulphate
B. Lead iodide
C. Silver chloride
D. Silver nitrate
16. Which compound is not formed by the precipitation method?
A. NH4Cl
B. PbSO4
C. BaSO4
D. AgCl
17. Which of the following salts can be prepared by precipitation method?
A. Ammonium sulphate
B. Barium sulphate
C. Lead (II) nitrate
D. Silver nitrate
18. Silver chloride when precipitated by mixing aqueous solutions of silver nitrate and hydrochloric acid, is
initially a white precipitate but turns grey on standing. The reason for this change is that the precipitate
A. Is decomposed by light
B. Reacts with oxygen in the air
C. Reacts with dilute hydrochloric acid
D. Reacts with an excess silver nitrate solution
19. What is the test for oxygen gas?
A. Damp blue litmus paper turns red
B. Damp red litmus paper turns blue
C. Glowing splint relights
D. Lit splint goes with a sound

Salts 107
20. What is the test for carbon dioxide?
A. Limewater turns cloudy
B. Glowing splint relights
C. Lit splint goes with a sound
D. Damp blue litmus paper turns red
21. What is the test for chlorine gas?
A. Glowing splint relights
B. Lit splint goes with a sound
C. Damp blue litmus paper turn red and is then bleached white
D. Damp red litmus paper turns blue
22. What is the test for ammonia gas?
A. Glowing splint relights
B. Lit splint goes with a sound
C. Damp blue litmus paper turn red and is then bleached white
D. Damp red litmus paper turns blue
23. What is the test for hydrogen gas?
A. Glowing splint relights
B. Lit splint goes with a sound
C. Damp blue litmus paper turn red and is then bleached white
D. Damp red litmus paper turns blue
24. What colour precipitate does chlorine ion produce?
A. Yellow
B. Blue
C. Green
D. White
25. What colour precipitate do copper ions produce?
A. Yellow
B. Blue
C. Green
D. White
26. A. Copper (II) sulphate crystals can be prepared in the laboratory by reacting copper (II) carbonate
with a dilute acid
i. Name the dilute acid which reacts with copper (II) carbonate to form copper (II) sulphate.
ii. Write a balanced chemical equation with state symbols for the reaction.
iii. Describe the procedure for the preparation of copper (II) sulphate solution from copper (II)
carbonate and the named acid in i. above.
B. Describe what is observed when an excess of sodium hydroxide solution is added to a solution
containing copper (II) ions and name one product formed.
C. Describe a chemical test to show the presence of sulphate ions in the solution and state what
would be observed.
27. A. Sodium sulphate is a soluble salt that can be prepared from dilute sulphuric acid
i. Name the other chemical that can be reacted with dilute sulphuric acid to prepare the salt.
ii. Write a balanced chemical equation with state symbols for the reaction above.
iii. Describe in detail how you would prepare pure crystals of sodium sulphate using the named
reactants.
B. The salt prepared is a normal salt
i. What is a normal salt?
ii. If the reaction above the reactants were to form an acid salt, give the name of the acid salt
that would be formed from the two reactants and give its chemical formula.

108 Salts
28. The word equation for the reaction used to for the insoluble salt barium sulphate is shown below
Barium nitrate + sodium sulphate → barium sulphate + sodium nitrate
i. Write the ionic equation for this reaction.
ii. Describe how a pure, dry sample of barium sulphate can be obtained from the reaction.
iii. What term is used to describe the method used to prepare the barium sulphate?

Salts 109
SOLUTIONS
1. C
2. B
3. A
4. C
5. B
6. D
7. B
8. D
9. D
10. A
11. D
12. D
13. C
14. C
15. D
16. A
17. B
18. A
19. C
20. A
21. C
22. D
23. B
24. D
25. B

26. A. i. Sulphuric acid


ii. CuCO3(s) + H2SO4(aq) → CuSO4(aq) + H2O(l)
iii. Add dilute sulphuric acid to a beaker and warm it gently. Add copper (II) carbonate slowly
and while stirring. All the solutions to cool down. Once the solution is cool, filter it using a
filter funnel and paper, allowing the filtrates into the evaporation dish. Gently heat the
solution. Leave the remaining solution of copper (II) sulphate to cool and crystallise.
B. The solution forms a blue precipitate. Copper hydroxide (Cu(OH)2) is formed.
C. Add the solution containing sulphate ions into the beaker. Add few drops of hydrochloric acid.
Then, add few drops of barium nitrate solution. If sulphate ions are present, a white precipitate
is formed.

27. A. i. Sodium hydroxide


ii. 2NaOH(aq) + H2SO4(aq) → Na2SO4(aq) + 2H2O(l)
iii. Add sodium hydroxide into a conical flask. Add a few drops of indicator (phenolphthalein
or methyl orange). Note the name of the indicator and the initial colour of the solution. Fill
up the burette with dilute sulphuric acid to the zero mark. Add acid from the burette into the
conical flask slowly. Swirl the flask carefully until the indicator changes colour. Read the
volume of the acid used to neutralise sodium hydroxide solution. Repeat the procedure now
without the indicator. Transfer the solution to the evaporating dish and heat gently to
evaporate some liquid. Leave the remaining solution to crystallise.
B. i. Normal salt is a salt formed when metal ions replace all hydrogen ions from the acid or
ammonium ions.
ii. Sodium bisulphate, NaHSO4

110 Salts
28. i. Ba2+(aq) + SO2−(aq) → BaSO4(s)
ii. Put sodium sulphate into a beaker. Add barium sulphate t the beaker. Stir the solution gently.
A precipitate of barium sulphate is formed. Filter the precipitate from the solution using a filter
funnel and paper. The precipitate remains on the filter paper. Wash the precipitate with water
and leave it to dry in a warm place.
iii. Precipitation

Salts 111
12 CHEMICAL REACTIONS

Introduction
Substances combine or break down chemically and form other substances by the process referred to as
chemical reaction. Substances used up in the chemical reaction are called reactants and substances formed are
called products. During the chemical reaction, bonds of reactants are broken down and form new bonds of
products.

What factors affect the rate of reaction? This unit covers this question and many more.

Specific outcomes
By the end of this unit, you will be able to
❖ Write a chemical reaction
❖ Describe the collision theory
❖ Describe experiments used to determine the rate of reaction
❖ Describe the factors that affect the rate of reaction
❖ Describe the Le Chatelier’s principle
❖ Haber process:
• State conditions for Haber process
• State the importance of the Haber process

112 Chemical Reactions


CHEMICAL REACTIONS
12.1 CHEMICAL REACTION
• A chemical reaction is a process in which one or more substances are converted to one or more
different substances
• Substances can be elements or compounds
• A chemical reaction rearranges the constituent atoms of initial substances and creates different
substances as products
• Bonds in reactants are broken, atoms rearrange and form new bonds in products
AB + CD → AD + BC

❖ COMPONENTS OF THE CHEMICAL REACTION

• Reactants: Substances used up in a chemical reaction. They are placed on the left side of the
chemical equation.
• Products: Substances formed at the end of the chemical reaction. They are placed on the right
side of the chemical equation.
• Yield arrow: The arrow that indicates the direction of the reaction.
• Coefficient: The number in front of the chemical symbol. It indicates how many molecules of a
substance participate in a chemical reaction.
• The chemical equation has to be balanced.

12.2 RATE OF CHEMICAL REACTION


• Rate of reaction is defined as the measure of how fast reactants are consumed and products are
formed.
• The study of the rate of reaction is called chemical kinetics.

❖ COLLISION THEORY
• Collision theory states that for a chemical reaction to take place reacting atoms, molecules or
ions must collide with each other.
• The collision should have sufficient energy to break the bonds in reactants. This energy is
called activation energy.
• Reactants should collide with proper orientation. Proper orientation is a collision that occurs at
a right angle and with atoms involved in bond formation.
• The collision that has sufficient energy and with proper orientation is called an effective
collision.
• The more effective collisions, the faster the rate of reactions.

Chemical Reactions 113


12.3 MEASURING THE RATE OF REACTIONS
• There are two common ways of measuring the rate of reaction.
• Measuring the volume of gas produced against time, and,
• Measuring the change in mass of reactants against time.

❖ EXPERIMENT: MEASURE THE VOLUME OF GAS PRODUCED AGAINST TIME


➢ SET UP
• Pour 50cm3 of dilute hydrochloric acid into
the conical flask.
• Add few marble chips (calcium carbonate)
into the flask.
• Seal the flask with cork and start the
stopwatch.
• Record the reading on the gas syringe
every 10 seconds.
• Plot a graph of the results, with time
on the x-axis and volume of gas
produced on y-axis.

➢ RESULTS
• The gradient on the graph is steepest at
the start of the reaction.
• Gradient gradually decreases with time.
• Finally, the gradient becomes zero.

➢ INTERPRETATION OF THE RESULTS


• Calcium carbonate in marble chips reacted with hydrochloric acid producing calcium
chloride, water and carbon dioxide.
CaCO3(s) + 2HCl(aq) → CaCl2(aq) + H2O(l) + CO2(g)
• Carbon dioxide is collected in the gas syringe which measures the amount produced.
• The steeper the gradient on the graph, the faster the rate of reaction.
• The gradient is steepest at the start of the reaction indicating that the rate of reaction is
fastest at the start of the reaction. At this point, the concentration of calcium carbonate and
hydrochloric acid is high. The high reactants concentration led to the fastest rate of
reaction.
• The gradient gradually decreases with time indicating that the rate of reaction slows with
time. This is due to a decrease in reactants concentration resulting in a decrease in carbon
dioxide formation.
• Zero gradient means no reaction is taking place. Therefore, zero gradients at the end of the
reaction indicate that the reaction is complete. At this point, the limiting reactants are used
up.

➢ CONCLUSION
• The rate of reaction is inversely proportional to the time taken.

114 Chemical Reactions


❖ EXPERIMENT: MEASURE THE CHANGE IN MASS OF REACTANTS
AGAINST TIME
➢ SET UP
• Pour 50cm3 of dilute hydrochloric acid into
the conical flask.
• Add few marble chips (calcium carbonate)
into the flask.
• Cover the flask with cotton wool to allow
air to escape but prevent marble chips and
acid from splashing from the flask.
• Record the mass on the electronic balance
every 10 seconds.
• Plot a graph of the results with time on the x-axis and change in mass on the y-axis.

➢ RESULTS
• The results produce a negative gradient.
• The gradient on the graph is steepest at
the start of the reaction.
• Gradient gradually decreases with time.
• Finally, the gradient becomes zero.

➢ INTERPRETATION OF THE RESULTS


• Calcium carbonate in marble chips reacts with hydrochloric acid producing calcium chloride,
water and carbon dioxide.
CaCO3(s) + 2HCl(aq) → CaCl2(aq) + H2O(l) + CO2(g)
• The negative gradient indicates that the mass of reactants decreases as the reaction proceed. The
decrease in mass is due to carbon dioxide that is formed and escapes the reaction flask.
• The steeper the gradient, the faster the rate of reaction.
• The gradient is steepest at the start of the reaction indicating that the rate of reaction is fastest at
the start of the reaction. At this point, the reactants concentrated is high. This leads to high
carbon dioxide formation which escapes the flask. Therefore, a rapid decrease in mass.
• The gradient gradually decreases with time indicating that the rate of reaction slows with time.
This is due to a decrease in reactant concentration.
• Zero gradient means no reaction is taking place. Therefore, zero gradients at the end of the
reaction indicate that the reaction is complete. At this point, the limiting reactant is used up.

➢ CONCLUSION
• The rate of reaction is inversely proportional to the time taken.

Chemical Reactions 115


12.4 FACTORS AFFECTING THE RATE OF REACTION
1. CONCENTRATION OF REACTANTS
• Increasing the concentration of reactants increases the rate of reaction.
• An increase in reaction rate is because there are more particles per unit volume. As a result, the
frequency of effective collision increases.

2. SIZE OF REACTANT PARTICLES


• Reducing the size of solid reactant particles increases the rate of reaction.
• Breaking solid reactants increase the surface area. Therefore, there is an increased surface area
for the other reactants to collide with increasing effective collision frequency.

3. TEMPERATURE
• Increasing temperature increases the rate of reaction.
• At higher temperature particles move faster and collide more effectively. This increases the
frequency of effective collision.

4. PRESSURE OF REACTANTS
• Only gases are compressible, so pressure change only affects reactions that involve gases.
• The rate of reaction increases with an increase in pressure.
• Higher pressure moves particles closer. This increases the frequency of effective collision.

5. PRESENCE OF CATALYST
• Catalysts are chemical substances that speed up a chemical reaction without themselves being
changed.
• They speed up a reaction by lowering the activation energy. Activation energy is the minimum
amount of energy required to start a chemical reaction.
• Just a small amount of catalyst is required.
• Catalyst can be reused.
• Most catalysts are transition metals such as titanium, nickel, iron, copper.

12.5 CHEMICAL EQUILIBRIUM


• Chemical equilibrium is a state at which a forward reaction is equal to the backward reaction of a
chemical reaction.
• In other terms, it is a state when reactants are combining to form products and products are breaking
to form reactants at the same pace.
• Reactions at equilibrium are indicated by a double arrow (⇋).
N2(g) + 3H2(g) ⇋ 2NH3(g)
NH3(g) + HCl(g) ⇋ NH4Cl(s)

116 Chemical Reactions


❖ LE CHATELIER’S PRINCIPLE
• When the rate of reaction is zero, the net concentration of reactants and products is equal. At this
stage, the chemical reaction is at equilibrium.
• Le Chatelier’s principle states that if there is a change in the system due to a change in
concentration, temperature or pressure, equilibrium will shift to offset the change until a new
equilibrium is established.
• In other terms, if a chemical reaction is at equilibrium any change in reactants or products
concentration, temperature or pressure will resume the reaction toward products or reactants until it
reaches another chemical equilibrium.
• Increasing reactants concentration or reducing products concentration shifts the equilibrium toward
the products. Therefore, more products are formed. The vice versa is also true.
• Increasing pressure shifts the equilibrium toward the side with fewer molecules of gas. Add
coefficients on both sides of a balanced chemical reaction. The side with fewer coefficients is where
the equilibrium will shift when pressure is increased and vice versa.
• In endothermic reactions, increasing temperature shifts the equilibrium toward products. In
exothermic reactions, increasing temperature shifts the equilibrium toward reactants.
• Catalyst does not shift the equilibrium but only causes the reaction to reach equilibrium faster.

12.6 HABER PROCESS


• Ammonia is manufactured using the Haber process.
• The process uses nitrogen gas and hydrogen gas.
N2(g) + 3H2(g) ⇋ 2NH3(g) ΔH = − 92 kJ/mol
• Nitrogen is obtained from air.
• Hydrogen is produced from the reaction of methane and steam under high pressure.
• The Haber process is a reversible reaction. Therefore, if ammonia concentration increases the
reverse reaction, decomposition of ammonia to nitrogen and hydrogen occurs.

❖ EFFECTS OF TEMPERATURE
• The Haber process is an exothermic process, a process that releases heat to the surrounding.
• Therefore, increasing temperature shifts the equilibrium toward reactants and decreases the
formation of ammonia.
• However, low temperature reduces the rate of reaction.

❖ EFFECTS OF PRESSURE
• The Haber process has four molecules on reactants and two molecules on products.
• Therefore, increasing pressure shifts the equilibrium toward the product, so the formation of
ammonia increases.

Chemical Reactions 117


❖ CONDITIONS FOR THE HABER PROCESS
1. Temperature of 450oC
• Higher temperature favours the decomposition of ammonia into nitrogen and hydrogen.
• Low temperature causes the reaction to be too slow.
• Therefore, a relative temperature of 450oC is used.

2. Pressure of 200 atmospheric pressure


• High pressure increases ammonia formation.
• However, it is too expensive to achieve high pressure and equipment cannot withstand extreme
pressure.
• Therefore, 200 atmospheric pressure is used.

3. Iron catalyst
• An iron catalyst increases the rate of reaction.

❖ IMPORTANCE OF HABER PROCESS


• The Haber process is used to manufacture ammonia which is needed for fertiliser.

118 Chemical Reactions


REVIEW QUESTIONS
1. The substances that are present before a chemical reaction takes place are called
A. Reactants
B. Products
C. Coefficients
D. Elements
2. Substances that are formed in a chemical reaction are called
A. Reactants
B. Products
C. Coefficients
D. Elements
3. Numbers in front of the chemical symbol which indicate the number of molecules participating in a
reaction is called
A. Reactants
B. Products
C. Coefficients
D. Elements
4. A reversible reaction is indicated by
A. a forward arrow
B. a backward arrow
C. an upright arrow
D. a double arrow
5. Which of the following statement is true about what happens during a chemical reaction?
A. Bonds of reactants are broken, and bonds of products are formed
B. Bonds of reactants are formed, and bonds of products are broken
C. Bonds of reactants and products are broken
D. Bonds of reactants and products are formed
6. To be effective, a collision requires
A. Sufficient energy
B. Proper orientation
C. Sufficient energy and proper orientation
D. Reaction mechanism
7. The minimum amount of energy required to initiate a chemical reaction is called
A. Kinetic energy
B. Free energy
C. Energy of enthalpy
D. Activation energy
8. In general, if the temperature of a chemical reaction is increased the rate of reaction
A. Increases
B. Decreases
C. Remain the same
D. Cannot be predicated
9. If the concentration of reactants is low, the rate of reaction
A. Increases
B. Decreases
C. Remain the same
D. Cannot be predicted

Chemical Reactions 119


10. If the pressure of reactants is increased, the rate of reaction
A. Increases
B. Decreases
C. Remains the same
D. Cannot be predicted
11. If a catalyst is added to a chemical, the rate of reaction
A. Increases
B. Decreases
C. Remain the same
D. Cannot be predicted
12. The equation, N2(g) + 3H2(g) ⇋ 2NH3(g) shows a system
A. in a chemical equilibrium
B. in physical equilibrium
C. that does not reach equilibrium
D. that does not change
13. Which of the following does not affect the equilibrium of a reaction?
A. Adding a catalyst
B. Increasing temperature
C. Decreasing concentration of reactants
D. Increasing pressure
14. Which of the following correctly explain how a catalyst affects the rate of reaction?
A. Catalyst increases the rate of reaction by increasing activation energy
B. Catalyst increases the rate of reaction by decreasing activation energy
C. Catalyst decrease the rate of reaction by increasing activation energy
D. Catalyst decrease the rate of reaction by decreasing activation energy
15. With the other factors being constant, which of the following would give the fastest reaction when
calcium carbonate reacts with dilute hydrochloric acid, HCl?
A. 100g of HCl in 1dm3
B. 50g of HCl in 500dm3
C. 20g of HCl in 50dm3
D. 20g of HCl in 100dm3
16. Ammonia is made by a reversible reaction between nitrogen and hydrogen
N2(g) + 3H2(g) ⇋ 2NH3(g) ΔH = − 92 kJ/mol
What is the effect of reducing the temperature to the equilibrium reaction above?
A. Less ammonia is formed
B. Less heat heated is formed
C. More ammonia is formed
D. More nitrogen is formed

120 Chemical Reactions


17. Some marble chips (calcium carbonate) were reacted with excess hydrochloric acid as shown by the
equation below.
CaCO3(s) + 2HCl(aq) → CaCl2(aq) + H2O(l) + CO2(g)
Carbon dioxide produced was collected in a gas syringe and its volume was recorded every 10s.

The results are given in the table below.


Time/s 0 10 20 30 40 50 60 70 80
Volume/cm3 0 25 45 55 65 70 70 70 70
A. Plot a graph of the volume of carbon dioxide produced in cm3 against time in seconds.
B. Use your graph to determine the
i. volume of gas produced in the first 25s of the reaction.
ii. time taken to produce 60cm3 of carbon dioxide gas.
C. How long did it take for the reaction to end?
D. What was the total volume of gas produced in the experiment?
E. State two factors that can be altered to increase the rate of the above reaction.

18. A learner used the experimental arrangement below to investigate the effect of concentration on the rate of
reaction between magnesium and excess dilute hydrochloric acid.

The learner experimented three times using different concentrations of hydrochloric acid. The same mass
of magnesium ribbon was used in each case and the experiments were carried out at a temperature of
25oC. The results are shown in the table below.
No. Concentration of hydrochloric acid Volume of hydrogen formed
(mol/dm3) (min/dm3)
1 0.5 15
2 1.0 30
3 1.5 45
A. From the results, what is the effect of concentration on the rate of reaction between magnesium ribbon
and dilute hydrochloric acid?
B. Explain why concentration affects the rate of reaction.
C. State the effect on the rate of reaction if
i. magnesium powder was used instead of ribbon.
ii. the reaction was carried at 15oC of 25oC.

Chemical Reactions 121


SOLUTIONS
1. A
2. B
3. C
4. D
5. A
6. C
7. D
8. A
9. B
10. A
11. A
12. A
13. A
14. B
15. A
16. C

17. A.

B. i. 48cm3
ii. 34 seconds
C. 50 seconds
D. 70 cm3
E. Temperature and concentration of hydrochloric acid.

18. A. Increasing the concentration of hydrochloric acid increases the rate of reaction.
B. Increasing the concentration of reactants increases the surface area, so more reactant collides.
C. i. Rate of reaction would increase.
ii. Rate of reaction would decrease.

122 Chemical Reactions


13 ENERGY CHANGES

Introduction
All chemical reactions involve some change in energy. Some reactions release energy to the surrounding
while others absorb energy from the surrounding. Chemical reactions that release energy to the surrounding
are called exothermic reactions. Chemical reactions that absorb energy from the surrounding are called
endothermic reactions.

Specific outcomes
This unit covers energy changes that occur during a chemical reaction. By the end of this unit, you will be
able to:
❖ Enthalpy change:
• Define enthalpy change
• Calculate the enthalpy change of reactions
❖ Define activation energy and transition state
❖ Describe exothermic reactions
❖ Describe endothermic reactions
❖ Describe different types of fuels

Energy Changes 123


ENERGY CHANGES
13.1 ENTHALPY CHANGE (ΔH)
• Enthalpy change is the amount of heat absorbed or released during a chemical reaction.
• The symbol for enthalpy change: ΔH
• Unit: J/mol (commonly used unit is kJ/mol).
• During a chemical reaction bonds in reactants are broken then new bonds are formed in products.
• Bond breaking absorbs heat from the surrounding while bond formation releases heat to the
surrounding.
• Enthalpy change is simply the difference between heat released during bond formation in reactants
and heat absorbed during the bond breaking in products.

ΔH = H(products) + H(reactants) where


ΔH = enthalphy change
H(products) = heat involved in product bond formation
H(reactants) = heat involved in reactant bond breaking

❖ EXAMPLES
1. Hydrogen and bromine react to form hydrogen bromide according to the equation:
H2(g) + Br2(g) → 2HBr(g)
The table below shows some bond energies.
Bond Bond energy (kJ/mol)
H−H 436
H − Br 366
Br − Br 193

A. Calculate the amount of energy needed to break the bond in 1 mole of H2 and 1 mole of Br2.
B. Calculate the amount of energy produced when 2 moles of hydrogen bromide are produced.
C. Find the ΔH of the reaction above.

2. Hydrogen and chlorine react to form hydrogen chloride gas according to the equation:
H2(g) + Cl2(g) → 2HCl(g)
Calculate the enthalpy change for the reaction above using the bond energy below.
Bond Bond energy (kJ/mol)
H−H 436
H − Cl 431
Cl − Cl 242

3. Hydrogen and oxygen react to form water according to the equation:


2H2(g) + O2(g) → 2H2O(g)
Calculate the enthalpy change for the reaction above using the bond energy below.
Bond Bond energy (kJ/mol)
H−H 436
O=O 490
H−O 464

124 Energy Changes


❖ SOLUTIONS
1. A. Energy absorbed to break H − H and Br − Br
H = 436 + 193
= + 629 kJ/mol Positive sign indicates energy absorbed

B. Energy released to form 2 H − Br


H = − 2(366)
= − 732 kJ/mol Negative sign indicates energy absorbed

C. ΔH = H(reactant) + H(product)
= − 732 + 629
= − 103 kJ/mol Negative sign indicates energy absorbed

2. Energy absorbed to break H − H and Cl − Cl


H = 436 + 242
= + 678 kJ/mol

Energy released to form 2 H − Br


H = − 2(431)
= − 862 kJ/mol

ΔH = H(reactant) + H(product)
= − 862 + 678
= − 184 kJ/mol

3. Energy absorbed to break 2 H − H and O = O


ΔH = 2(436) + 490
= + 1362 kJ/mol

Energy released to form 2 H − O − O (2 × 2 H − O)


ΔH = − 2 × 2(464)
= − 1856 kJ/mol

ΔH = H(reactant) + H(product)
= −1856 + 1362
= − 494 kJ/mol

Energy Changes 125


13.2 ACTIVATION ENERGY AND TRANSITION STATE
❖ ACTIVATION ENERGY
• Activation energy is the amount of energy required to start a reaction.
• The energy is needed to break the reactant’s bonds.

❖ TRANSITION STATE
• The transition state is the unstable intermediate state of deformed bonds.
• It has the highest energy state along with the reaction.

13.3 EXOTHERMIC REACTIONS


• An exothermic reaction is a reaction that releases energy to the surrounding.
reaction → product + energy
• Energy is released usually in form of heat or light.
• These reactions create new bonds therefore release energy.
• The reactants initially require activation energy to break bonds.
• The energy released upon new bond formation is greater than the energy needed to break bonds of
reactants.
• Therefore, enthalpy (ΔH) of exothermic reactions is negative.
• The diagram shows an exothermic reaction graph.

• Examples of exothermic reactions


1. Combustion
CH4(g) + 2O2(g) → CO2(g) + 2H2O(g) + energy ΔH = − 890 kJ/mol

2. Formation of ammonia
N2(g) + 3H2(g) → 2NH3(g) + energy ΔH = − 46 kJ/mol

3. Formation of water
1
H2(g) + 2O2(g) → H2O(g) + energy ΔH = − 286 kJ/mol

4. Respiration
C6H12O6(aq) + 6O2(g) → 6CO2(g) + 6H2O(l) + energy ΔH = − 2803 kJ/mol

126 Energy Changes


13.4 ENDOTHERMIC REACTIONS
• An endothermic reaction is a reaction that absorbs energy from the surrounding.
reactants + energy → products
• Endothermic reactions break down bonds that require energy.
• Reactants initially require activation energy.
• The energy required during bond breaking is greater than the energy released during bond
formation.
• Therefore, enthalpy (ΔH) of endothermic reactions is positive.
• The diagram shows an exothermic reaction graph.

• Examples of endothermic reactions


1. Photosynthesis
6CO2(g) + 6H2O(l) + energy → C6H12O6(aq) + 6O2(g) ΔH = + 2803 kJ/mol

2. Melting
ice + energy → liquid

3. Decomposition by heat
CaCO3(s) + heat → CaO(s) + CO2(g) ΔH = + 178 kJ/mol
1
H2O(l) + heat → H2(g) + 2O2(g) ΔH = + 286 kJ/mol
3
Al2O3(s) + heat → 2Al(s) + 2O2(g) ΔH = + 1676 kJ/mol

13.5 FUEL
• A fuel is any substance that produces a large amount of energy when it undergoes a chemical
reaction.
• Fuels undergo exothermic reactions and produce heat.
• There are various forms of fuels.

1. FOSSIL FUEL
• Fossil fuels are mainly hydrocarbon compounds in wood, coal, crude oil and natural gas.
• Burning of these fuels with oxygen, a process called combustion, releases a large amount of
energy that is used in homes, transport and industries.
• Disadvantages of fossil fuels
i. They are non-renewable. Their supply is limited.
ii. Produces carbon dioxide which contributes to global warming.
iii. Incomplete combustion produces carbon monoxide which may be harmful.
iv. Some contain sulphur which during combustion become sulphur dioxide. This may
eventually become acid rain.

Energy Changes 127


2. HYDROGEN
• Burning hydrogen with oxygen produces a large amount of heat energy.
• This reaction produces steam which is non-pollutant to the environment.
2H2(g) + O2(g) → 2H2O(g) + energy
• Hydrogen as a fuel is used in rocket.
• Disadvantage: it is very expensive.

3. NUCLEAR ENERGY
• Nuclear energy is the energy released by the decay of radioactive isotopes.
• Nuclear fission, the splitting of a radioactive isotope into smaller nuclei, produces a huge amount of
energy.
• Uranium is commonly used.
• Heat energy produced is used to boil water into steam that turns a turbine, producing electricity.
• Disadvantages of nuclear energy.
i. Expensive to build.
ii. Radioactive are harmful to health.
iii. Radioactive isotopes such as uranium are limited.

128 Energy Changes


REVIEW QUESTIONS
1. Enthalpy change is the amount
A. temperature change of a system
B. pressure changes in a system
C. energy absorbed or released from the system
D. all the above
2. What happens during a chemical reaction?
A. Old bonds are broken and new bonds are formed
B. Old bonds are formed and new bonds are broken
C. Old and new bonds are formed
D. Old and new bonds are broken
3. The minimum amount of energy needed to initiate a chemical reaction is called?
A. Enthalpy
B. Transition state
C. Energy change
D. Activation energy
4. The greater the activation energy
A. The lesser the energy needed to start a reaction
B. The more energy needed to start a reaction
C. The lesser the energy needed to complete a reaction
D. The more energy needed to complete a reaction
5. The symbol Δ stands for
A. mass of
B. change in
C. rate of
D. heat stored
6. The difference between the enthalpies of the products and reactants is denoted by
A. ΔH
B. H
C. ΔG
D. G
7. For exothermic reactions, ΔH is always
A. zero
B. positive
C. negative
D. constant
8. For an exothermic reaction, the products
A. The energy released in bond formation is greater than the energy used in bond breaking
B. The energy absorbed in bond breaking is greater than the energy released in bond formation
C. The energy released in bond formation is the same as the energy used in bond breaking
D. The net energy released and used is always zero
9. For endothermic reactions, ΔH is always
A. zero
B. positive
C. negative
D. constant
10. For an endothermic reaction, the products
A. The energy released in bond formation is greater than the energy used in bond breaking
B. The energy absorbed in bond breaking is greater than the energy released in bond formation
C. The energy released in bond formation is the same as the energy used in bond breaking
D. The net energy released and used is always zero

Energy Changes 129


11. Methane is a hydrocarbon whose formula is CH4. The gas burns completely air to produce carbon dioxide,
water and energy.
A. i. Write a balanced chemical equation for the complete combustion of methane.
ii. Why should methane never be burnt in limited air?
B. i. Methane is used as fuel in the industry. Using the bond energies in the table below, calculate the
enthalpy of combustion of methane.
Bond Bond energy (kJ/mol)
C−C 348
C−H 412
C=O 831
H−O 464
O=O 490
ii. Calculate the amount of heat that would be produced from the combustion of 1200cm3 of
methane gas measured at r.t.p.
C. Draw the energy level diagram for the combustion of methane.

12. The diagram below shows the energy level diagram for tissue respiration.

A. What does X represent?


B. Is tissue respiration exothermic or endothermic? Give a reason for your answer.
C. Calculate the amount of heat change involved when 3.6g of glucose reacts completely with oxygen.

130 Energy Changes


13. The diagram below shows an energy level diagram for the burning of hydrogen.

A. What energy change occurs when hydrogen burns?


B. What is the enthalpy of combustion of hydrogen?
C. Is the reaction that occurs in burning hydrogen endothermic or exothermic? Give a reason for
your answer.
D. What is X and why is it important?
E. State the enthalpy change of reaction for the reaction represented by the chemical equation below.
2H2O(g) → 2H2(g) + O2(g)

Energy Changes 131


SOLUTIONS
1. C
2. A
3. D
4. B
5. B
6. A
7. C
8. A
9. B
10. B

11. A. i. CH4(g) + 2O2(g) → CO2(g) + 2H2O(l)


ii. With limited air, carbon monoxide instead of carbon dioxide is produced. Carbon monoxide
is harmful.

B. i. Energy absorbed to break CH4 and 2O2


H = 4(412) + 2(490)
= + 2628 kJ/mol

Energy released to form CO2 and 2H2O


H = − 2(831) − 2(2 × 464)
= − 3518 kJ/mol

ΔH = H(reactant) + H(product)
= − 3518 + 2628
= − 890 kJ/mol

volume given
ii. n =
molar volume
1.2
=
24
= 0.05 mol

H = 0.05 mol × (− 890 kJ/mol)


= 44.5kJ

C.

132 Energy Changes


12. A. X represents the activation energy.
B. Exothermic reaction. The energy released in products formation is greater than the energy used by
the reactants.
mass given
C. n (glucose) =
molar mass
3.6
=
180
= 0.02 mol

H = 0.02 mol × (− 224 kJ/mol)


= 4.48kJ

13. A. Energy is released to the surrounding.


B. H = − 494kJ/mol
C. Exothermic reaction. The energy released in products formation is greater than the energy used by
the reactants.
D. X is the activation energy. It is the energy needed to start a chemical reaction.
E. ΔH = 494kJ

Energy Changes 133


14 REDOX REACTIONS

Introduction
Redox reactions are types of reactions that involve reduction and oxidation. This unit covers these types of
reactions. You will learn what oxidation and reduction are, how to write an ionic equation and many more.

Specific outcomes
By the end of this unit, you will be able to:
❖ Define the following terms:
• Redox reaction
• Oxidation
• Reduction
• Oxidising agent
• Reducing agent
❖ Describe redox reactions in terms of:
• Oxygen
• Hydrogen
• Electrons
• Oxidation number
❖ Write ionic equations
❖ Describe how to test for oxidising agents and reducing agents

134 Redox Reactions


REDOX REACTIONS
14.1 REDOX REACTIONS
• Redox stands for reduction oxidation.
• Redox reaction, also called oxidation reduction reaction, is a chemical reaction in which one
substance is reduced and another is oxidised simultaneously.
• Oxidation and reduction of substances occur simultaneously. As one substance is reduced the other
is oxidised.
• An oxidising agent is a substance that causes the oxidation of the other substance. Therefore, the
substance that is reduced in a reaction is the oxidising agent.
• A reducing agent is a substance that causes the reduction of the other substance. For example, the
substance that is oxidised in a reaction is the reducing agent.

1. OXYGEN IN REDOX REACTION


• Oxidation is the gain of oxygen by the substance. A substance that has gained oxygen is
said to be oxidised.
• Reduction is the loss of oxygen by the substance. A substance that has lost oxygen is said to
be reduced.
• The reaction below shows a redox reaction involving oxygen.

• Lead (Pb) is oxidised. As a result, it gains oxygen and forms lead(II) oxide (PbO).
• Silver(I) oxide (Ag2O) is reduced. As a result, it loses oxygen and forms silver metal (Ag).
• Silver(I) oxide is the oxidising agent. It causes the oxidation of the lead metal.
• Lead is the reducing agent. It causes the reduction of silver(I) oxide.

❖ EXAMPLES
1. Identify the substance oxidised and reduced, the oxidising agent and the reducing agent
in the following redox reactions.
A. CuO(s) + H2(g) → Cu(s) + H2O(l)
B. Fe2O3(s) + 3CO(g) → 2Fe(s) + 3CO2(s)

❖ SOLUTIONS
1. A. Hydrogen gas (H2) is oxidised to water (H2O).
Copper (II) oxide (CuO) is reduced to copper (Cu).
Copper (II) oxide is the oxidising agent.
Hydrogen gas is the reducing agent.

B. Carbon monoxide (CO) is oxidised to carbon dioxide (CO2).


Iron(III) oxide (Fe2O3) is reduced to iron (Fe).
Iron(III) oxide is the oxidising agent.
Carbon monoxide is the reducing agent.

Redox Reactions 135


2. HYDROGEN IN REDOX REACTION
• Oxidation is the loss of hydrogen by the substance. A substance that has lost hydrogen is said to be
oxidised.
• Reduction is the gain of hydrogen by the substance. A substance that has gained hydrogen is said to
be reduced.
• The reaction below shows a redox reaction involving hydrogen.

• Hydrogen sulphide (H2S) is oxidised. As a result, it loses hydrogen and becomes sulphur (S).
• Chlorine is reduced. As a result, it gains hydrogen and forms hydrochloric acid (HCl).
• Hydrogen sulphide is the reducing agent. It causes the reduction of chlorine.
• Chlorine is the oxidising agent. It causes the oxidation of hydrogen sulphide.

❖ EXAMPLES
1. Identify the substance oxidised and reduced, the oxidising agent and the reducing agent in the
following redox reactions.
A. 4HCl(s) + O2(g) → 2Cl2(g) + 2H2O(l)
B. CH4(g) + 2O2(g) → CO2(g) + 2H2O(g)

❖ SOLUTIONS
1. A. Hydrochloric acid (HCl) is oxidised to chlorine.
Oxygen (O2) is reduced to water (H2O).
Hydrochloric acid is the reducing agent.
Oxygen is the oxidising agent.

B. Methane (CH4) is oxidised to carbon dioxide (CO2).


Oxygen (O2) is reduced to water (H2O).
Methane is the reducing agent.
Oxygen is the oxidising agent.

14.2 OXIDATION NUMBER


• Oxidation number, also called oxidation state, is the number of electrons an atom gains or losses in
a chemical combination.

❖ RULES FOR ASSIGNING OXIDATION NUMBERS


1. Oxidation number of free elements is zero, for example:
• Na = 0
• Cu = 0
• Zn = 0

2. Oxidation number of uncharged elements is zero, for example:


• N2 = 0
• Cl2 = 0
• O2 = 0

136 Redox Reactions


3. Oxidation number of ions is the charge of an ion, for example:
• Cl− = −1
• Na+ = +1
• Ca2+ = +2
• O2− = −2
• Fe3+ = +3

4. The sum of oxidation numbers in a neutral compound is zero, for example:


• NaCl = Na+Cl− = +1 + (−1) = 0
• AgO = Ag2+O2− = +2 + (−2) = 0
• MgCl2 = Mg2+Cl2− = +2 + 2(−1) = 0

5. The sum of oxidation numbers in polyatomic ions is equal to the charge of the compound, for example:
• SO42− = S6+O42− = +6 + 4(−2) = −2
• NO3− = N5+O3− = +5 + 3(−2) = −1
• OH− = O2−H+ = −2 + (+1) = −1

6. The oxidation number of some elements is fixed.


• Group I = +1
• Group II = +2
• Group VII is usually −1
• Hydrogen is usually +1
• Oxygen is usually −2

❖ EXAMPLES
1. Find the oxidation number of the following.
A. Na
B. F2
C. Cu2+
D. Ag in AgO
E. Na in Na2O
F. C in CH2Cl2
G. Cr in Cr2O72−
H. Mn in KMnO4

❖ SOLUTIONS
1. A. Na = 0
B. F2 = 0
C. Cu2+ = +2
D. AgO; Ag + (−2) = 0
Ag = +2

E. Na2O; 2(Na) + (−2) = 0


2(Na) = + 2
Na = +1

F. CH2Cl2; C + 2(+1) + 2(−1) = 0


C+2–2=0
C=0

Redox Reactions 137


G. Cr2O72−; 2(Cr) + 7(−2) = −2
2(Cr) − 14 = − 2
2(Cr) = 12
Cr = + 6

H. KMnO4; +1 + Mn + 4(−2) = 0
Mn + 1 – 8 = 0
Mn = +7

3. ELECTRONS IN REDOX
• Oxidation is the loss of electrons. A substance that loses electrons is oxidised.
• Reduction is the gain of electrons. A substance that gains electrons is reduced.
• The reaction below shows a redox reaction involving the gain and loss of electrons.

• Chlorine (Cl2) is reduced. As a result, it gains one electron and become Cl−.
• Hydrogen (H2) is oxidised. As a result, it loses one electron and become H+.
• Chlorine is the oxidising agent.
• Hydrogen is the reducing agent.

❖ EXAMPLES
1. Identify the substance oxidised and reduced, the oxidising agent and the reducing agent in the
following redox reactions.
A. 2Na(s) + Cl2(g) → 2NaCl(aq)
B. 4Fe(s) + 3O2(g) → 2Fe2O3(s)

❖ SOLUTIONS
1. A. Sodium (Na) is oxidised to Na+.
Chlorine (Cl2) is reduced to Cl−.
Sodium is the reducing agent.
Chlorine is the oxidising agent.

B. Iron (Fe) is oxidised to Fe3+.


Oxygen is reduced to O2−.
Iron is the reducing agent.
Oxygen is the oxidising agent.

138 Redox Reactions


4. OXIDATION NUMBER IN REDOX
• Oxidation is the increase in oxidation number. Therefore, a substance whose oxidation number
increases is oxidised.
• Reduction is the decrease in oxidation number. Therefore, a substance whose oxidation number
decreases is reduced.
• The reaction below shows a redox reaction involving an increase and decrease in oxidation number.

• Methane (CH4) is oxidised. As a result, the carbon oxidation number increases from −4 to +4.
• Oxygen (O2) is reduced. Therefore, the oxygen oxidation number decreases from 0 to −2.
• Methane is the reducing agent.
• Oxygen is the oxidising agent.

❖ EXAMPLES
1. Identify the substance oxidised and reduced, the oxidising agent and the reducing agent in the
following redox reactions.
A. Fe2O3(s) + 3CO(g) → 2Fe(g) + 3CO2(g)
B. H2(g) + I(g) → 2HI(g)

❖ SOLUTIONS
1. A. Iron(III) oxide is reduced. As a result, the iron oxidation number decreases from +3 to 0.
Carbon monoxide (CO) is oxidised. As a result, the carbon oxidation number increases
from +2 to +4.
Iron(III) oxide is the oxidising agent.
Carbon monoxide is the reducing agent.

B. Hydrogen (H2) is oxidised. As a result, the hydrogen oxidation number increases from 0
to +1.
Iodine (I2) is reduced as. As a result, the iodine oxidation number decreases from 0 to
−1.

❖ REDOX REACTIONS SUMMARY


Oxidation Reduction
Gain of oxygen Loss of oxygen
Loss of hydrogen Gain of hydrogen
Loss of electrons Gain of electrons
Increase in oxidation number Decrease in oxidation number

Redox Reactions 139


14.3 IONIC EQUATION
• The ionic equation is a chemical equation that shows participating substances in a dissociated form.
• It is written similar to the molecular formula.
• The difference between the molecular formula and the ionic formula is that the molecular formula
shows substances involved in a chemical reaction as though they exist in molecules. In contrast, the
ionic equation shows substances in an aqueous solution in a dissociated form.
• The net ionic equation shows only substances that are directly involved in a chemical reaction.
Substances that do not take part in a chemical reaction called spectator ions are removed.
Molecular equation: NaCl(aq) + AgNO3(aq) → AgCl(s) + NaNO3(aq)
Ionic equation: Na+(aq) + Cl−(aq) + Ag+(aq) + NO−3 (aq) → AgCl(s) + Na+ + NO−3(aq)
Net ionic equation: Ag+(aq) + Cl−(aq)+ → AgCl(s)

Na+ and NO−3 are spectator ions, they


remain unchanged in reactants and
products therefore, they are removed

❖ HOW TO WRITE AN IONIC EQUATION


1. Write and balance the chemical equation.
Zn(s) + 2HCl(aq) → ZnCl2(aq) + H2(g)
2. Split ionic compounds that are in an aqueous solution.
Zn(s) + 2H+(aq) + 2Cl−(aq) → Zn2+(aq) + 2Cl− (aq) + H2(g)
3. Cancel out spectator ions. These are ions that remain unchanged in reactants and products. In
step 2, 2Cl− are spectator ions.
Zn(s) + 2H+(aq) → Zn2+(aq) + H2(g)

❖ EXAMPLES
1. Write the ionic equation and net ionic equation of the following molecular formula.
A. HCl(aq) + NaOH(aq) → NaCl(aq) + H2O(l)
B. MgCl2(aq) + 2AgNO3(aq) → 2AgCl(aq) + Mg(NO3)2(aq)

❖ SOLUTIONS
1. A. HCl(aq) + NaOH(aq) → NaCl(aq) + H2O(l)

Ionic equation: H+(aq) + Cl−(aq) + Na+(aq) + OH−(aq) → Na+(aq) + Cl−(aq) + H2O(l)

Net ionic equation: H+(aq) + OH−(aq) → H2O(l)

B. MgCl2(aq) + 2AgNO3(aq) → 2AgCl(aq) + Mg(NO3)2(aq)

Ionic equation: Mg2+(aq) + 2Cl−(aq) + 2Ag+(aq) + NO3−(aq) → 2Ag+(aq) + 2Cl−(aq) + Mg2+(aq) +


2NO3−(aq)

Net ionic equation: MgCl2(aq) + 2AgNO3(aq) → 2AgCl(aq) + Mg(NO3)2(aq)

140 Redox Reactions


14.4 TESTING FOR REDUCING AND OXIDISING AGENTS
❖ TEST FOR REDUCING AGENTS
1. ACIDIFIED POTASSIUM DICHROMATE (VI) SOLUTION
• Acidified potassium dichromate (VI) (K2Cr2O7) is an oxidising agent.
• Add acidified potassium dichromate (VI) solution to the sample.
• If the reducing agent is present the colour changes from orange to green.

2. ACIDIFIED POTASSIUM MANGANATE (VII) SOLUTION


• Acidified potassium manganate (VII) (KMnO4) is an oxidising agent.
• Add acidified potassium manganate (VII) solution to the sample.
• If the reducing agent is present the colour changes from purple to colourless.

❖ TEST FOR OXIDISING AGENT


1. POTASSIUM IODIDE SOLUTION
• Potassium iodide (KI) is a reducing agent.
• Add KI solution to the sample.
• If the oxidising agent is present the colour changes from colourless to brown.

2. IRON (III) SULPHATE


• Iron(III) sulphate (FeSO4) is a reducing agent.
• Add Iron(III) sulphate to the sample.
• If the oxidising agent is present the colour changes from green to brown.

Redox Reactions 141


REVIEW QUESTIONS
1. The type of reaction that involves one species oxidised and another reduced is called
A. Addition reaction
B. Displacement reaction
C. Substitution reaction
D. Redox reaction
2. A substance that has the potential to cause another substance to be oxidised is known as
A. Oxidising agent
B. Reducing agent
C. Donor
D. Recipient
3. A substance that has the potential to cause another substance to be reduced is known as
A. Oxidising agent
B. Reducing agent
C. Donor
D. Recipient
4. Involving oxygen, oxidation is
A. Gain of oxygen
B. Loss of oxygen
C. Share of oxygen
D. Elimination of oxygen
5. Involving oxygen, reduction is
A. Gain of oxygen
B. Loss of oxygen
C. Share of oxygen
D. Elimination of oxygen
6. In this reaction; ZnO(s) + C(s) → Zn(g) + CO(g), ZnO is
A. Gains oxygen
B. Oxidised
C. Reduced
D. Reducing agent
7. In this redox reaction; ZnO(s) + C(s) → Zn(g) + CO(g), C is
A. Loses oxygen
B. Oxidised
C. Reduced
D. Oxidising agent
8. Involving hydrogen, oxidation is
A. Gain of hydrogen
B. Loss of hydrogen
C. Share of hydrogen
D. Elimination of hydrogen
9. Involving hydrogen, reduction is
A. Gain of hydrogen
B. Loss of hydrogen
C. Share of hydrogen
D. Elimination of hydrogen
10. In this redox reaction, 4HCl(g) + O2(g) → 2Cl2(g) + 2H2O(g), which substance is the oxidising agent?
A. Cl2
B. H2O
C. HCl
D. O2

142 Redox Reactions


11. In this redox reaction, 4HCl(g) + O2(g) → 2Cl2(g) + 2H2O(g), which substance is the reducing agent?
A. Cl2
B. H2O
C. HCl
D. O2
12. Involving electrons, oxidation is
A. Gain of electrons
B. Loss of electrons
C. Share of electrons
D. Elimination of electrons
13. Involving electrons, reduction is
A. Gain of electrons
B. Loss of electrons
C. Share of electrons
D. Elimination of electrons
14. In this redox reaction, 2K(s) + I2(s) → 2KI(s), which gain electrons
A. K
B. I2
C. KI
D. All the above
15. In this redox reaction, 2K(s) + I2(s) → 2KI(s), which loss electrons
A. K
B. I2
C. KI
D. All the above
16. Involving oxidation number, oxidation is
A. Increase in oxidation number
B. Decrease in oxidation number
C. Oxidation number remains uncharged
D. None of the above
17. Involving oxidation number, reduction is
A. Increase in oxidation number
B. Decrease in oxidation number
C. Oxidation number remains uncharged
D. None of the above
18. In this redox reaction, 4Fe(s) + 3O2(g) → 2Fe2O3(s), which substance has the oxidation number increased?
A. Fe2O3
B. Fe
C. O2
D. All the above
19. In this redox reaction, 4Fe(s) + 3O2(g) → 2Fe2O3(s), which substance has the oxidation number decreased?
A. Fe2O3
B. Fe
C. O2
D. All the above
20. What is the oxidation number of Cr in K2Cr2O7?
A. +5
B. + 2
C. + 7
D. + 6

Redox Reactions 143


21. What is the oxidation number of Mn in MnO4− ?
A. +5
B. + 2
C. + 7
D. + 6
22. The chemical equation below shows a redox reaction used to prepare chlorine gas in the laboratory.
2KMnO4(aq) + 16HCl(aq) → 2MnCl2(aq) + 2KCl(aq) + 5Cl2(g) + 8H2O(l)
A. What is meant by redox reaction?
B. Calculate the oxidation number of
i. Mn in KMnO4
ii. Mn in MnCl2
iii. Cl in HCl
C. Is a change from KMnO4 to MnCl2 oxidation or reduction? Give a reason for your answer.
D. Is a change from HCl to Cl2 oxidation or reduction? Give a reason for your answer.

23. When chlorine gas is bubbled into iron(II) chloride solution the following reaction takes place.
FeCl2(aq) + Cl2(g) → FeCl3(aq)
A. Write a balanced chemical equation for the reaction above.
B. What colour change is observed in this reaction?
C. Which substance is oxidised? Give a reason for your answer.
D. Which substance is reduced? Give a reason for your answer.
E. Write an ionic equation for the reaction above.

144 Redox Reactions


SOLUTIONS
1. D
2. A
3. B
4. A
5. B
6. C
7. B
8. B
9. A
10. D
11. C
12. B
13. A
14. B
15. A
16. A
17. B
18. B
19. C
20. D
21. C

22. A. Redox reaction is a chemical reaction in which one substance is reduced and another is oxidised
Simultaneously.
B. i. KMnO4; +1 + Mn + 4(−2) = 0
Mn + 1 − 8 = 0
Mn − 7 = 0
Mn = +7

ii. MnCl2; Mn + 2(−1) = 0


Mn – 2 = 0
Mn = +2

iii. HCl; +1 + Cl = 0
Cl = −1

C. Reduction, because manganese (Mn) oxidation number decreases from +7 in KMnO4 to +2 in MnCl2.
D. Oxidation, because chlorine (Cl) oxidation number increases from −1 in HCl to 0 in Cl2.

23. A. 2FeCl2(aq) + Cl2(g) → 2FeCl3(aq)


B. Pale green solution changes to pale orange.
C. Iron(II) chloride (FeCl2) is oxidised, because Fe oxidation number increases from +2 in FeCl2
to +3 in FeCl3.
D. Chlorine (Cl2) is reduced, because its oxidation number decreases from 0 in chlorine gas (Cl2)
to −1 in iron(III) chloride ( FeCl3).

Redox Reactions 145


15 ELECTROLYSIS

Introduction
What happens when an electric current is passed through the ionic compound in an aqueous solution? The
ionic compound is split into smaller components. This process is called electrolysis. Electrolysis has several
real life applications. Most metal objects we use are coated with another metal to prevent rusting or to make
them attractive. This is done by electrolysis. The process is also used to extract and purify some metals.

Specific outcomes
This unit covers electrolysis. By the end of this unit, you will be able to:
❖ Define the following terms:
• Electrolysis
• Electrolyte
• Anode
• Cathode
❖ Describe the preferential discharge theory
❖ Describe the electrolysis of:
• Lead (II) bromide
• Acidified water
• Sodium chloride
• Copper (II) sulphate using graphite electrode and copper electrode
❖ Describe the extraction of aluminium
❖ Describe electroplating and galvanising
❖ Work out electrolysis calculations

146 Electrolysis
ELECTROLYSIS
15.1 ELECTROLYSIS
• Electrolysis is the decomposition of an ionic compound into its components using an electric
current.

❖ INDUSTRIAL USES OF ELECTROLYSIS


1. Manufacture of sodium hydroxide, chlorine and hydrogen gas.
2. Purification of copper.
3. Electroplating. A process of coating an object with a layer of metal for protection or making it
attractive.
4. Extraction of aluminium.

❖ COMPONENTS OF ELECTROLYSIS
1. ELECTROLYTE
• An electrolyte is an ionic compound in a molten or aqueous solution to be decomposed.
• An electrolyte should conduct electricity.

2. ELECTRODE
• An electrode is an electric conductor that carries an electric current in and out of the
electrolyte.
• Commonly used electrodes are platinum and graphite.

3. ANODE
• An anode is a positively charged electrode.
• It attracts anions, the negatively charged ions.
• On anode, negatively charged ions lose electrons and become neutral elements.
• Therefore, anions are oxidised at the anode. The oxidation process takes place here.
• For example, two chloride ions (Cl−) lose two electrons, form a bond and become chlorine
gas (Cl2).
2Cl−(aq) → Cl2(g) + 2e−

4. CATHODE
• A cathode is a negatively charged electrode.
• It attracts cations, positively charged ions.
• On cathode, positively charged ions gain electrons and become neutral atoms.
• Therefore, cations are reduced at the cathode. Reduction takes place here.
• For example, copper ion (Cu2+) gains two electrons and become a copper atom.
Cu2+(aq) + 2e− → Cu(s)

Electrolysis 147
15.2 DISCHARGE
• Discharge is the formation of neutral atoms from ions.
• Negative ions lose electrons and become neutral atoms.
• Positive ions gain electrons and become neutral atoms.

❖ PREFERENTIAL DISCHARGE THEORY


• Preferential discharge theory states that if more than one type of cation, anion or both are
present in the electrolyte, discharge depends on the position of ions in the electrochemical
series, the concentration of ions in the electrolyte and the nature of the electrode.
1. POSITION OF IONS IN ELECTROCHEMICAL SERIES
Electrochemical Cation Anion
series
Difficult K+ SO42−
Na+ NO3−
Ca2+
Mg2+ Cl−
Al2+ Br−
Zn2+ I−
Fe2+
Sn2+
Pb2+
H+ OH−
Cu2+
Easy Ag+

• Ions that are lower in the electrochemical series preferentially get discharged than those
above them.
• For example, if an electrolyte has H+ and Ag+, Ag+ get discharged instead of H+.
However, if it contains H+ and Ca2+, H+ gets discharged instead of Ca2+.

2. CONCENTRATION OF IONS IN ELECTROLYTE


• The ion with a higher concentration is more likely to be discharged.
• In high concentrated solution, iodide ion (I−), bromide ion (Br−) and chloride ion (Cl−)
are preferentially discharged over hydroxide ion (OH−) although it is harder to
discharge them compared to hydroxide ion.

3. NATURE OF ELECTRODE
• If the electrolysis contains inert electrodes such as graphite or platinum, these
electrodes do not take part in the chemical reaction of the electrolysis.
• If it contains active electrodes such as copper or silver, they take part in the chemical
reaction and are discharged.

148 Electrolysis
15.3 ELECTROLYSIS OF LEAD(II) BROMIDE

• Lead(II) bromide is heated until it becomes molten. Lead(II) bromide has to be in molten form so
that is it able to conduct electricity.
• Electricity is passed through the molten lead(II) bromide.
• Electrolyte used: molten lead(II) bromide.
• Electrode used: graphite.
• Ions present: Pb2+ and Br−.

❖ REACTION AT CATHODE
• Cathode attracts lead ions (Pb2+).
• Pb2+ gains two electrons from the cathode and become Pb atom.
• The half equation at the cathode.
Pb2+(aq) + 2e− → Pb(l)

❖ REACTION AT ANODE
• Anode attracts bromide ions (Br−).
• Br− loses an electron and becomes a bromine atom.
• Two bromine atoms bond and become bromine gas (Br2).
• Bromine gas bubbles out.
• Test for bromine: it decolourises when shaken with alkene solution.
• The half equation at the anode.
2Br−(aq) → Br2(g) + 2e−

❖ OVERALL EQUATION
Pb2+(aq) + 2Br−(aq) + 2e− → Pb(l) + Br2(g) + 2e− add half equations at cathode and anode
Pb2+(aq) + 2Br−(aq) → Pb(l) + Br2(g) cancel 2e− on both sides

Electrolysis 149
15.4 ELECTROLYSIS OF ACIDIFIED WATER

• This is the electrolysis of water into hydrogen gas and oxygen gas.
• The electrolyte used: diluted sulphuric acid.
• Electrode used: graphite or platinum.
• Ions present: H+, OH− and SO42−.

❖ REACTION AT CATHODE
• Cathode attracts hydrogen ions (H+).
• H+ gain an electron from the cathode and become hydrogen gas.
• Two hydrogen atoms formed bonds and become hydrogen gas (H2).
• Hydrogen gas bubbles out.
• Test for hydrogen gas: produce a pop sound when lit.
• The half equation at the cathode.
2H+(aq) + 2e− → H2(g)

❖ REACTION AT ANODE
• Anode attracts hydrogen ions (OH−) and sulphate ions (SO42−).
• OH− is preferentially discharged over SO42−.
• OH− loses an electron to the anode and become oxygen gas (O2) and water (H2O).
• Test for oxygen: glowing wooden splint relights.
• The half equation at the anode.
4OH−(aq) → 2H2O(l) + O2(g) + 4e−

❖ OVERALL EQUATION
4H+(aq) + 4OH−(aq) + 4e− → 2H2(g) + O2(g) + 2H2O(l) + 4e− add both half equations
+ −
4H (aq) + 4OH (aq) → 2H2(g) + O2(g) + 2H2O(l) cancel 4e− on both side

150 Electrolysis
15.5 ELECTROLYSIS OF SODIUM CHLORIDE (BRINE)

• This is the electrolysis of sodium chloride to produce hydrogen gas and chlorine gas.
• Electrolyte used: sodium chloride (brine) solution.
• Electrode used: graphite.
• Ions present: Na+, H+, OH− and Cl−.

❖ REACTION AT CATHODE
• Cathode attracts sodium ions (Na+) and hydrogen ions (H+).
• H+ is preferentially discharged over Na+.
• H+ gains an electron from the cathode and become a hydrogen atom.
• Two hydrogen atoms formed bonds and become hydrogen gas (H2).
• Hydrogen gas bubbles out.
• Test for hydrogen gas: produce a pop sound when lit.
• The half equation at the cathode.
2H+(aq) + 2e− → H2(g)

❖ REACTION AT ANODE
• Anode attracts hydrogen ions (OH−) and chloride ions (Cl−).
• Cl− is preferentially discharged over OH−.
• Cl− loses an electron and become chlorine atom.
• Two chlorine atoms bond and form chlorine gas.
• Chlorine gas bubbles out.
• Test for chlorine: a pale green gas with a pungent odour, it turns a damped blue litmus paper red
then bleached to white.
• The half equation at the anode.
2Cl−(aq) → Cl2(g) + 2e−

❖ ELECTROLYTE RESIDUE
• Na+ and OH− remain in the solution as electrolyte residue.
• These ions chemically bond and form sodium hydroxide (NaOH).

❖ OVERALL EQUATION
• 2NaCl(aq) + 2H2O(l) → H2(g) + Cl2(g) + 2NaOH(aq)

Electrolysis 151
15.6 ELECTROLYSIS OF COPPER(II) SULPHATE SOLUTION
• This is the electrolysis of copper(II) sulphate to form copper metal and oxygen gas.
• Electrolyte used: copper(II) sulphate (CuSO4).
• The electrode used: electrolysis of copper (II) sulphate is performed using graphite or copper
electrodes.
1. USING GRAPHITE ELECTRODE

• Electrolysis of copper(II) sulphate using graphite electrode produces copper metal and
hydrogen gas.
• Ions present: Cu2+, H+ (from H2O), SO42− and OH− (from H2O).

❖ REACTION AT CATHODE
• Cathode attracts copper ions (Cu2+) and hydrogen ions (H+).
• Cu2+ is preferentially discharged over H+.
• Cu2+ loses two electrons to the cathode and become copper metal. As a result, it forms a
deposit on the surface cathode.
• Half reaction at the cathode.
Cu2+(aq) + 2e− → Cu(s)

❖ REACTION AT ANODE
• Anode attracts sulphate ions (SO42−) and hydroxide ions (OH−).
• OH− is preferentially discharged over SO42−.
• OH− loses an electron to the anode and become oxygen gas (O2) and water (H2O).
• The half equation at the anode.
4OH−(aq) → 2H2O(l) + O2(g) + 4e−

❖ OVERALL REACTION
2Cu2+(aq) + 4OH−(aq) + 4e− → 2Cu(s) + 2H2O(l) + O2(g) + 4e− add both half equations
2Cu2+(aq) + 4OH−(aq) → 2Cu(s) + 2H2O(l) + O2(g) cancel 4e− on both side

152 Electrolysis
2. USING COPPER ELECTRODE

• This is the electrolysis of copper(II) sulphate using copper electrode produces copper metal and
copper ions.
• Copper metal is deposited at the cathode while the copper electrode at the anode is dissolved.
• The mass of copper deposited at the cathode is equal to the mass of copper dissolved at the anode.
• This process is used in the purification of copper.
• Ions present: Cu2+, H+ (from H2O), SO42− and OH− (from H2O).

❖ REACTION AT CATHODE
• Cathode attracts copper ions (Cu2+) and hydrogen ions (H+).
• Cu2+ is preferentially discharged over H+.
• Cu2+ loses two electrons to the cathode and become copper metal. As a result, it forms a deposit
on the surface cathode.
• The half reaction at the cathode.
Cu2+(aq) + 2e− → Cu(s)

❖ REACTION AT ANODE
• Anode attracts sulphate ions (SO42−) and hydroxide ions (OH−).
• However, copper anode discharges itself producing copper ion (Cu2+).
Cu(s) → Cu2+(aq) + 2e−

❖ OVERALL REACTION
Cu2+(aq) + Cu(s) → Cu(s) + Cu2+(aq)

Electrolysis 153
15.7 EXTRACTION OF ALUMINIUM
• Aluminium is mined from bauxite. Bauxite is a mixture of aluminium oxide (Al2O3) and other
minerals.
• Electrolysis is used to obtain aluminium from aluminium oxide.
• Aluminium oxide is heated until it becomes molten.
• Cryolite (Na3AlF3) is added to reduce the melting point of aluminium oxide.
• Electrode used: graphite.
• Ions present: Al3+ and O2−.

❖ REACTION AT CATHODE
• Cathode attracts aluminium ions (Al3+).
• Al3+ gains three electrons and become aluminium metal.
• The half reaction at the cathode.
Al3+(aq) + 3e− → Al(s)

❖ REACTION AT ANODE
• Anode attracts oxide ions (O2−).
• O2− loses two electrons and become an oxygen atom.
• Two oxygen atoms bond and form oxygen gas.
2O2−(aq) → O2(g) + 4e−

❖ OVERALL REACTION
4Al3+(aq) + 6O2−(aq) + 12e− → 4Al(s) + 3O2(g) + 12e−
4Al3+(aq) + 6O2−(aq) → 4Al(s) + 3O2(g)

154 Electrolysis
15.8 ELECTROPLATING
• Electroplating is the process of coating an object with a layer of metal by electrolysis.
• Electroplating protects objects and makes them attractive.
• During electroplating;
i. cathode must contain an object to be electroplated.
ii. anode must contain an electroplating metal.
iii. electrolyte must contain electroplating ions, for example, CuSO4 for copper electroplating,
AgNO3 for silver electroplating, ZnSO4 for zinc electroplating.

❖ GALVANISING
• Galvanising is the process of electroplating iron or steel with zinc metal.
• Galvanised iron or steel is protected from rust.
• During galvanising, the cathode must contain steel to be electroplated. In addition, the anode
must contain zinc metal and zinc sulphate electrolyte is used.

➢ AT CATHODE
• Zinc ion (Zn2+) gains two electrons and form zinc metal.
• Zinc metal formed is coated on steel metal.

➢ AT ANODE
• Zinc metal loses two electrons and forms Zn2+ ion.
• Zn2+ ions are released into the solution.
• The anode supplies more Zn2+ ions to the solution.

Electrolysis 155
15.9 ELECTROLYSIS CALCULATION
1. CHARGE TRANSFER
• When electric current passes through the electrolyte the negatively charged electrode (cathode)
attracts positively charged ions (cations) while the positively charged electrode (anode) attracts
negatively charged ions (anions).
• At the cathode, cations gain electrons and become neutral atoms. Therefore, cations are reduced
to neutral atoms.
• At the anode, anions lose electrons and become neutral atoms. Therefore, anions are oxidised to
neutral atoms.
• The amount of charge transfer depends on the amount of current flowing and the time taken.
• Formula:

Q = It where
Q = charge transfer (in Coulomb, C)
I = current flow (in Ampere, A)
t = time (in seconds, s)

❖ EXAMPLES
1. Calculate the amount of charge transferred when 8A current is used for 30 minutes during
electrolysis.
2. A current of 3A was passed through a solution of sulphuric acid for 10 minutes. How many
coulombs were passed through the solution?

❖ SOLUTIONS
1. Q = It Q=?
Q = 8 × 1800 I=8A
Q = 14400 C t = 30 × 60s
= 1800s
2. Q = It Q=?
Q = 3 × 600 I=3A
Q = 1800 C t = 10 × 60s
= 600s

2. MOLE CALCULATION
• During electrolysis, the amount of a substance in moles produced depends on the amounts of
charges transferred.
• According to Faraday, 96500 coulombs is equivalent to one mole. This value is Faraday’s constant.
• Therefore, knowing the amount of charges transferred during the electrolysis the number of moles
of a substance can be calculated.
• The formula for calculating the mole of a substance produced during electrolysis is as given:

Q where
n= or
F n = number of moles (in mol)
Q = charge transferred ( in C)
It
n= F = Faraday’s constant = 96500 C
F I = current flow (in Ampere, A)
t = time (in seconds, s)

156 Electrolysis
❖ EXAMPLES
1. How many moles of copper is produced if a current 1.5A is passed for 1 hour?
2. How many moles of aluminium is produced from an aqueous solution of aluminium chloride using
20A current for 10 min?

❖ SOLUTIONS
It n=?
1. n =
F I = 1.5A
1.5 × 3600 t = 1h × 60min × 60s
n=
96500 = 3600s
n = 0.06 mol F = 96500 C

It n=?
2. n =
F I = 20A
20 × 600 t = 10min × 60s
n=
96500 = 600s
n = 0.006 mol F = 96500 C

3. MASS CALCULATION
• When the number of moles is known the mass of a substance produced during electrolysis can be
calculated.
• The formula for calculating the mass of a substance produced during electrolysis is as given:

m = n × Mr where
m = mass produced (in grams)
n = moles of substance (in mol)
Mr = molar mass of substance (in g/mol)

❖ EXAMPLES
1. How many grams of copper can be plated out from an aqueous copper (II) nitrate solution using
a 3A current for 40 min?
2. How much silver can be plated out with a 12A current running for 3.5 hours?

❖ SOLUTIONS
It n=?
1. n =
F I = 3A
3 × 2400 t = 40min × 60s
n=
96500 = 2400s
n = 0.075 mol F = 96500 C

mass = mole × molar mass mass = ?


mass = 0.075 × 63.5 mole = 0.075 mol
mass = 4.76g Mr (Cu) = 63.5 g/mol

Electrolysis 157
It n=?
2. n=
F I = 12A
12 × 12600 t = 3.5hr × 60min × 60s
n=
96500 = 12600s
n = 1.57 mol F = 96500 C

mass = mole × molar mass mass = ?


mass = 1.57 × 108 mole = 1.57 mol
mass = 169.56g Mr (Cu) = 108 g/mol

4. VOLUME CALCULATION
• The volume of gas produced during electrolysis at room temperature and pressure can be calculated
from the known number of moles.
• Note; 1 mole of any substance at room temperature and pressure occupies 24dm3. This value is
known as molar volume.
• The formula for calculating the volume of a gas produced during electrolysis is as given:

Volume of gas = mole of gas × 24dm3

❖ EXAMPLES
1. 0.2 moles of chlorine gas were produced during the electrolysis of molten sodium chloride.
Calculate the volume of chlorine at room temperature and pressure
2. What volume of oxygen is formed by passing 10A through acidified water for 30min at room
temperature and pressure?

❖ SOLUTIONS
1. Volume of gas = mole of gas × 24 dm3 Volume of Cl2 = ?
Volume (Cl2) = 0.2 × 24
Mole of Cl2 = 0.2 mol
Volume (Cl2) = 4.8 dm3
Molar volume = 24 dm3

It n=?
2. n =
F I = 10A
10 × 1800 t = 30min × 60s
n=
96500 = 1800s
n = 0.026 mol F = 96500 C

Volume of gas = mole of gas × 24dm3 Volume of O2 = ?


Volume (O2) = 0.026 × 24 Mole of O2 = 0.026 mol
Volume (O2) = 0.62 dm3 Molar volume = 24 dm3

158 Electrolysis
REVIEW QUESTIONS
1. Which of the following is likely not the application of electrolysis?
A. Purification of sodium
B. Purification of copper
C. Manufacture of hydrogen
D. Electroplating
2. Ion compound in an aqueous solution that is decomposed during electrolysis is known as
A. Cathode
B. Electrode
C. Electrolyte
D. Anode
3. An electric conductor that brings and take away electric current from the electrolyte is known as
A. Cathode
B. Electrode
C. Anode
D. Conductor
4. The positively charged electrode is called
A. Cathode
B. Electrode
C. electrolyte
D. anode
5. The negatively charged electrode is called
A. Cathode
B. Conductor
C. Electrolyte
D. Anode
6. The positively charged ion is known as
A. Cation
B. Cathode
C. Anion
D. Anode
7. The negatively charged ion is known as
A. Cation
B. Cathode
C. Anion
D. Anode
8. What ions are attracted to the anode?
A. Positively charged ions
B. Negatively charged ions
C. Neutral atoms
D. All the above
9. What charge is attracted to the cathode
A. Positively charged ions
B. Negatively charged ions
C. Neutral atoms
D. All the above
10. At the anode, negatively charged ions are
A. Oxidised
B. Reduced
C. Oxidised and reduced
D. None of the above

Electrolysis 159
11. At the cathode, positively charged ions
A. Oxidised
B. Reduced
C. Oxidised and reduced
D. None of the above
12. Which of the following statements is true concerning lead (II) bromine electrolysis?
A. Lead ions are attracted to the anode
B. Bromine ions are attracted to the cathode
C. Lead(II) bromine electrolyte is heated to a molten state
D. All the above
13. What gases are produced during the electrolysis of acidified water?
A. Hydrogen only
B. Oxygen only
C. Sulphur only
D. Hydrogen and oxygen
14. What are the products of sodium chloride electrolysis?
A. Hydrogen and oxygen
B. Oxygen, chlorine and sodium hydroxide
C. Hydrogen, chlorine and sodium hydroxide
D. Sodium hydroxide and water
15. During electrolysis of copper(II) sulphate, what happens at cathode and anode when copper electrodes are
used
A. At the cathode, copper ions are reduced to copper metal while at the anode hydroxide ions are
oxidised to water and oxygen.
B. At the cathode, copper ions are reduced to copper metal while at the anode copper metal is oxidised to
copper ions.
C. At the cathode, hydroxide ions are reduced to water and oxygen while at the anode copper metal is
oxidised to copper ions.
D. At the cathode, hydroxide ions are reduced to water and oxygen while at the anode sulphate ions are
oxidised to sulphur dioxide.
16. During aluminium extraction, what happens at cathode and anode
A. At the cathode, aluminium ions gain electrons and become aluminium metal while at node oxide ions
lose electrons and form oxygen gas.
B. At the cathode, oxide ions gain electrons and become oxygen gas while at the anode aluminium ions
lose electrons and become aluminium metal.
C. Both A and B are true.
D. None of the above.
17. During galvanising of steel with zinc, what should be placed at cathode and anode
A. At cathode zinc and anode steel
B. Zinc at cathode and anode
C. Steel at cathode and anode
D. At cathode steel and anode zinc
18. Electrolysis is used in the purification of copper
A. During the purification of copper, state the names of the following
i. electrolyte
ii. cathode
iii. anode
B. Draw a simple diagram showing how copper is purified
C. Write ionic equations for the reactions taking place at the electrode
D. A current of 10A is passed through the electrolyte for 20 minutes during the purification of copper.
Calculate the mass of pure copper obtained in 20 minutes

160 Electrolysis
19. Aluminium is extracted from bauxite by electrolysis of molten alumina dissolved in cryolite
A. State why cryolite is used during aluminium extraction.
B. Write down the formula of two ions present in the molten electrolyte.
C. Write equations for the reactions taking place at electrodes.

20. The diagram below shows the apparatus used in the silver plating of an iron article. During the
electroplating, a current of 3A was passed through the aqueous electrolyte for 40 minutes.

A. Define electroplating.
B. State the names of electrode L and electrolyte M which must be used for successful plating to occur.
C. Write equations for the reactions occurring at the electrodes.
D. What mass of silver is deposited on the iron article?
E. What changes are seen at the anode during electrolysis?
F. If the iron article was to be coated with copper instead of silver, state the materials needed to be used
for electrode M and electrolyte M.
G. State two reasons why electroplating is used.
H. State three other applications of electrolysis.

Electrolysis 161
SOLUTIONS
1. A
2. C
3. B
4. D
5. A
6. A
7. C
8. B
9. A
10. A
11. B
12. C
13. D
14. C
15. B
16. A
17. D

18. A. i. Copper(II) sulphate


ii. Copper
iii. Copper

B.

C. Cu2+(aq) + Cu(s) → Cu(s) + Cu2+(aq)

It
D. n=
F
10 × 1200
n=
96500
n = 0.124 mol

mass = mol × molar mass


mass = 0.124 × 63.5
mass = 7.87g

19. A. Cryolite reduces the melting point of alumina.


B. Al3+ and O2−
C. At cathode; Al3+(aq) + 3e− → Al(s)
At anode; 2O2−(aq) → O2(g) + 4e−

162 Electrolysis
20. A. Electroplating is a coating of an object with metal using electrolysis.
B. Electrode L: silver
Electrolyte M: silver nitrate
C. At cathode: Ag+(aq) + e− → Ag(s)
At anode: Ag(s) → Ag+(aq) + e−

It
D. n =
F
3 × 2400
n=
96500
n = 0.075 mol

mass = mol × molar mass


mass = 0.075 × 108
mass = 8.1g

E. The mass of the anode (electrode L) will be decreasing.


F. Electrode to use: copper
Electrolyte to use: copper(II) sulphate
G. i. to protect objects from corrosion.
ii. to make objects attractive.
H. i. purification of copper.
ii. extraction of aluminium.
iii. manufacture of compounds such as sodium hydroxide, hydrogen, chlorine.

Electrolysis 163
16 METALS

Introductions
About three-quarters of all the known elements are metals. In real life, they have countless uses. For example,
we use metals such as copper to make utensils and electric wires, iron for construction and gold to make
jewellery pieces. Most metals are mined from the earth’s crust as ore. How are metals extracted from their
ore? Can metals be combined? This unit covers these and many more questions.

Specific outcomes
By the end of this unit, you will be able to
❖ State the physical and chemical properties of metals
❖ State the reactivity series of metals
❖ Describe the following reactions:
• Metals with oxygen
• Metals with water
• Meals with dilute acid
• Displacement reaction
• Metal oxide with carbon
• Metal oxide with hydrogen
• Decomposition of metal carbonates
❖ Describe the extraction of the following metals:
• Iron
• Copper
• Zinc
❖ State the uses common metals
❖ Define an alloy, types and their uses
❖ State ways of preventing rusting

164 Metals
METALS
16.1 PROPERTIES OF METALS
❖ PHYSICAL PROPERTIES OF METALS
• They are strong and hard.
• They are ductile; that is, they can be stretched into a thin wire.
• They are malleable; that is, they can be bent into a without breaking
• They are shiny (lustrous) when polished.
• They are good conductors of electricity and heat.
• They have a high melting point and boiling point.
• They have high density.
• They are solid at room temperature except for mercury which is liquid.
• They are sonorous; produce sound when hit.

❖ CHEMICAL PROPERTIES OF METALS


• They form positive ions in chemical reactions.
• They react with non-metals to form ionic compounds.

16.2 REACTIVITY SERIES OF METALS

Reactive series of metals


Potassium Most reactive
Sodium
Calcium
Magnesium
Aluminium
Carbon
Zinc
Iron
Tin
Lead
Hydrogen
Copper
Mercury
Silver
Gold Least reactive

• Metals form a positive ion when they react with other substances such as oxygen, water or acid.
• Reactivity series arrange metals in order of reactivity, the tendency of a metal to react with other
elements to form compounds.
• The more reactive a metal is, the faster it reacts with other elements.
• The more reactive a metal is, the easier it loses electrons and become a positive ion.
• A very reactive metal displaces a less reactive metal.
• The more reactive a metal is, the more susceptible it is to be corroded with oxygen from air or
water. For example, sodium readily reacts with air. Therefore it is kept in oil to prevent reaction
with air. On the other hand, metals like gold are not easily corroded therefore they are found in pure
form naturally.
• The more reactive a metal is the more difficult it is to extract it from its metal ore.
• Aluminium appears to be less reactive than its position in the reactivity series because it rapidly
forms an aluminium oxide layer on the metal surface. This oxide layer tends to reduce its reactivity.

Metals 165
16.3 REACTION OF METALS WITH OXYGEN
• The process by which a metal reacts with oxygen in the air at room temperature is called corrosion.
• Metals react with oxygen to form metal oxide.
M(s) + O2(g) → MO(s)
• Very reactive metals readily burn in air.
2Na(s) + O2(g) → 2Na2O(s)
• Moderate reactive metals readily burn in the air when heated.
2Na(s) + O2(g) → 2Na2O(s)
• Less reactive metals do not burn brightly while others do not burn at all. However, they still react
with oxygen to form metal oxide when strongly heated.
4Ag(s) + O2(g) → 2Ag2O(s)

16.4 REACTION OF METALS WITH WATER


• Potassium, sodium and calcium react with cold water and form metal hydroxide and hydrogen gas.
2K(s) + 2H2O(l) → 2KOH(aq) + H2(g)
2Na(s) + 2H2O(l) → 2NaOH(aq) + H2(g)
Ca(s) + 2H2O(l) → Ca(OH)2(aq) + H2(g)
• Magnesium, zinc, aluminium and iron reacts with steam and forms metal oxide and hydrogen gas.
Mg(s) + H2O(g) → MgO(s) + H2(g)
2Al(s) + 3H2O(g) → Al2O3(s) + 3H2(g)
Zn(s) + H2O(g) → ZnO(s) + H2(g)
3Fe(s) + 4H2O(g) → Fe3O4(s) + 4H2(g) Fe3O4 is a combined form of FeO and Fe2O3
• Lead, copper, silver and gold do not react with either cold water or steam.

16.5 REACTION OF METALS WITH DILUTE ACID


• Metals react with dilute acid to form a salt and hydrogen gas.
metal + acid → salt + hydrogen
2Na(s) + 2HCl(aq) → 2NaCl(aq) + H2(g)
Mg(s) + 2HCl(aq) → MgCl2(aq) + H2(g)
• Potassium, sodium, calcium, magnesium, aluminium, zinc and iron react with dilute acid.
• Alkali metals react vigorously with acid.
• Aluminium, zinc and iron react moderately to slowly with dilute acid.
• Copper, silver and gold do not react with dilute acid.

16.6 DISPLACEMENT REACTION


• Displacement reaction is a chemical reaction in which a more reactive element displaces a less
reactive element, for example:
A + BC → AC + B
• In the illustrative example above element A displaces element B in compound BC and form
compound AC.
• A more reactive metal in the reactivity series displaces a less reactive metal.
2Na(s) + ZnO(s) → Na2O(s) + Zn(s)
2Na(s) + MgCl2(aq) → 2NaCl(g) + Mg(s)

166 Metals
16.7 REACTION OF METAL OXIDE WITH CARBON
• Carbon reacts with metal oxide and forms metal and carbon dioxide.
• Very reactive metals such as potassium sodium form strong metal oxide bonds. Therefore it is
difficult for carbon to displace a metal from metal oxide.
• Less reactive metals form a weak metal oxide bond; therefore, it is easy for carbon to displace a
metal from metal oxide.
• The lower the position in the reactivity series the easier for carbon to remove oxygen from the metal
oxide by heating.
2Ag2O(s) + C(g) → 4Ag(s) + CO2(g)
2CuO(s) + C(g) → 2Cu(s) + CO2(g)

16.8 REACTION OF METAL OXIDE WITH HYDROGEN


• Hydrogen reacts with metal oxide and forms water if the metal is less reactive
• The lower the position in the reactivity series the easier for hydrogen to remove oxygen from metal
oxide by heating
CuO(s) + H2(g) → Cu(s) + H2O(l)
Ag2O(s) + H2(g) → 2Ag(s) + H2O(g)

16.9 DECOMPOSITION OF METAL CARBONATE


• Heat can decompose a metal carbonate to metal and carbon dioxide
• Very reactive metals such as potassium and sodium form a very stable metal carbonate and are not
easily decomposed by heat
• Carbonates of moderate metals such as calcium carbonate and magnesium carbonate can be
decomposed by heat to metal oxide and carbon dioxide
CaCO3(s) → CaO(s) + CO2(g)
MgCO3(s) → MgO(s) + CO2(g)
• Carbonates of less reactive metals such as silver carbonate are completely decomposed to metal,
carbon dioxide and oxygen by heating
2Ag2CO3(s) → 4Ag(s) + 2CO2(g) + O2(g)

16.10 EXTRACTION OF METALS


• Metal elements are mined from the earth’s crust as ore.
• Metal ore contains needed elements and unwanted materials.
• Metal oxides are the commonest metal ores but can also be found as metal sulphides or metal
carbonates.
• A pure metal element can be separated from the metal ore by extraction.
• The more reactive a metal is the difficult it is to separated it from its metal ore.
• Metals that are more reactive than carbon are extracted by electrolysis, while metals that are less
reactive than carbon can be extracted by reduction with carbon.
• Potassium, sodium, calcium, magnesium and aluminium are more reactive than carbon. There
are extracted by electrolysis.
• Zinc, iron, tin, lead, copper and silver are less reactive than carbon. They are extracted by
reduction with carbon or carbon monoxide.
• Gold is very unreactive therefore it is found in a pure state in nature.

Metals 167
16.11 EXTRACTION OF IRON
• Iron is extracted from iron ore using a blast furnace.
• There are four types of iron ores; hematite (Fe2O3), magnetite (Fe3O4) , limonite (2Fe2O3∙3H2O)
and siderite (FeCO3). Hematite and magnetite are the commonest iron ores.
• Hematite contains iron(III) oxide (Fe2O3).
• Limestone (calcium carbonate) and coke (carbon) are required in iron extraction.
• Hematite, limestone and coke are loaded from the top of the blast furnace.
• The furnace is heated to a temperature over 1000oC.

❖ REACTIONS IN BLAST FURNACE


1. Oxygen in the air reacts with coke and form carbon dioxide.
C(s) + O2(g) → CO2(g)
2. Carbon dioxide reacts with coke and forms carbon monoxide.
CO2(g) + C(s) → 2CO(g)
3. Carbon monoxide reduces iron(III) oxide to iron. Iron in molten form sinks to the bottom
because of its high density.
Fe2O3(s) + 3CO(g) → 2Fe(l) + 3CO2(g)
4. Calcium carbonate in limestone undergoes thermal decomposition to calcium oxide and carbon
dioxide.
CaCO3(s) → CaO(s) + CO2(g)
5. Calcium oxide reacts with silicon dioxide, an impurity in iron ore and form calcium silicate
(slag). Slag formed floats on top of molten iron. Therefore, molten iron and slag can be drained
from the bottom of the furnace separately.
CaO(s) + SiO2(g) → CaSiO3(l)

❖ PRODUCTS
• Molten iron and slag are the necessary products from the blast furnace.
• Iron from the furnace is cooled into the iron cast.
• Iron casts are ready for use or alloyed into steel.
• Slag is used for road construction.

168 Metals
16.12 EXTRACTION OF COPPER
• Copper is extracted from copper ore using roasting in a furnace.
• There are various types of copper ore with different ore concentrations such as chalcopyrite
(CuFeS2), chalcocite (Cu2S), cuprite (Cu2O), bornite (Cu5FeS4) and malachite (Cu2CO3(OH)2).
• Chalcopyrite (also called copper pyrite) is the commonest copper ore. However, it is a low grade
copper ore. Therefore, the ore contains a low percentage of copper.

❖ EXTRACTION OF COPPER FROM CHALCOPYRITE


1. Chalcopyrite is roasted in the reverberatory furnace to produce copper (I) sulphide (Cu2S),
iron (II) sulphate (FeS). FeS further react with oxygen to form iron (II) oxide (FeO).
2CuFeS2(s) + O2(g) → Cu2S(s) + 2FeS(s) + SO2(g)
2FeS(s) + 3O2(g) → 2FeO(s) + 2SO2(g)
2. The ore is then reacted with silica in the blast furnace. During the process, FeO combines
with silica to form ferrous silicate (slag).
FeO(s) + SiO2(g) → FeSiO3(slag)
3. Slag forms an upper molten layer leaving a lower molten layer called copper matte. Copper
matte consists of Cu2S and FeS.
4. Copper matte is transported to the Bessemer converter. FeS is oxidised to FeO and further
combines with silica to form slag.
2FeS(l) + 3O2(g) → 2FeO(s) + 2SO2(g)
FeO(s) + SiO2(g) → FeSiO3(slag)
5. Cu2S is roasted in the air to form impure copper metal and sulphur dioxide.
Cu2S(l) + O2(g) → 2Cu(l) + SO2(g)
6. Copper is then purified by electrolysis.
7. Sulphur dioxide is used to make sulphuric acid.

❖ EXTRACTION OF COPPER FROM MALACHITE


1. Malachite is roasted and dried to concentrate the copper ore
2. Copper carbonate is thermally decomposed to form copper (II) oxide
CuCO3(s) → CuO(s) + CO2(s)
3. Copper is heated with coke in the furnace to form impure copper metal
2CuO(s) + C(s) → 2Cu(s) + CO2(g)
4. Copper is then purified by electrolysis

16.13 EXTRACTION OF ZINC


• Zinc is extracted from zinc ore called zinc blende
• Zinc blende contain zinc sulphide (ZnS)
• Extraction is done using the smelter

❖ REACTIONS
1. Zinc sulphide is heated in air to form zinc oxide and sulphur dioxide
ZnS(s) + 3O2(g) → 2ZnO(s) + 2SO2(g)
2. Oxygen in the air reacts with coke and form carbon dioxide
C(s) + O2(g) → CO2(g)
3. Carbon dioxide reacts with coke and form carbon monoxide
CO2(g) + C(s) → 2CO(g)
4. Zinc oxide is reduced by carbon monoxide to zinc and carbon dioxide
ZnO(s) + CO(g) → Zn(s) + CO2(g)

Metals 169
16.14 USES OF METALS
1. Copper is used for making electrical wires and cooking utensils because it is a good conductor of
heat and electricity.
2. Iron is used in building constructions because it has high tensile strength.
3. Gold, silver and platinum are used for making jewellery because they are luster and malleable.
4. Zinc is used as a protective coat of iron.
5. Aluminium is used in aircraft bodies because it is a light metal.

16.15 ALLOY
• Alloy is a mixture of two or more metals or metals and non-metals.
• Most alloys are a mixture of metals.
• A pure metal is too soft for use because layers of atoms slide over each other easily.
• Mixing two metals with different atom sizes decreases the sliding of atoms. Therefore, the
formed metal is strong and tough for everyday use.

❖ TYPES OF ALLOYS
1. STAINLESS STEEL
• Stainless steel is an alloy of iron, carbon, nickel and chromium.
• It is hard and resistant to corrosion from air and water.
• It is mainly used for cutleries such as knives, spoons and forks.

2. INVAR
• Invar is an alloy of iron and nickel.
• It expands very little when heated.
• It is used in temperature sensitive devices such as precision instruments, engine valves,
clock arms.

3. OTHER ALLOYS AND THEIR USES


1. STEEL (iron and carbon): building and home constructions, cutting tools.

2. BRONZE (copper and tin): statue and decorative objects.

3. BRASS (copper and zinc): water taps and door fittings.

4. GOLD WITH SILVER, COPPER OR TIN: pieces of jewellery.

170 Metals
15.16 RUSTING
• Iron reacts with water and oxygen and forms a solid brown product called rust. The process is
called rusting.
• Rust is a hydrated iron(III) oxide.
• Water and air are needed for rusting to occur.
4Fe(s) + 3O2(g) + 2H2O(l) → 2Fe2O3∙H2O(s)

❖ HOW TO PREVENT RUSTING


1. Surface protection
• Cover iron with a layer of another substance.
• These include painting, plastic grease or oil.

2. Sacrificial protection
• Coat iron with another metal.
• The coated metal must be more reactive than iron.
• Therefore, the coated metal corrodes first instead of iron.
• Galvanising is coating iron or steel with zinc.

3. Stainless steel
• Iron alloyed with less reactive metals.
• This causes iron to be more resistant to rusting.

Metals 171
REVIEW QUESTIONS
1. Which of the following is a property of metals?
A. They are good conductors of heat and electricity
B. They are malleable
C. They are ductile
D. All the above
2. Which of the following correctly lists the metals in decreasing order of reactivity?
A. Potassium, copper, magnesium, gold
B. Silver, sodium, aluminium, zinc
C. Sodium, magnesium, zinc, silver
D. Lead, calcium, copper, iron
3. Gold is often found in a pure state naturally because
A. It is very rare
B. It is very reactive
C. It is very unreactive
D. It is easily corroded
4. Metals react with oxygen to form
A. Metal oxide
B. Metal hydroxide
C. Salts
D. None of the above
5. Which of the following statements concerning the reaction of meals with water is correct?
A. Very reactive metals such as potassium and sodium react with cold water and form metal hydroxide
and hydrogen gas
B. Moderate react metals such as magnesium and aluminium react with steam and form a metal oxide
and hydrogen gas
C. Such metals such as gold and silver do not react with cold water or steam
D. All the above
6. Which of the following metal carbonate can be decomposed to a metal completely by heating?
A. Silver
B. Calcium
C. Potassium
D. Magnesium
7. What happens to iron(III) oxide in a blast furnace
A. It is oxidised by calcium carbonate to iron
B. It is reduced by carbon monoxide to iron
C. It is oxidised by calcium carbonate to iron
D. It is reduced by carbon monoxide to iron
8. Comparing pure iron and steel, pure iron is
A. Harder
B. Softer
C. Tougher
D. All the above
9. What two conditions are necessary for the rusting of iron?
A. Oxygen and hydrogen
B. Hydrogen and nitrogen
C. Oxygen and water
D. Water and hydrogen

172 Metals
10. Bronze is an alloy of
A. Copper and tin
B. Copper and iron
C. Copper and zinc
D. Copper and lead
11. Brass is an alloy of
A. Copper and tin
B. Copper and iron
C. Copper and zinc
D. Copper and lead
12. Which of the following is not a good method of preventing rust?
A. Painting
B. Greasing
C. Heating
D. Galvanising
13. Which alloy is mainly used in cutlery?
A. Brass
B. Bronze
C. Invar
D. Stainless steel
14. Zinc is used in galvanising iron than copper because
A. It is more reactive than iron
B. It is less reactive than iron
C. It does not corrode
D. None of the above
15. Iron is extracted using a blast furnace
A. Name two ores of iron
B. Write the formula of the two ores mentioned above
C. What two other substances are introduced in the blast furnace together with iron ore
D. Write a chemical equation for the reduction of any ore named above
E. What property does iron have for it to be used to make iron sheets
F. Explain why aluminium is extracted by electrolysis while iron is extracted by reduction with carbon
16. Metals have a wide range of properties
A. State any three physical properties and three chemical properties of metals.
B. Explain why metals are lustrous.
C. Two or more metals can be mixed together or mixed with non-metals
i. What name is given to a mixture of metals.
ii. Name any metallic mixture, state the elements present in it and its use.
iii. Give two reasons metals are mixed.
D. Metals can be extracted by electrolysis or chemical reduction. Name two metals that can be extracted
i. electrolysis
ii. chemical reduction
E. List three problems associated with the extraction of metals.
F. Give three advantages of metal recycling.

Metals 173
SOLUTIONS
1. D
2. C
3. C
4. A
5. D
6. A
7. D
8. B
9. C
10. A
11. C
12. C
13. D
14. A

15. A. Hematite and magnetite


B. Hematite; (Fe2O3)
Magnetite (Fe3O4)
C. Fe2O3(s) + 3CO(g) → 2Fe(l) + 3CO2(g)
E. It is highly malleable therefore can be easily hammered into thin sheets.
F. Aluminium is more reactive than carbon. It forms a stronger bond that carbon cannot displace.

16. A. Physical properties


i. metals have high melting and boiling point.
ii. metals are malleable.
iii. metals are ductile.

Chemical properties
i. metals react with oxygen to form metal oxide.
ii. metals react with dilute acids to form salt and hydrogen gas.
iii. metals react with water to form metal hydroxide or metal oxide and hydrogen gas.

B. Metals are lustrous because the surface of the metal reflects light which gives them a shiny
appearance.

C. i. Alloy
ii. Bronze. It contains a mixture of copper and tin. Bronze is used to decorate objects.
iii. To make objects attractive.
To protect objects from corrosion.

D. i. Aluminium and sodium


ii. Copper and zinc

E. It causes air, water and land pollution.


It produces noise pollution.
It causes physical disturbance of the landscape.

F. Preserve mineral resources.


Saves energy needed to produce metals.
Reduce air emission of pollutant gases.

174 Metals
17 ATMOSPHERE AND ENVIRONMENT

Introduction
The earth’s atmosphere contains a mixture of naturally occurring gases such as oxygen, carbon dioxide and
nitrogen. In addition to these gases, some harmful gases are introduced into the atmosphere such as carbon
monoxide and sulphur dioxide. These gases pose a threat to the globe. What are the sources of these gases?
How can we reduce the emission of these gases? This unit answers these questions.

Specific outcomes
By the end of this unit, you will be able to:
❖ State the composition of air
❖ Describe an experiment used to calculate the composition of oxygen in the air.
❖ State the source, harmful effect and prevention of the following pollutants:
• Carbon monoxide and carbon soot
• Sulphur dioxide
• Oxides of nitrogen
• Unburnt hydrocarbons
❖ Describe the effect of CFC on the ozone layer.
❖ Describe the greenhouse effect.

Atmosphere and Environment 175


ATMOSPHERE AND ENVIRONMENT
17.1 ATMOSPHERE
• The Earth’s atmosphere is made up of air containing a mixture of different gases.

❖ COMPOSITION OF AIR
Composition of air
Nitrogen 78%
Oxygen 21%
Argon 0.9%
Carbon dioxide 0.03%
Other 0.09%

❖ PERCENTAGE COMPOSITION OF OXYGEN IN AIR

Equation
Oxygen + copper → copper (II) oxide
O2(g) + 2Cu(s) → 2CuO(s)

• A known volume of gas in the gas syringe is passed over the burning copper powder.
• The burning copper uses oxygen in the air hence decreasing the volume of gas in the gas
syringes.
• The percentage composition of oxygen in the air can be calculated by finding the volume of
oxygen used and multiply it by 100%.

Volume of O2 = initial volume of air in gas syringes − final volume of air in gas syringes
volume of O2
Percentage composition of O2 = × 100%
initial volume of air

17.2 AIR POLLUTION


• Air pollution is the presence of harmful gases in the air that are dangerous and to organisms and
affect the climate.
• Air pollutants are harmful substances released into the air.
• The following are the main air pollutants and their effects on organisms and climate.
1. CARBON MONOXIDE AND CARBON SOOT
• Carbon monoxide (CO) is a toxic, colourless and odourless gas produced from the
incomplete combustion of fossil fuels.
• Carbon soot (C) are fine black powders produced from incomplete combustion.
• Both carbon monoxide and carbon soot are produced from incomplete combustion, the
process of burning fossil fuels with insufficient oxygen.
• With sufficient oxygen, fossil fuels are completely burnt to carbon dioxide and water. The
process of burning fossil fuels to carbon dioxide and water is known as complete
combustion. Example of complete combustion.
Methane + oxygen → carbon dioxide + water
CH4(g) + O2(g) → CO2(g) + 2H2O(l)

176 Atmosphere and Environment


• With insufficient oxygen, fossil fuels are incompletely burnt to carbon monoxide or carbon soot and
water. The process of burning fossil fuels to carbon monoxide or carbon soot and water is known as
incomplete combustion. Examples of incomplete combustion.
Methane + oxygen → carbon monoxide + water
2CH4(g) + 3O2(g) → 2CO(g) + 2H2O(l)
Methane + oxygen → carbon monoxide + water
CH4(g) + O2(g) → C(s) + 2H2O(l)

❖ HARMFUL EFFECTS OF CARBON MONOXIDE AND CARBON SOOT


i. Carbon monoxide is lighter than oxygen, hence combines faster with hemoglobin in RBC. This
causes oxygen deprivation in vital organs resulting in cell death.
ii. Carbon soot when inhaled irritates the nose, throat and lungs. They can cause lung damage and
lung cancer.

❖ PREVENTION
i. Install catalytic converter in cars. Catalytic converters reduce air pollutant emissions from car
engines by converting them to harmless substances. For example, it can convert carbon monoxide to
carbon dioxide.
ii. Install electrostatic precipitators in industries and factories to reduce the emission of carbon soot.

2. SULPHUR DIOXIDE
• Sulphur dioxide is a pungent and colourless gas.
• Fossil fuels contain sulphur elements.
• Burning of these fossil fuels combine sulphur and oxygen and form sulphur dioxide.
Sulphur + oxygen → sulphur dioxide
S(s) + O2(g) → SO2(g)
• Sulphur dioxide in the air dissolves in rainwater and form dilute sulphuric acid called acid rain.
Sulphur dioxide + oxygen → sulphur trioxide
SO2(s) + O2(g) → SO3(g)
Sulphur trioxide + water → sulphuric acid
SO3(g) + H2O(l) → H2SO4(aq)

❖ HARMFUL EFFECTS OF SULPHUR DIOXIDE


i. Inhaling sulphur dioxide causes lung irritation and eye irritation.
ii. Become acid rain with rainwater. Acid rain kills living organisms on land and in the aquatic.

❖ PREVENTION
i. Reduce using fossil fuels.
ii. Desulphurisation. The process of removing sulphur dioxide from smoke by treating it with
calcium carbonate.
Sulphur dioxide + calcium carbonate → calcium sulphate + carbon dioxide
SO2(g) + CaCO3(s) → CaSO3(s) + CO2(g)
Calcium sulphate can further be oxidised to produce gypsum. Gypsum is used to make
plasterboard for lining interior walls and ceilings.

Atmosphere and Environment 177


3. OXIDES OF NITROGEN (NITROGEN MONOXIDE AND NITROGEN DIOXIDE)
• Nitrogen combines with oxygen and form oxides of nitrogen (NOx).
• The commonest oxides of nitrogen are nitrogen monoxide (NO) and nitrogen dioxide (NO2).
• These compounds are formed at high temperature conditions such as in a combustion engine or
lightning.
Nitrogen + oxygen → nitrogen monoxide
N2(g) + O2(g) → 2NO(g)
Nitrogen monoxide + oxygen → nitrogen dioxide
2NO(g) + O2(g) → 2NO2(g)
• Oxides of nitrogen combines with rain water and form acid rain.
Nitrogen monoxide + oxygen → nitrogen dioxide
2NO(g) + O2(g) → 2NO2(g)
Nitrogen dioxide + water → nitric acid + nitrous acid
2NO2(g) + H2O(l) → HNO3(aq) + HNO2(aq)

❖ HARMFUL EFFECTS
i. Inhaling oxides of nitrogen irritate lungs.
ii. Become acid rain with rainwater. Acid rain kills living organisms on land and in the aquatic.

❖ PREVENTION
i. Install catalytic converter in cars. Catalytic converter can convert oxides of nitrogen to nitrogen
gas.

4. UNBURNT HYDROCARBONS AND OZONE


• With incomplete combustion of fuels, some hydrocarbons are released as unburnt.
• These hydrocarbons are part of volatile organic compounds (VOCs).
• The reaction of VOCs with nitrogen dioxide in the presence of sunlight produces photochemical
smog a brown haze containing ozone (O3) and other air pollutants.

❖ HARMFUL EFFECTS
i. Unburnt hydrocarbons are carcinogenic. They cause cancer.
ii. Photochemical smog causes eye, nose, throat and lung irritation.

❖ PREVENTION
i. Install catalytic converters in cars to prevent unburnt hydrocarbon release.
ii. Stop using chlorofluorocarbons (CFCs) products. These products deplete the ozone layer.

17.3 OZONE LAYER


• The Earth’s atmosphere is protected from ultraviolet (UV) radiation from the sun by the ozone
layer.
• The ozone layer is a thick layer found about 15 − 30 km from the earth’s surface.
• The ozone layer contains ozone (O3) molecules.
• It protects the earth by reflecting UV radiation from the sun.
• Substances such as chlorofluorocarbons (CFCs) deplete the ozone layer by reacting with ozone
molecules.
• These create holes in the ozone layer where excess UV radiation passes.
• UV radiations are harmful to organisms. To humans, it causes sunburn, skin cancer and rapid aging.
• Ozone layer depletion also affects agricultural productivity.
• Prevention: stop using chlorofluorocarbons (CFCs) products.

178 Atmosphere and Environment


17.4 GLOBAL WARMING
❖ GREENHOUSE EFFECT
• The greenhouse effect is the trapping of heat from sunlight by greenhouse gases causing an
increase in atmospheric temperature.
• Some greenhouse gases include carbon dioxide, methane, nitric oxide and CFCs.
• The heat from sunlight has to be reflected to space by the earth’s ground and other surfaces.
• However, greenhouse gases trap this heat from escaping back into space.
• This causes an increase in Earth’s atmospheric temperature and contributes to global warming.

❖ PREVENTION OF GLOBAL WARMING


i. Reduce fossil fuel burning to reduce carbon dioxide emissions.
ii. Use alternative energy sources such as hydro, solar and electricity.
iii. Animal manures and rotting vegetables produce methane. These can be used as biomass
fuel to prevent methane emissions.

Atmosphere and Environment 179


REVIEW QUESTIONS
1. Which gas makes up most of the Earth’s atmosphere?
A. Methane
B. Oxygen
C. Carbon dioxide
D. Nitrogen
2. What is the composition of oxygen in the atmosphere?
A. 0.9%
B. 78%
C. 21%
D. 0.03%
3. What is the composition of carbon dioxide in the atmosphere?
A. 0.9%
B. 78%
C. 21%
D. 0.03%
4. What is the composition of nitrogen in the atmosphere?
A. 0.9%
B. 78%
C. 21%
D. 0.03%
5. Air pollution causes
A. Soil erosion
B. Global warming
C. Respiratory problems
D. None of the above
6. Which of the following gases has the highest affinity for blood hemoglobin?
A. Nitrogen
B. Oxygen
C. Carbon dioxide
D. Carbon monoxide
7. Which of the following gases is produced from the incomplete combustion of fossil fuels?
A. Carbon dioxide
B. Carbon monoxide
C. Oxygen
D. Nitrogen
8. Which of the following is a greenhouse gas?
A. Nitrogen
B. Argon
C. Carbon dioxide
D. Oxygen
9. Which of the following can result in the formation of acid rains in the atmosphere?
A. Nitrogen oxides and sulphur oxides
B. Nitrogen and carbon monoxide
C. Carbon monoxide and carbon dioxide
D. Oxygen and argon
10. Photochemical smog is produced by the reaction of the following compounds in the presence of sunlight
A. Sulphur oxides and carbon dioxide
B. Nitrogen oxides and ground level ozone
C. Nitrogen oxides and volatile organic carbon compounds
D. Sulphur oxides and volatile organic carbon compounds

180 Atmosphere and Environment


11. The true statement about the greenhouse effect is that it is
A. Caused by a combination of many gases
B. Caused by CO2
C. Caused only by CO2, CFC, CH4 and NO2 gases
D. None of the above
12. Which of the following cause global warming?
A. Carbon dioxide
B. Oxygen
C. Nitrogen
D. Hydrogen
13. The ozone layer is mainly depleted by
A. Excess CO
B. Excess CO2
C. CFCs
D. Ozone

Atmosphere and Environment 181


SOLUTIONS
1. D
2. C
3. D
4. B
5. C
6. D
7. B
8. C
9. A
10. C
11. C
12. A
13. C

182 Atmosphere and Environment


18 ORGANIC CHEMISTRY

Introduction
Organic chemistry is simply the study of organic compounds. Organic compounds are compounds with
carbon−hydrogen bonds. The main element of organic compounds is carbon. However, organic compounds
usually have other elements such as hydrogen, oxygen, nitrogen, sulphur and phosphorus.

Organic compounds are very important in our daily lives. For example, petrol and diesel, the fuels we use to
power cars are naturally occurring organic compounds. A large portion of our body consists of organic
compounds. They are found in the food we eat. In addition, they can also be synthesized to form plastics and
clothes we wear.

This unit introduces you to organic chemistry. You will learn that organic compounds in the same group
have similar structures and properties.

Specific outcomes
By the end of this unit, you will be able to
❖ Define the following terms:
• Organic chemistry
• Organic compound
• Functional group
• Homologous series
❖ Name an organic compound
❖ Define the following terms:
• Hydrocarbon
• Saturated compound
• Unsaturated compound
❖ Alkanes:
• Describe alkanes
• State the reactions of alkanes
❖ Alkenes:
• Describe alkenes
• State the reactions of alkenes
❖ Alcohols:
• Describe alcohols
• Describe the formation of alcohols
• State the reactions of alcohols
❖ Carboxylic acids:
• Describe carboxylic acids
• Describe the formation of carboxylic acids
❖ Esters:
• Describe esters
• Describe the formation of esters

Organic Chemistry 183


ORGANIC CHEMISTRY
18.1 ORGANIC CHEMISTRY
• Organic chemistry is the study of the structure, properties, composition, reactions and synthesis of
organic compounds.
• Organic compounds are molecules composed of carbon and hydrogen. In addition, they can contain
other elements such as oxygen, nitrogen, sulphur and phosphorus.

❖ FUNCTIONAL GROUP
• A functional group is a group of atoms that is responsible for the chemical properties of
compounds.

❖ HOMOLOGOUS SERIES
• A homologous series is a group of organic compounds with the same functional group and
similar chemical properties.
• The compound in the same homologous series shows a trend in physical properties with the
increase in the number of carbon atoms or molecular mass.
• Homologous series studied at this level are alkanes, alkenes, alcohols, carboxylic acids and
esters. The table below shows the functional group of these compounds.

184 Organic Chemistry


❖ NAMING OF ORGANIC COMPOUNDS
• The suffix of an organic compound is a group of letters at the end of the molecule’s name.
• They indicate the type of functional group present in the organic compound reflecting the class to
which the molecule belongs.
• The table below shows the suffix and the corresponding class of organic compounds.
Suffix Class of organic compounds Example
-ane alkane methane
-ene alkene butene
-ol alcohol ethanol
-ioc acid Carboxylic acid Ethanoic acid

• The prefix of an organic compound is a group of letters at the beginning of the molecule’s name.
• They indicate the number of carbon atoms in the compound. For example, butane has four carbons;
ethanoic acid has two carbons; propene has three carbons.
• The table below shows the common prefixes.
Prefix Number of carbon atoms Example
1 Meth- Methane
2 Eth- Ethanol
3 Prop- Propanoic acid
4 But- Butane
5 Pent- Pentanol
6 Hex- Hexane
7 Hept- heptane
8 Oct- Octanoic acid
9 Non- Nonene
10 Dec- Decane

❖ HYDROCARBON COMPOUNDS
• Hydrocarbon compounds are compounds that contain only carbon and hydrogen atoms
• Alkanes and alkenes are hydrocarbon compounds

❖ SATURATED COMPOUNDS
• Saturated compounds are organic compounds that contain only single bonds
• All carbon atoms in a saturated compound are bonded to four hydrogen or carbon atoms
• Examples of saturated compounds; all alkanes are saturated compounds

❖ UNSATURATED COMPOUNDS
• Unsaturated compounds are organic compounds that contain a double bond
• Examples of unsaturated compounds; all alkenes are unsaturated

Organic Chemistry 185


18.2 ALKANES
• Alkanes are saturated hydrocarbon compounds.
• They have a general formula: CnH2n + 2, where n = 1, 2, 3, …
• Their names end with -ane.
• They have similar chemical properties.
• They show a gradual variation in physical properties with each increase in the number of carbon
atoms.
• They are insoluble in water.
• Alkanes with carbon 1 − 4 are gas, 5 − 15 are liquid and above 15 are solid at room temperature.
• The melting point and boiling point increases with the increase in the number of carbon atoms.
• Density increases with the increase in the number of carbon atoms.
• Flammability decreases with the decrease in the number of carbon atoms.
• Below are the names, chemical structure and chemical formula of the first five members of alkanes.

❖ REACTIONS OF ALKANES
1. COMBUSTION
• Alkanes burn in air with oxygen to form carbon dioxide and water.
C5H12(l) + 8O2(g) → 5CO2(g) → 6H2O(l)
• With insufficient oxygen, carbon monoxide and water are produced.
• Alkanes are usually used as fuels, for example, petrol and diesel.

2. REACTION WITH HALOGENS


• In the presence of ultraviolet light, alkanes react with halogens to form alkyl halides.
• This reaction is a substitution reaction. Halogen replaces hydrogen atom from the alkane.
CH4(g) + Br2(g) → CH3Br(g) + HBr(g)

❖ CRACKING OF ALKANES
• Long chain alkanes can be broken down into shorter chain alkanes and alkenes using heat. This
process is called cracking.
• The process is done at high temperature and pressure in the presence of zeolite (aluminium oxide
and silicon dioxide) catalyst.
• For example, decane can be broken down into pentane and pentene.
C10H22(l) → C5H12(l) + C5H10(l)
• Cracking is important in the formation of more useful smaller such as petrol and diesel from long
chained alkanes and the formation of alkenes.

186 Organic Chemistry


18.3 ALKENES
• Alkenes are unsaturated hydrocarbons.
• They have a general formula: CnH2n, where n = 2, 3, 4, …
• Their names end with -ene.
• They have similar chemical properties.
• They show a gradual variation in physical properties with each increase in the number of carbon
atoms.
• They are insoluble in water.
• Alkenes with carbon 1 − 4 are gas, 5 − 15 are liquid and above 15 are solid at room temperature.
• The melting point and boiling point increases with the increase in the number of carbon atoms.
• Density increases with the increase in the number of carbon atoms.
• Below are the names, chemical structure and chemical formula of the first four members of alkenes.

❖ USES OF ALKENES
• Alkenes are commonly used in industries as starting materials for the synthesis of other
substances.
• They are used in the synthesis of alcohol, acids, fuel and other substances such as plastics.

❖ REACTIONS OF ALKENES
1. COMBUSTION OF ALKENES
• Alkenes burn with sufficient oxygen to produce carbon dioxide and water.
• With insufficient oxygen, they produce carbon monoxide.
2C5H10(l) + 15O2(g) → 10CO2(g) + 10H2O(g)

2. HYDROGENATION
• Alkenes can be converted to alkanes by reacting them with hydrogen gas.
• This is an addition reaction. Hydrogen atoms are added to the C = C double bond.
• The process occurs at high temperature and pressure in the presence of nickel catalyst.
• Hydrogenation is used in the formation of margarine from vegetable oil.

Organic Chemistry 187


3. BROMINATION
• Bromination is the addition of bromine to the molecule.
• Bromine can be added to the C = C double bond of alkenes.
• Bromination is an addition reaction where a bromine molecule is added to the double of alkenes
• The process is used to test for unsaturated compounds.
• Mix the reddish-brown bromine water with an alkene. The decolourisation of bromine water shows
the presence of an alkene.

4. ADDITION OF WATER (HYDRATION)


• Alcohol can be produced by reacting alkene with steam.
• This is an addition reaction. The water molecule (H-OH) is added to the double bond.
• The process takes place at the temperature of 300oC, pressure of 60 atm and in the presence of
phosphoric acid catalyst.

5. POLYMERISATION
• Polymerisation is the process by which smaller molecules are chemically combined to form large
molecules.
• Smaller repeating molecules that are chemically combined to form a large molecule are called
monomers.
• A large molecule formed by polymerisation is called a polymer.
• Smaller alkenes can be combined chemically to form a large molecule.
• For example, thousands of ethene molecules can be joined together to make poly(ethene). Ethene is
a monomer and poly(ethene) a polymer.

• Importance of polymerisation
• Polymerisation is used in plastic formation.

188 Organic Chemistry


18.4 ALCOHOLS
• Alcohol is a homologous series with a general formula CnH2n + 1OH, where n = 1, 2, 3, …
• Alcohols have an −OH (hydroxyl) functional group.
• Their names end with −ol.
• The most important alcohol is ethanol.
• Below are the names, chemical structure and chemical formula of the first four members of
alcohols.

❖ PHYSICAL PROPERTIES OF ALCOHOL


i. They are colourless liquids with a characteristic odour.
ii. They are soluble in water.
iii. The boiling point increases with the increase in the number of carbons.

❖ USES OF ALCOHOL
i. Alcoholic drink
ii. Preservative
iii. Organic solvent
iv. Fuel

❖ FORMATION OF ALCOHOL
1. FERMENTATION OF SUGAR
• Alcohol is produced by fermenting sugar with yeast.
glucose → ethanol + carbon dioxide
C6H12O(aq) → 2C2H5OH(aq) + 2CO2(g)

2. REACTION OF ALKENE WITH STREAM


• The reaction between alkene and stream at 300oC and 60 atm in the presence of phosphoric
acid produce ethanol.
ethene + steam → ethanol
C2H4(g) + H2O(g) → C2H5OH(l)

Organic Chemistry 189


❖ REACTIONS OF ALCOHOL
1. COMBUSTION
• Alcohol burns in air to form carbon dioxide and water.
ethanol + oxygen → carbon dioxide + water
C2H5OH(aq) + 3O2(g) → 2CO2(g) + 3H2O(l)

2. OXIDATION
• Oxidation of alcohol produces carboxylic acid and water.
• Oxidising agents such as oxygen and acidified potassium dichromate oxidises alcohol to
carboxylic acid and water.
ethanol + oxygen → ethanoic acid + water
C2H5OH(aq) + O2(g) → CH3COOH(aq) + H2O(l)

ethanol + acidified potassium dichromate + sulphuric acid → ethanoic acid + water


K2Cr2O7
C2H5OH(aq) CH3COOH(aq) + H2O(l)
H2SO4

3. ESTERIFICATION
• The reaction between alcohol and carboxylic acids produces an ester (− O − group) and water.

4. REACTION WITH SODIUM


• When sodium is reacted with alcohol the reaction gives off hydrogen gas and sodium alkoxide
solution.
• For example, when a piece of sodium is dropped into ethanol. It reacts producing sodium ethoxide
and hydrogen gas.
Sodium + ethanol → sodium ethoxide + hydrogen gas
2Na(s) + 2C2H5OH → 2C2H5ONa(aq) + H2(g)

5. DEHYDRATION OF ALCOHOL
• When alcohol is heated with heated at 170oC with excess concentrated sulphuric acid it gives off
alkene and water.
• This reaction is an elimination reaction, where the water (H−OH) molecule is removed from the
alcohol.
• For example, ethanol gives ethene and water.

190 Organic Chemistry


18.5 CARBOXYLIC ACIDS
• Carboxylic acids are homologous series with a general formula CnH2n + 1COOH, where n = 0, 1, 2,...
• They have a carboxyl (−COOH) functional group.
• Their names end with −oic acid.

❖ PHYSICAL PROPERTIES OF CARBOXYLIC ACIDS


i. They are colourless liquids.
ii. They have a higher boiling point than other compounds with the same molar mass such as
alkanes, alkenes and alcohol. This is because the boiling point increases with the increase in the
carbon atoms.
iii. They are soluble in water. As the number of carbon atoms increases, the solubility decreases.

❖ PREPARATION OF CARBOXYLIC ACIDS


• Oxidation of alcohol produces carboxylic acid and water.
• Oxidising agents such as acidified potassium dichromate oxidises alcohol to carboxylic acid and
water.

ethanol + acidified potassium dichromate + sulphuric acid → ethanoic acid + water


K2Cr2O7
C2H5OH(aq) CH3COOH(aq) + H2O(l)
H2SO4

❖ CHEMICAL PROPERTIES OF CARBOXYLIC ACIDS


i. They are weak acids. They partially ionise in water.
CH3COOH(aq) → CH3COO−(aq) + H+(aq)
ii. They react with metals to form salt and water.
ethanoic acid + sodium → sodium ethanoate + hydrogen gas
2CH3COOH(aq) + 2Na(s) → 2CH3COONa(aq) + H2(g)
iii. They react with bases to form salt and water. This is a neutralisation process.
ethanoic acid + sodium hydroxide → sodium ethanoate + water
CH3COOH(aq) + NaOH(aq) → CH3COONa(aq) + H2O(l)
iv. They react with carbonates or bicarbonates to form salt, carbon dioxide and hydrogen gas.
ethanoic acid + sodium carbonate → sodium ethanoate + carbon dioxide + water
2CH3COOH(aq) + Na2CO3(aq) → CH3COONa(aq) + CO2(g) + H2O(l)
v. They react with aqueous ammonia to form ammonium salt.
ethanoic acid + ammonia → ammonium ethanoate
CH3COOH(aq) + NH3(aq) → CH3COONH4(aq)

Organic Chemistry 191


18.6 ESTERS
• Esters are organic compounds made from the reaction between carboxylic acid and alcohol.
• The process by which esters are formed is called esterification.
• Concentrated sulphuric acid is used as a catalyst.
• General formula: carboxylic acid + alcohol → ester + water.
• For example, ethanoic acid reacts with ethanol in the presence of concentrated sulphuric acid and
forms ethyl ethanoate and water.

192 Organic Chemistry


REVIEW QUESTIONS
1. Hydrocarbon compounds contain only
A. Carbon and hydrogen
B. Carbon and oxygen
C. Hydrogen and oxygen
D. Carbon, hydrogen and oxygen
2. A group of atoms that determine the family name and chemical reactivity of the organic compound it is
found is called
A. Identifying group
B. Functional group
C. Organic group
D. Compound group
3. Which of the following statement is true concerning unsaturated compounds?
A. Compounds that contain single bonds only
B. Compounds that contain at least one double bond
C. Compounds composed of carbon and hydrogen only
D. Compounds composed of carbon, hydrogen and oxygen
4. The distinction between saturated hydrocarbons and unsaturated hydrocarbon relates to
A. Solubility in water
B. Melting point
C. The presence of single bond or double bonds
D. The number of carbon and hydrogen present
5. Which of the following is not typical of most organic compounds?
A. High flammability
B. Poor solubility in water
C. Covalent bonding
D. High boiling point
6. Isomers are compounds with the same
A. Molecular formula but different structure
B. Structure but the different molecular formula
C. Different structures and a different molecular formula
D. Molecular formula but a different atomic mass
7. Which statement is true concerning isomers?
A. They have the same chemical and physical properties
B. They have the same chemical properties but different physical properties
C. They have the same physical properties but different chemical properties
D. They have different chemical and physical properties
8. What is the name of the four-carbon straight chain alkane compound?
A. Pentane
B. Hexane
C. Methane
D. Butane
9. What is the name of the one-carbon alkane compound?
A. Methane
B. Octane
C. Propane
D. Butane

Organic Chemistry 193


10. Which is the correct order of melting points of alkenes, from the lowest to the highest
A. Decane > butane > hexane > methane
B. Hexane > methane > decane > butane
C. Methane > butane > hexane > decane
D. Butane > hexane > decane > methane
11. Alkenes contain at least one of the following bonds
A. Single bond
B. Double bond
C. Triple bond
D. Hydrogen bond
12. What is the name of the two-carbon straight chain alkene compound?
A. Propane
B. Propene
C. Ethane
D. Ethene
13. What is the name of the six-carbon straight chain alkene compound?
A. Hexane
B. Hexene
C. Ethane
D. Ethene
14. The process by which large hydrocarbons are converted into smaller ones is called
A. Fermentation
B. Dehydration
C. Cracking
D. Dehydrogenation
15. The reaction of an alkene with bromine to produce an alkyl bromide is called
A. Bromination
B. Hydrolysis
C. Hydration
D. Condensation
16. The reaction between bromine and alkene is an example of
A. Condensation reaction
B. Addition reaction
C. Combustion
D. Substitution reaction
17. The functional group of alcohol is called
A. Alkyl
B. Ester
C. Hydroxyl
D. Carbonyl
18. Which of the following types of compounds is produced by the hydration of alkenes?
A. Alcohol
B. Alkene
C. Carboxylic acid
D. Ester
19. The addition reaction of alkene and hydrogen produce what type of compound?
A. Alcohol
B. Carboxylic acid
C. Alkane
D. Ester

194 Organic Chemistry


20. The process by which smaller alkene units are made into a larger molecule is called
A. Halogenation
B. Hydrogenation
C. Fermentation
D. Polymerisation
21. When alcohol is reacted with acidified potassium dichromate in the presence of concentrated sulphuric acid
which of the following reaction type is likely to occur
A. Dehydration
B. Oxidation
C. Substitution
D. Hydration
22. Which of the following types of compounds is produced by the dehydration of alcohol?
A. Alkene
B. alkane
C. Carboxylic acid
D. Ester
23. The functional group of the carboxylic acid is called
A. Alkyl
B. Ester
C. Hydroxyl
D. Carbonyl
24. Which of the following is the functional group in carboxylic acids?
A. −OH
B. −CO
C. −COOH
D. −O−
25. Which of the following types of compounds is produced by the reaction of alcohol and carboxylic acid?
A. alkane
B. Alkene
C. Polyethene
D. Ester
26. Which of the following is the functional group in an ester?
A. −COOH
B. −C−O−C−
C. −OH
D. C=O
27. Which of the following is the correct ending for the name of an ester?
A. −oate
B. −oic
C. −ol
D. −ene
28. Methanoic acid is a member of a homologous series of compounds called carboxylic acids. It is
characteristic of any homologous series that all the members have the same
A. General formula
B. Physical properties
C. Relative molecular mass
D. Structural formula

Organic Chemistry 195


29. Which catalyst is used in the preparation of ethyl ethanoate from ethanol and ethanoic acid?
A. Concentrated sulphuric acid
B. Nickel
C. Phosphoric acid
D. Yeast

30. A. The following compounds are members of the homologous series of alkenes.
CH4, C2H6, C3H8, C4H10, and C5H12
i. State the formulae of the next two members in the series.
ii. State the reason whether the compound C9H18 belongs to the family of alkanes or not.
iii. Among the five members given state with the reason the one with the highest boiling point.

B. The demand for petrol worldwide increases every year. To meet the demand, large molecules of
hydrocarbons are cracked.
i. What do you understand by the term cracking?
ii. State two types of cracking.
iii. Write down the two possible products when the compound C11H24 is cracked.

31. A. Name the organic compounds with the structural formulae shown below.

B. A hydrocarbon belonging to the alkane family has the molecular formula C4Hx, where x is the
number of hydrogen atoms in one molecule of the hydrocarbon.
i. State the general formula of the alkanes.
ii. Use the general formula to find the value of x in C4Hx and state the name of the hydrocarbon.
iii. Draw the structures of the two isomers of the above hydrocarbon.

32. 4.2g of a gaseous hydrocarbon Z was analysed and was found to contain 3.6g carbon and 0.6g of hydrogen.
The relative formula mass of hydrocarbon Z is 28
A. Using the information provided above deduce the molecular formula of the hydrocarbon Z
B. Draw the molecular structure of hydrocarbon Z
C. i. To which homologous series does hydrocarbon Z belong?
ii. Write the general formula of the homologous series to which hydrocarbon Z belongs
D. The analysed hydrocarbon was later burn in limited supply in the air although such a reaction is
dangerous
i. Explain why the burning of hydrocarbon Z in a limited supply of air is dangerous
ii. Write the balanced chemical equation for the burning of hydrocarbon Z in a limited supply of
air. Include the state symbols
E. Can hydrocarbon Z be polymerised? Give a reason for your answer

196 Organic Chemistry


F. Hydrocarbon Z was bubbled into aqueous bromine and acidified potassium manganate (VII).
State what was observed when
i. hydrocarbon Z was bubbled into aqueous bromine
ii. hydrocarbon Z was bubbled into acidified potassium manganate (VII)

33. An ester is formed from carboxylic acid and alcohol. The carboxylic acid has the molecular formula C4H8O2
A. i. Name the carboxylic acid
ii. Draw the structural formula of the carboxylic acid
iii. What is the empirical formula of a carboxylic acid?

B. Analysis of the alcohol shows it has the following percentage composition by mass, 52.2% carbon,
13.0% hydrogen and 34.8% oxygen
i. Work out the empirical and molecular formulae of the alcohol (Mr = 46)
ii. Draw the full structural formula of the alcohol and state its name
iii. Write a chemical equation of the reaction between the carboxylic acid in A.i. and alcohol in
B.ii. using structural diagram

34. The structural formula of an ester is given below

A. Describe how the above ester can be prepared in the laboratory by naming the reagents and the
conditions needed for a successful reaction
B. What observation will confirm that an ester has been formed in the rection?

Organic Chemistry 197


SOLUTIONS
1. A
2. B
3. B
4. C
5. D
6. A
7. D
8. D
9. A
10. C
11. B
12. D
13. B
14. C
15. A
16. B
17. C
18. A
19. C
20. D
21. B
22. A
23. D
24. C
25. D
26. B
27. A
28. A
29. A

30. A. i. C6H14
ii. C9H18 does not belong to the alkane family because it lacks two hydrogen atoms.
iii. C5H12 has the highest boiling because of the strong force of attraction between molecules.

B. i. Cracking is the process of breaking down longer hydrocarbons into shorter ones using heat.
ii. Catalytic cracking and steam cracking.
iii. C6H14 and C5H10

31. A. i. Propanol
ii. Ethanoic acid
iii. Polyvinyl chloride

B. i. CnH2n + 2
ii. x = 10
name of hydrocarbon; butane

iii.

198 Organic Chemistry


mass given Mole = ?
32. A. n (C) =
molar mass Mass given (C) = 3.6g
3.6 Mr (C) = 12 g/mol
=
12
= 0.3 mol

mass given Mole = ?


n (H) =
molar mass Mass given (H) = 0.6
0.6 Mr (H) = 1 g/mol
=
1
= 0.6 mol

mol of Mg mol of C
Ratio of C = Ratio of H =
smallest mol smallest mol
0.3 0.6
= =
0.3 0.3
=1 =2

Empirical formula = C1H2


= CH2

molar mass
n=
empirical mass
28
n=
13
n=2

Molecular formula = empirical formula × n


= (CH2) × 2
= C2H4

B.

C. i. alkane
ii. CnH2n

D. i. Burning hydrocarbon Z in limited air produces carbon monoxide that causes suffocation.
ii. C2H4(g) + 2O2(g) → 2CO(g) + 2H2O(g)

E. Yes, because hydrocarbon Z is unsaturated.

F. i. Reddish-brown bromine solution decolourises.


ii. The purple potassium permanganate (VII) changes brown.

Organic Chemistry 199


33. A. i. Butanoic acid

ii.

iii. C2H4O

% given Mole = ?
B. i. n (C) =
molar mass % (C) = 52.2%
52.2 Mr (C) = 12 g/mol
=
12
= 4.35 mol

% given Mole = ?
n (C) =
molar mass % (H) = 13%
13.0 Mr (H) = 1 g/mol
=
1
= 13 mol

% given Mole = ?
n (O) =
molar mass % (O) = 53.01%
34.8 Mr (O) = 16 g/mol
=
16
= 2.175 mol

mol of C mol of H mol of O


Ratio of C = Ratio of H = Ratio of O =
smallest mol smallest mol smallest mol
4.35 13 2.175
Ratio of C = Ratio of H = Ratio of O =
2.175 2.175 2.175
Ratio of C = 2 Ratio of H = 6 Ratio of O = 1

Empirical formula = C2H6O1


= C2H6O

molar mass
n=
empirical mass
46
n=
46
n=1

Molecular formula = empirical formula × n


= (C2H6O) × 1
= C2H6O

200 Organic Chemistry


B. ii. Structural formula;

iii. Name; ethanol

iii.

34. A. The reaction can be prepared by reacting propanol and ethanoic acid in the presence of concentrated
sulphuric acid.
B. A pleasant smell of ester.

Organic Chemistry 201


19 MACROMOLECULES

Introduction
Plastics play essential roles in our daily lives. Think of plastic bags, plastic containers, plastic housing
smartphones and computers, pens, and some clothes. All these materials consist of larger molecules that were
made by chemically joining smaller molecules. These larger molecules are called macromolecules. Some
macromolecules are naturally occurring while others are synthesized. The commonest naturally occurring
macromolecules are carbohydrates, proteins and fats, the nutrients found in most food we eat.

Specific outcomes
This unit covers common synthetic macromolecules. By the end of this unit, you will be able to:
❖ Define the following terms:
• Macromolecule
• Monomer
• Polymer
• Polymerisation
❖ Differentiate addition polymerisation and substitution polymerisation
❖ Write the structure and state the uses of the following synthetic macromolecules:
• Nylon
• Terylene
❖ State examples of natural macromolecules

202 Macromolecules
MACROMOLECULES
19.1 MACROMOLECULES
• Macromolecules are large molecules made by chemically joining together many small molecules.
• Smaller molecules that are chemically joined are called monomers.
• A large molecule formed by chemically joining smaller molecules is called a polymer.
• Polymerisation is the addition of monomers to form one large polymer.

❖ TYPES OF POLYMERISATION
1. ADDITION POLYMERISATION
• Addition polymerisation is the formation of a polymer by joining monomers without the
production of other substances.
• In addition polymerisation, no atoms are lost.
• The process occurs with unsaturated monomers such as alkenes.
• For example, thousands of ethene molecules can be joined together by addition
polymerisation to make poly(ethene). Ethene is a monomer and poly(ethene) a polymer.
• Poly(ethene) is used to make plastics.

2. CONDENSATION POLYMERISATION
• Condensation polymerisation is the formation of polymers by joining of monomer with the
elimination of smaller molecules.
• In condensation polymerisation, each time a bond is formed between two monomers a
smaller molecule is lost usually water.
• Common polymers formed by condensation polymerisation are nylon and terylene.

❖ NYLON
• Nylon is formed by polymerisation of dicarboxylic acid (adipic acid) and diamine
(hexamethylene diamine).
• The linkage between the monomers is called an amide linkage. Therefore, the polymer
is called polyamide.
• The formation of amide results in a loss of water.

Macromolecules 203
❖ USES OF NYLON
i. Making clothing
ii. Fishing nets
iii. Rugs and carpets
iv. Parachutes

❖ TERYLENE
• Terylene is formed by polymerisation of dicarboxylic acid (terephthalic acid) and diol (ethylene
glycol: alcohol with 2 −OH groups).
• The linkage between the monomers is called an ester linkage. Therefore, the polymer is called
polyester.
• The formation of amide results in a loss of water.

❖ USES OF TERYLENE
i. Making clothing
ii. Plastic bottles
iii. Safety belts and ropes

19.2 NATURAL MACROMOLECULES


1. CARBOHYDRATES
• Carbohydrates contain carbon, hydrogen and oxygen.
• They have a general formula Cn(H2O)n
• The monomer of carbohydrates is glucose.
• Condensation polymerisation of glucose forms starch.
• Starch can be broken down into glucose by the process called hydrolysis reaction.

2. PROTEINS
• Proteins contain carbon, hydrogen, oxygen, nitrogen and sometimes sulphur.
• The monomers of proteins are amino acids.
• Condensation polymerisation of amino acids produces proteins.
• The peptide bond between two amino acids is similar to the amide linkage found in nylon.
• Proteins can be broken down into amino acids by the process called hydrolysis reaction.

204 Macromolecules
3. FATS
• Fats contain carbon, hydrogen and oxygen.
• The monomers of fats are fatty acids and glycerol.
• A single fat molecule contains one glycerol and three fatty acids.
• The bond between a fatty acid and glycerol is similar to the ester linkage found in terylene.
• Fats can be broken down into fatty acids and glycerol by hydrolysis reaction.

Macromolecules 205
REVIEW QUESTIONS
1. The process by which smaller molecule units are chemically joined to form larger molecules is called?
A. Cracking
B. Polymerisation
C. Esterification
D. Hydrolysis
2. Smaller molecule units that are reacted to form a large molecule are called?
A. Monomer
B. Polymer
C. Ester
D. Amide
3. A large molecule formed by reacting smaller units is known as
A. Alkane
B. Monomer
C. Ester
D. Polymer
4. By what reaction type is a polymer formed from monomers
A. Addition reaction
B. Substitution reaction
C. Condensation reaction
D. Hydrolysis
5. By what reaction type is a polymer broken down into monomers
A. Elimination reaction
B. Substitution reaction
C. Condensation reaction
D. Hydrolysis
6. A type of polymerisation which involves the formation of a large molecule from smaller molecules
without the production of other molecules is known as
A. Substitution condensation
B. Condensation polymerisation
C. Addition condensation
D. Elimination polymerisation
7. A type of polymerisation which involves the formation of a large molecule from smaller molecules with
the production of other molecules such as water is known as
A. Substitution condensation
B. Condensation polymerisation
C. Addition condensation
D. Elimination polymerisation
8. Which of the following does not describe a polymer?
A. Polymers are large molecules
B. Polymers are made of monomers
C. Polymers are broken by the process of hydrogenation
D. Polymers usually form by covalent bond
9. A polymer formed by polymerisation of ethene is known as
A. Terylene
B. Nylon
C. Bakelite
D. Polyethene

206 Macromolecules
10. PVC (polyvinyl chloride) is prepared by the polymerisation
A. propene
B. 1-fluoroethene
C. 1-chloroethene
D. Ethene
11. A polymer formed by polymerisation of dicarboxylic acid and diamine is known as
A. Polyvinyl chloride
B. Nylon
C. Bakelite
D. Terylene
12. A polymer formed by polymerisation of dicarboxylic acid and diol is known as
A. Teflon
B. Nylon
C. Polyvinyl chloride
D. Terylene
13. Which of the following plastics is thermally stable?
A. Polyethene
B. Polypropene
C. Polyvinyl chloride
D. Polytetrafluoroethylene
14. The basic units of proteins are
A. Fatty acids and glycerol
B. Amino acids
C. Glucose
D. Nucleic acids
15. The basic units of carbohydrates are
A. Fatty acids and glycerol
B. Amino acids
C. Glucose
D. Nucleic acids
16. The basic units of fats are
A. Fatty acids and glycerol
B. Amino acids
C. Glucose
D. Nucleic acids
17. Which of the following can be used to obtain amino acids from proteins?
A. Condensation
B. Decomposition
C. Hydration
D. Hydrolysis
18. Terylene is a polyester formed from carboxylic acid and alcohol units
A. Name two monomers used to make terylene
B. Draw the structural formula of terylene showing only four monomer units
C. State two uses of terylene
19. Nylon is a polyamide used to make clothing material
A. Name the two monomers used to make nylon
B. Draw part of the structure of nylon showing at least three monomer units
C. Which natural macromolecule contains the amide linkages?

Macromolecules 207
20. The structure of tetrafluoroethylene is shown below

A. Draw the structure of the polymer formed when tetrafluoroethylene molecules polymerise (show
at least four monomer units). Give the name of the polymer.
B. What type of polymerisation is involved in the formation of the polymer above?
C. When 10 tonnes of tetrafluoroethylene polymerised, 9 tonnes of the polymer were obtained.
Calculate the percentage yield.
D. Name another commercially available polymer containing a halogen and state its uses.

208 Macromolecules
SOLUTIONS
1. B
2. A
3. D
4. C
5. D
6. C
7. B
8. C
9. D
10. C
11. B
12. D
13. D
14. B
15. C
16. A
17. D

18. A. dicarboxylic acid and diol

B.

C. Uses of terylene
i. making clothing materials
ii. making plastic bottles

19. A. dicarboxylic acid and diamine

B.

C. Protein

20. A. Structure;

B. Addition polymerisation
actual yield
C. Percentage yield = × 100%
theoretical yield
9
= × 100%
10
= 90%

D. Polyvinyl chloride
Uses of polyvinyl chloride
i. Pipes
ii. Cables
iii. Plastic bottles
iv. Window frames
v. Plastic tubes

Macromolecules 209

You might also like